Entire Course (including glossary) (2024)

Table of Contents
Chapter 2. Income Form 1041 Page 1 Comparison to Form 1040 General Correction of Erroneous Form 1099 Reporting Correction of Erroneous Form 1099 Reporting (cont'd) Note Municipal Bond Interest Income Classification Adjusted Gross Income Allowable Trade or Business Expenses Expenses That Offset Income Proprietorship (Schedule C) Proprietorship (Schedule C) (cont'd) Farm (Schedule F) Note Qualified Business Income (QBI) Material Participation Issues Technical Advice Memorandum 201317010 Mattie K. Carter Trust v. U.S., 91 AFTR 2d 2003-1946 (2003) Frank Aragona Trust v. Commissioner, 142 T.C. No. 9 (2014) Frank Aragona Trust v. Commissioner, 142 T.C. No. 9 (2014) (cont'd) Frank Aragona Trust v. Commissioner, 142 T.C. No. 9 (2014) (cont'd) Material Participation Tests Note Depreciation, Depletion and Amortization Estates Estates (cont'd) Trusts Flow-Through Entities IRC Section 179 Trusts and Estates Grantor Trusts Note Note Study Question 5 Capital Gains or Losses (Form 1041, Schedule D) Note Basis of Property Acquired from Decedent Alternate Valuation Modified Carryover Basis for 2010 Note Consistent Basis Rules Issues Involving Brokerage Firms Holding Period Decedent's Personal Residence Decedent's Personal Residence (cont'd) Other Assets Note Tax Rates Principal and Income Acts Note Rental Income General Real Estate Professional Real Estate Professional (cont'd) Pass-through Income Partnerships Basis in Partnership Interest Note Losses S Corporations Estates Holding S Corporation Stock Note Grantor Trusts as S Shareholders Grantor Trusts as S Shareholders (cont'd) Testamentary Trusts as S Shareholders for Two-Year Period Note Voting Trusts as S Shareholders Voting Trusts as S Shareholders (cont'd) Qualified Subchapter S Trusts (QSSTs) QSST Election Note QSST Election (cont'd) Election Requirements Election Requirements (cont'd) Election Requirements (cont'd) Note Electing Small Business Trusts (ESBTs) Electing Small Business Trusts (ESBTs) (cont'd) Electing Small Business Trusts (ESBTs) (cont'd) Electing Small Business Trusts (ESBTs) (cont'd) Electing Small Business Trusts (ESBTs) (cont'd) Electing Small Business Trusts (ESBTs) (cont'd) Note Distributions of Passive Activities Study Question 6 General Compensation of the Decedent Annuities and Retirement Accounts Annuities and Retirement Accounts (cont'd) Chapter 3. Deductions General Note Note Qualified Residence Interest Qualified Residence of Beneficiary Decedent's Residence Note Requirements Investment Interest Investment Income Investment Interest (cont'd) Note Deductions Note Reporting Study Question 7 General Deductible Taxes Nondeductible Taxes Reporting State and Local Income Taxes State and Local Sales Taxes Property Taxes Property Taxes Related to Trade or Business Decedent's Accrued Property Taxes Study Question 8 Fiduciary Fees (cont'd) General Authorization to Make Charitable Contribution Contributions Made From Amounts Included in Gross Income Gross Income Defined Note Tax-Exempt Income Election to Deduct in Year Prior to Contribution Election Deadline Note Election Procedure Estate Permanent Set Aside Provision Note Note Special Final Year of Trust or Estate Rule Reporting Distributions to Charitable Beneficiaries Study Question 9 General Carryovers Carryovers (cont'd) Note NOL Reduces Distributable Net Income Procedure Procedure (cont'd) Note NOL – Excess Deductions on Termination of an Estate or Trust General Expenses Directly Allocable to Income Expenses Not Directly Allocable to Income Note Allocation Based on Gross Income Allocation Based on Value of Assets Generating Taxable and Tax-Exempt Income Allocation Based on Income Types Over the Life of the Trust Tax Return and Recordkeeping Requirements Adjusted Gross Income of Trust or Estate Adjusted Gross Income of Trust or Estate (cont'd) Note Expenses Customarily or Ordinarily Incurred by an Individual Expenses Customarily or Ordinarily Incurred by an Individual (cont'd) Specific Costs Addressed in Reg. §1.67-4(d) Ownership Costs Note Tax Preparation Fees Investment Advisory Fees Investment Advisory Fees (cont'd) Note Appraisal Fees Other Costs Identified by the IRS as Not Subject to the 2% Limit Bundled Fees Hourly Versus Non-Hourly Fees in Bundled Fees Expenses Specifically Excluded From Allocation Reasonable Method of Allocation Election to Deduct Administrative Expenses on Form 1041 for an Estate Election to Deduct Administrative Expenses on Form 1041 for an Estate (cont'd) Note Qualified Business Income Deduction Chapter 4. Accounting Income and the Applicable Principal and Income Act Accounting Income Accounting Income versus Taxable Income Definition States That Have Not Yet Adopted States That Have Not Yet Adopted (Cont'd) States That Have Not Yet Adopted 2008 Amendments States That Have Not Yet Adopted 2008 Amendments (cont'd) States That Have Adopted With Unique Amendments Focus On Uniform Act and 2008 Amendments Fiduciary Income and Principal Act of 2018 Superior Status of Definitions in Trust Applicable State Act Trust Principal Trust Income Note Study Question 10 Study Question 11 General Default Allocation Note Allocation of Receipts Funds Received from Entities Exclusions General Rule Mutual Funds Note Liquidations of Trust's Interest in Entity Tests for Partial Liquidation Distributions from Trusts or Estates Distributions from Trusts or Estates (cont'd) Business Conducted by Trust Option to Maintain Separate Accounting Records Eligible Activities Items Received from Trustor or Decedent Payments Received Income Interest Received from Estate Proceeds from Sale of Assets Note Rental Property Note Receipts from Bonds and Other Types of Debt Instruments Receipts from Bonds and Other Types of Debt Instruments (cont'd) Proceeds of Insurance Policies and Similar Contracts Deferred Compensation Special Rules Qualified Terminable Interest Property (QTIP) Power of Appointment Note Liquidating Assets Interests in Natural Resources Interests in Timber Interests in Timber (cont'd) Property Not Productive of Income Property Not Productive of Income (cont'd) Derivatives and Options Note Asset-Backed Securities Asset-Backed Securities (cont'd) Note Insubstantial Allocations General Trustee, Investment Advisory, and Custodial Service Compensation Regular Compensation Accountings, Judicial Proceedings, and Other Matters Expenditures for Expenses Incurred in Administration, Management, or Preservation of Trust Property Insurance Transfer Taxes Debt Payments Environmental Expenses Income Taxes Taxes Related to Pass-Through Income Adjustments Based on Taxes Depreciation Trustee' Power to Reallocate Study Question 12 Chapter 5. Income Distribution Deduction (Sch. B) Distributable Net Income (DNI) Calculation of Distributable Net Income Calculation of Distributable Net Income (cont'd) Note Study Question 13 Income Distribution Deduction for Simple Trusts Note Example 5.2 Income Distribution Deduction for Estates and Complex Trusts Note Example 5.3 Study Question 14 Chapter 6. Taxation of Distributions to Beneficiaries (Schedule K-1) Differences between Trust K-1s and Other Types of K-1s Distributions to Beneficiaries Amounts Required to Be Distributed to the Beneficiary Amounts Otherwise Properly Paid, Credited, or Required to Be Distributed Exception for Specific Gifts and Bequests Exclusions Study Question 15 Election to Treat Distributions as Made in Preceding Year Note Election to Treat Distributions as Made in Preceding Year (cont'd) Election to Treat Distributions as Made in Preceding Year (cont'd) Note Allocation of Distribution Deduction to Beneficiaries Specific Allocations of Trust/Estate Income Note Allocation of Deductions to Types of Income Note Allocation of Deductions to Types of Income (cont'd) Note Special Rules for Allocation of Charitable Contributions Two-Tier Allocation for Complex Trusts and Estates Tier 1 Mandatory Distributions Tier 2 Distributions Study Question 16 Final Year Deductions Excess Deductions on Termination Note Capital Loss Carryover Net Operating Loss Carryover Allocation of Final Year Deductions Alternative Minimum Tax Adjustment Tax Credits Tax Credits (cont'd) Other Information Chapter 7. Tax Computation Fiduciary Income Tax Brackets Study Question 17 Personal Exemptions Alternative Minimum Tax (Schedule I, Form 1041) Computation of Alternative Minimum Taxable Income Computation of Alternative Minimum Taxable Income (cont'd) Computation of Alternative Minimum Taxable Income (cont'd) Alternative Minimum Tax Rates for Trusts and Estates Study Question 18 Study Question 19 General Foreign Tax Credit General Business Credit Note Prior Year Minimum Tax Credit Deferral Preferences and Adjustments Exclusion Preferences and Adjustments Calculation General Net Investment Income Exempt Income Note Undistributed Net Investment Income Study Question 20 Study Question 21 Adjusted Gross Income Limitation Adjusted Gross Income Limitation (cont'd) Adjusted Gross Income Limitation (cont'd) Special Rule for Bankruptcy Estates Exempted Trusts and Estates Purely Charitable Trusts Charitable Purpose Estates Trusts Exempt from Tax Charitable Remainder Trusts Trusts Exempt from Tax by Statute Grantor Trusts Foreign Estates Foreign Trusts Common Trust Funds Designated Settlement Funds Cemetery Perpetual Care Funds Electing Alaska Native Settlement Trusts Trusts Not Exempted Computing the Tax Net Investment Income Tax and Electing Small Business Trusts Net Investment Income Tax and Electing Small Business Trusts (cont'd) General Liability for Tax Wages Paid by Estate/Trust and Decedent/Beneficiary Chapter 8. Penalties Underpayment of Estimated Income Tax General Rule for Required Payments General Rule for Required Payments (cont'd) Note Due Dates for Estimated Taxes Exception for Initial Years of Estate or Qualified Revocable Trust No Tax Liability in Prior 12 Months Penalty Calculation Failure to Pay Tax When Due Failure to File Tax Return (Late Filing) Penalty Note Failure to Provide Information Timely Study Question 22 General Substantial Understatement of Income Tax Defined Burden of Proof Exceptions Note Negligence or Disregard of the Rules or Regulations Note Consistent Basis Penalty Study Question 23 Chapter 9. Extended Returns, Amended Returns and Claims for Refund General Form 7004 Note 10-Day Grace Period for Termination of Extension Note Study Question 24 General Note Statute of Limitations Issues Note Claims for Refund Net Operating Loss Carryback Note Bad Debt or Worthless Security Tentative Refund Study Question 25 General Note Substantial Understatement Note Substantial Understatement (cont'd) Note Statute Mitigation Provisions Statute Mitigation Provisions (cont'd) Request for Prompt Assessment of Taxes Chapter 10. Grantor Trusts General Grantor Defined Power to Revoke the Trust or Return Corpus to the Grantor [IRC §676] Managerial Powers Power to Lend Money to Grantor Adverse Interest Note Power to Distribute Income to or for the Benefit of the Grantor [IRC §677] Conditional Distributions Possible Inheritance Insurance Note Grantor Retains a More than 5% Reversionary Interest in Trust Property or Income [IRC §673] Exception for Reversion upon Death of Minor Descendant Intestacy Postponed Date for Reacquisition Determining Value of Reversionary Interest Reversionary Interest of 10 Years or Less Retained by Grantor (Pre-March 2, 1986 Trusts) [IRC §673] Reversionary Interest of 10 Years or Less Retained by Grantor (Pre-March 2, 1986 Trusts) [IRC §673] (cont'd) Power over Beneficial Interests in the Trust [IRC §674] Powers of Appointment Note Power to Amend the Trust De Facto Control Note Administrative Powers under Which Grantor Can or Does Benefit [IRC §675] Administrative Powers under Which Grantor Can or Does Benefit [IRC §675] (cont'd) Note Administrative Powers under Which Grantor Can or Does Benefit [IRC §675] (cont'd) Note Administrative Powers under Which Grantor Can or Does Benefit [IRC §675] (cont'd) Note Note Trustee, Beneficiary, or Other Person Has the Power to Take Principal or Income for Him/Herself [IRC §678] Note Trustee, Beneficiary, or Other Person Has the Power to Take Principal or Income for Him/Herself [IRC §678] (cont'd) Note Trustee, Beneficiary, or Other Person Has the Power to Take Principal or Income for Him/Herself [IRC §678] (cont'd) Note Trustee, Beneficiary, or Other Person Has the Power to Take Principal or Income for Him/Herself [IRC §678] (cont'd) Note Trustee, Beneficiary, or Other Person Has the Power to Take Principal or Income for Him/Herself [IRC §678] (cont'd) Trustee, Beneficiary, or Other Person Has the Power to Take Principal or Income for Him/Herself [IRC §678] (cont'd) Trustee, Beneficiary, or Other Person Has the Power to Take Principal or Income for Him/Herself [IRC §678] (cont'd) Note Study Question 26 Study Question 27 Study Question 28 Study Question 29 General Income Treated as That of the Grantor Note Transactions Between Grantor and Trust Ownership of Grantor Trust Note Intentionally Defective Grantor Trust (IDGT) Grantor Trust and Exclusion of Gain on Sale of Residence Grantor Trust and Exclusion of Gain on Sale of Residence (cont'd) General How IDGTs Work Study Question 30 Example Exam Question 1 Exam Question 2 Exam Question 3 Exam Question 4 Exam Question 5 Exam Question 6 Exam Question 7 Exam Question 8 Exam Question 9 Exam Question 10 Exam Question 11 Exam Question 12 Exam Question 13 Exam Question 14 Exam Question 15 Exam Question 16 Exam Question 17 Exam Question 18 Exam Question 19 Exam Question 20 Exam Question 21 Exam Question 22 Exam Question 23 Exam Question 24 Exam Question 25 Exam Question 26 Exam Question 27 Exam Question 28 Exam Question 29 Exam Question 30 Exam Question 31 Exam Question 32 Exam Question 33 Exam Question 34 Exam Question 35 Exam Question 36 Exam Question 37 Exam Question 38 Exam Question 39 Exam Question 40 Exam Question 41 Exam Question 42 Exam Question 43 Exam Question 44 Exam Question 45 Exam Question 46 Exam Question 47 Exam Question 48 Exam Question 49 Exam Question 50 FAQs References

Chapter 2. Income

This chapter provides an overview of income as it reports to estates and trusts. Among the topics covered include dividing income between a decedent's final individual tax return and an estate, and completion of the Form 1041.


Entire Course (including glossary) (1)

Generally, trusts account for income on a tax basis in the same manner as do individuals. That means, generally, the concepts you've learned to determine what is taxable income reported on an individual return will apply when determining what is taxable income on a trust or estate tax return. But trusts most often have a mix of income that is more heavily weighted toward investment income and less often have income from the active conduct of a trade or business.


Entire Course (including glossary) (2)
  • The quasi-pass-through nature of a trust that arises from the income distribution deduction means that, in many cases, the determination of taxable status and the nature of income at the trust level will actually impact the tax paid at the beneficiary level.

  • Unlike partnerships and S corporations, but like individuals and C corporations, all of the trust's income will be reported on page 1 of the Form 1041.


Form 1041 Page 1

In general, income is reported on lines 1 to 8 of Form 1041. Those lines for 2022 returns are scheduled to be as follows:

1. Interest income

2a. Total ordinary dividends

2b. Qualified dividends (divided between those allocated to beneficiaries and those allocated to the estate/trust)

3. Business income or loss (from Form 1040 or Form 1040-SR, Schedule C)

4. Capital gain or (loss) (from Form 1041, Schedule D)

5. Rents, royalties, partnerships, other estates and trusts, etc. (from Form 1040 or Form 1040-SR, Schedule E)

6. Farm income or (loss) (from Form 1040 or Form 1040-SR, Schedule F)

7. Ordinary gain or (loss) (from Form 4797, Sales of Business Property)

8. Other income


Comparison to Form 1040


Entire Course (including glossary) (3)

One item to note is that in some cases (lines 3, 5, and 6) individual income tax forms are used to report the applicable income items of the trust or estate. But that is not the case for interest and dividends where, unlike on the Form 1040, there is no special schedule that is used to report the item, but rather only totals are reported on the front page of the return.

Schedule D is specially modified for trusts and estates, thus requiring a Form 1041 version of the form for reporting capital gains and losses. As we will discuss, capital gains and losses have a special treatment for trusts and estates that has caused the IRS to require additional details relating to whether the items are allocable to income beneficiaries or the estate or trust.


General

Interest income of the trust or estate is reported on line 1 of Form 1041. The same rules that apply to an individual's interest and dividend income apply to trusts and estates. That includes the availability of the lower capital gains rate on qualified dividends received by the trust. The advantageous rates and the income level at which they are applicable will be described in detail later in the chapter.

Correction of Erroneous Form 1099 Reporting

A decedent's final Form 1040 should report interest and dividends earned up to the date of death. Any dividends and interest received after that date should be reported on the return of the estate or, if applicable, the trust. Note that since executors do not get an employer ID number the moment after someone passes (nor would we really expect them to do so), quite often there will be some portion of the interest or dividends that should be reported by the estate that will be found on the Form 1099 issued to the individual.


Correction of Erroneous Form 1099 Reporting (cont'd)

The instructions(1) to Form 1041 deal with this eventuality. The instructions indicate that all of the dividends and interest reported in the decedent's name on Forms 1099-INT and 1099-DIV should be listed on the final Form 1040 for the decedent. However, any interest or dividends earned after the date of the decedent's death will need to be reported on Form 1041.

(1) Note: at the time of this writing the IRS had not yet issued instructions for the 2022 Form 1041. References to “instructions” in these materials refer to the IRS instructions for the 2021 Form 1041, which are not expected to substantively vary from the 2021 form instructions.

Under the last entry for interest and/or dividends on Schedule B of the decedent's final tax return, subtotal the dividends or interest (as appropriate). On the following line the notation “Form 1041” and the name and address shown on the Form 1041 should be listed and the resulting portion to be reported on Form 1041 shown as a subtraction. The net value should be reported on the decedent's Form 1040 and the amount subtracted should be reported on Form 1041.

Note

The return preparer should take care to ensure that these erroneously reported amounts are properly categorized on the tax returns for the decedent and the estate. As well, the preparer should make sure that the items are properly disclosed in order to avoid IRS matching issues.

  • Complete the return, reporting income as it is reported to the decedent, the estate, or the trust.

  • Show amounts properly reported by other taxpayers as negative numbers on the reporting schedule.

  • Attach a worksheet reconciling amounts reported in each return.


Municipal Bond Interest


Entire Course (including glossary) (4)

As with an individual, municipal bond interest is not included in taxable gross income of an estate or trust. However, remember that the state with which the estate or trust files most likely will tax municipal bond interest that is paid from another state. Similarly, if distributions are made to beneficiaries, those distributions will retain the character of the municipal bond interest distributed. That may mean that interest which would not be taxable in the decedent's state will nevertheless be taxable at the state level to the beneficiary who may reside in another state. Thus, the preparer generally will need to disclose the composition, by state, of the municipal bond interest that is treated as flowing out to the beneficiaries of the trust or estate on their Schedules K-1.

Entire Course (including glossary) (5)

If municipal bond interest has been received by a trust or estate, expenses other than administrative expenses must be allocated between taxable and tax-exempt income under IRC §265 just as is true for an individual.


To view this interactivity please view chapter 2, page 8

Interactivity information:

Income Classification

Interest and Dividend income poses a very practical problem for us. The problem is that the due date for providing Form 1099-DIV and Form 1099-INT to the trust or estate is January 31. February 15th is the due date for brokerages issuing consolidated statements. Even then, quite often a number of amended Forms 1099 are generally issued after that date.

A complex trust must make its distribution decision by the sixty-fifth day following the end of the tax year, which is very shortly after the initial Form 1099 receipt. Advisors may need to work from year-end statements, understanding that there likely will be differences with what is reported on the Form 1099 that will ultimately be received from the broker. This strategy should allow the fiduciary to attempt to make an informed decision about the impact of making or not making a distribution during the 65-day period and making the election.

As with an individual return, a trust or estate reports the net income from certain business-like activities after reduction for allowed expenses. Thus, lines 3, 5, 6 and 7 may be negative numbers on the Form 1041.

Adjusted Gross Income

The general rules for allowed expenses follow the rules customary for other taxable entities with business or income producing operations. Generally, the trust or estate is allowed a deduction in computing the trust's adjusted gross income (which exists for the trust or estate even though not given its own line on Form 1041) for the expenses that meet one of two criteria.


Entire Course (including glossary) (6)

Allowable Trade or Business Expenses

First, a deduction in computing adjusted gross income (and thus reported on Schedule C or F) is allowed for ordinary and necessary expenses incurred in carrying on a trade or business and most other deductions related to the trade or business. [IRC §§ 62(a)(1) and 162(a)] Such other deductions would include, if related to a trade or business:

  • Taxes (such as property taxes),

  • Interest expense, and

  • Depreciation (subject to a special rule discussed later).

However, as with individuals, certain deductions are specifically not allowed to be used in computing above-the-line income even if related to a trade or business. Most of these, found in IRC §§211-224, are generally not applicable to a trust or estate (such as a deduction for a contribution to an IRA or health savings account).

In general, business deductions would be reported either on Schedule C (for most businesses) or Schedule F (for farms).


Expenses That Offset Income


Entire Course (including glossary) (7)

The second category of above-the-line deductions allowed in computing adjusted gross income that will end up as an offset to income exists to pull one set of deductions found in the IRC §§211 – 224 range and allow for other deductions above the line for activities that don't rise to the level of a trade or business. IRC §62(a)(4) allows a deduction in computing adjusted gross income for expenses for the production of income under IRC §212 that are attributable to property held for production of rents or royalties. Deductions generally allowed for trades and businesses (such as interest, property taxes, etc.) and any deduction for depletion is also allowed to be taken in computing adjusted gross income.

As with a trade or business, the deduction for depreciation will be subject to special rules discussed later.

The deductions allowed for rents and royalties will be reported on the page 1 of Schedule E. This is the same Schedule E that is used with Form 1040 or Form 1040-SR.


Proprietorship (Schedule C)

Income from a trade or business operated by a trust or estate is reported on a Form 1040 or Form 1040-SR, Schedule C, with the net income flowing to line 3 of the Form 1041.

There are some special issues that will impact an unincorporated business operated by a trust or estate. One key issue will be the impact of the passive activity rules. If the trust or estate cannot show material participation in the business, losses will be limited to the amount of passive activity income recognized by the trust or estate. Any amount of losses in excess of that amount would be carried forward to future years, subject the same rules as apply to the passive losses of individuals. [IRC §469]


Entire Course (including glossary) (8)

Proprietorship (Schedule C) (cont'd)


Entire Course (including glossary) (9)

If the business is profitable but there is no material participation, the income will also be treated as part of net investment income when computing the net investment income tax under IRC §1411. However, there is one piece of good news. A trust or estate is not subject to the self-employment tax as that tax is limited to individuals. [IRC §1401(a)]

Example 2.1

Walter Zeus owned and operated a movie theatre as a sole proprietorship. Walter dies on June 30, 2022, and his estate operates the theatre for the remainder of the year. The theatre shows a profit of $100,000 for the year, 50% of it from operations before Walter's passing and 50% from later operations. The $50,000 of income earned before Walter's passing will be reported on his Form 1040 and will be subject to self-employment tax under IRC §1401. However, the $50,000 of earnings for the last half of the year will not be subject to self-employment tax as the income is that of the estate, an entity not subject to tax under IRC §1401.

The other issue an advisor dealing with either an estate or a trust that is receiving assets from a decedent is that the basis of the assets will be reset to fair market value as of the date of death. Thus, the trust or estate will need to treat these assets as newly acquired assets and restart depreciation. However, these assets will not qualify for bonus depreciation because it is inherited property. In addition, the first year of the trust or estate is most often a short tax year, and depreciation must be adjusted under the first-year depreciation rules.

Effective for estate tax returns filed after July 31, 2015, the “consistent basis” rules of IRC §6035 may apply for assets received from the decedent's estate if the decedent was required to file an estate tax return. When those rules apply the trust should receive a Form 8971 Schedule A showing the basis reported on the Form 706 for any assets received by the trust from the estate. In that case the final value reported on the Form 706 will serve as the maximum basis the trust may generally report, even if the trust might have an argument that the fair value was really higher than that reported on the Form 706.

Inquiry should be made of the trustee of a trust that received property from a decedent to determine if a Form 8971 Schedule A has been or possibly will be issued. If one is received it should remain as part of the trust's permanent tax records so long as the trust continues to hold any assets listed on that form.


Farm (Schedule F)

Similar to income from a trade or business, income from operation of a farm is reported on the Form 1040 or Form 1040-SR reporting schedule (Schedule F) and then carried to its own line on the Form 1041 (line 6).

Note

However, a special rule applies to farm rentals that may surprise those who work with sharecropping arrangements on individual tax returns. The instructions to Form 1041 make clear that Form 4835, Farm Rental Income and Expenses, is not to be used to report arrangements where rental income and expenses are based on crops or livestock produced by a tenant. Such rentals are to be reported directly on Schedule E like any other rental and not on either Form 4835 or Schedule F.

The reason for this difference most likely arises from the fact that, as was true for Schedule C trade or business income, farm income is not subject to self-employment tax when earned by a trust or estate return. On an individual return, a Form 4835 isolates the sharecropping arrangement from a farm being operated by the taxpayer which allows the IRS to more easily identify arrangements where there is no self-employment tax liability. Since there is no such issue for a trust or estate, there is no real need for the Form 4835 to report such arrangements.


Qualified Business Income (QBI)

The Tax Cuts and Jobs Act of 2017 (TCJA) added IRC Section 199A, which will be effective for tax years 2018 – 2025. This provision provides for a maximum 20% deduction of qualified business income. Qualified business income for an estate or trust includes items of income, deduction, gain, and loss from any trades or businesses that are effectively connected with the conduct of a trade or business within the United States and included or allowed in determining taxable income for the year. The deduction for QBI is available to estates and trusts that have QBI allocated from businesses (such as those reported on Schedules C, F, and E) and other pass-through entities. We will discuss the deduction related to QBI in greater detail in the next chapter.


Entire Course (including glossary) (10)

Material Participation Issues

Material participation by a trust or estate has been an area of dispute between taxpayers and the IRS. The IRS had held internally that, in its view, a trust absolutely cannot function as a real estate professional under IRC §469(c)(7). [See Chief Counsel Email 201244017, 11/2/12] For all practical purposes, a trust will have an extraordinarily difficult time meeting the material participation tests for a trade or business to escape passive treatment for income or loss from the activity.

Technical Advice Memorandum 201317010

In an extreme example, the IRS had ruled that a trust with a trustee who was the CEO of an S corporation did not meet the material participation requirements with regard to the S corporation activity. While the IRS official position had been that a trust must show participation by the trustee (and no one else), that participation had been in the trustee's capacity as a trustee—and in the S corporation CEO's case, his participation was as an employee of the corporation.

Mattie K. Carter Trust v. U.S., 91 AFTR 2d 2003-1946 (2003)

Of course, internal memos are all fine and good—the ultimate test of a position takes place in court. For a long time we had very little guidance here, just a single District Court case, Mattie K. Carter Trust v. U.S. While that guidance was taxpayer favorable, rejecting the IRS trustee-only test, it was a single District Court case which often is a shaky foundation on which to rest a position.


Frank Aragona Trust v. Commissioner, 142 T.C. No. 9 (2014)

Now, we have a lot more—the U.S. Tax Court weighed in with a published opinion rejecting the IRS views entirely. The case is the Frank Aragona Trust v. Commissioner.

First, the Court explicitly rejected the IRS view that a trust, by the nature of being a trust, could not perform the personal services necessary to meet the real estate professional definition necessary to apply IRC §469(c)(7). The Tax Court noted:

If the trustees are individuals, and they work on a trade or business as part of their trustee duties, their work can be considered “work performed by an individual in connection with a trade or business.” Sec. 1.469-9(b)(4), Income Tax Regs. We conclude that a trust is capable of performing personal services and therefore can satisfy the section 469(c)(7) exception.


Entire Course (including glossary) (11)

Frank Aragona Trust v. Commissioner, 142 T.C. No. 9 (2014) (cont'd)

The Court went on to state:

Indeed, if Congress had wanted to exclude trusts from the section 469(c)(7) exception, it could have done so explicitly by limiting the exception to “any natural person.” In section 469(i), the Internal Revenue Code does exactly that. Section 469(i) grants a $25,000 allowance to “any natural person” who fulfills certain requirements. That Congress did not use the phrase “natural person” but instead used the word “taxpayer” in section 469(c)(7) suggests that Congress did not intend to exclude trusts from the section 469(c)(7) exception, despite what the IRS argues here.

The Court also gives short shrift to the IRS's broader view that material participation must be met by the trustees only and only if acting purely in their capacity as trustees. The Court comments:

On the basis of these legal principles, the IRS would have us ignore the activities of the trust's non-trustee employees. Additionally, the IRS would have us ignore the activities of the three trustees who are employees of Holiday Enterprises, LLC. It reasons that the activities of these three trustees should be considered the activities of employees and not fiduciaries because: (1) the trustees performed their activities as employees of Holiday Enterprises, LLC, and (2) it is impossible to disaggregate the activities they performed as employees of Holiday Enterprises, LLC, and the activities they performed as trustees.

Note that the latter argument is exactly the one described in the TAM earlier with regard to the S corporation CEO.


Frank Aragona Trust v. Commissioner, 142 T.C. No. 9 (2014) (cont'd)


Entire Course (including glossary) (12)

The Court concludes that it is not proper to ignore the employee-trustees. Governing law requires them to act in the interests of the beneficiaries and, in this case, their employer was an entity 100% owned by the trust. Thus, the Court effectively found it absurd to ignore their activities when counting material participation.

While a victory for taxpayer, it is important to note the facts recited in the above paragraph—this was a case of a trust with trustees who were employees of an entity wholly owned by the trust. In most cases, things won't be quite that simple—and, for now, we are left to wonder about the impact if the trust was a minority shareholder, for instance.

This issue has become much more important since the net investment income tax under IRC §1411 has become applicable, especially given the low adjusted gross income threshold at which trusts become subject to the IRC §1411 tax. Hopefully, this result will spur the IRS to act on comments given in the preamble to the final IRC §1411 regulations stating that they planned to address active participation rules for trusts in guidance under IRC §469. However, to date such guidance has not appeared.

To view this interactivity please view chapter 2, page 21

Interactivity information:

Material Participation Tests

A trust will need to meet the same material participation tests as an individual, generally measured by considering the activities of the trustees of the trust. To be deemed materially participating, the trust will need to show it meets one of the same seven tests that must be met by individuals.

Those seven tests are:

  • The trust participates in the activity for more than 500 hours during such year;

  • The trust's participation in the activity for the taxable year constitutes substantially all of the participation in such activity of all individuals (including individuals who are not owners of interests in the activity) for such year;

  • The trust participates in the activity for more than 100 hours during the taxable year, and such individual's participation in the activity for the taxable year is not less than the participation in the activity of any other individual (including individuals who are not owners of interests in the activity) for such year;

  • The activity is a significant participation activity for the taxable year, and the individual's aggregate participation in all significant participation activities during such year exceeds 500 hours. A significant participation activity is one in which the trust has at least 100 hours of participation and which the trust does not otherwise qualify to treat as a material participation activity;

  • The trust materially participated in the activity (determined without regard to this test) for any five taxable years (whether or not consecutive) during the ten taxable years that immediately precede the taxable year;

  • The activity is a personal service activity, and the trust materially participated in the activity for any three taxable years (whether or not consecutive) preceding the taxable year; or

  • Based on all of the facts and circ*mstances, the trust participates in the activity on a regular, continuous, and substantial basis during such year. [Reg. §1.469-5T(a)]

As with an individual, a trust claiming to meet these tests has the burden of documenting the trust's qualifications to meet one of these tests.

Note

As a practical matter, the Frank Aragona Trust decision will benefit primarily trusts where the principal asset of the trust is an operating business in which the trustees are actively involved in managing the trust operations. The case will not solve the problem for a trustee who hires managers to run the business, but is not otherwise involved directly in the operation of the business, except to perform general oversight of the activity as a trust investment.

Depreciation, Depletion and Amortization

Depreciation, depletion, and amortization are subject to a specialized treatment in the income taxation of trusts and estates. This specialized treatment arises due to the special rules related to a trustee or executor's handling of depreciation under a state's uniform principal and income act.

Generally, an estate or trust may claim a deduction for any of these items only to the extent that the item is not apportioned to the beneficiaries of the trust or estate. The apportionment rule differs, depending on whether the entity in question is a trust or an estate.


Entire Course (including glossary) (13)

Estates

Depreciation, amortization, and depletion for an estate is allocated between the estate and the beneficiaries based on the amount of estate income allocated to each. [Reg. §§1.167(h)-1(c), 1.611-1(c)(5)]

Example 2.2

The Estate of Harry Jones held a rental property on which there was deductible depreciation of $20,000 for the year. The estate had accounting income of $40,000 for the year, of which $10,000 was distributed to Mary Jones, Harry's sole heir. In this case the $5,000 ($20,000 × ($10,000/$40,000)) of the depreciation would be allocated to the beneficiary, Mary, and the estate would be able to claim a deduction for $15,000 on Schedule E.


Entire Course (including glossary) (14)

Estates (cont'd)

One interesting side effect of this rule is that depreciation may be allocated to the beneficiaries of an estate in excess of the income allocated to the beneficiary. [Revenue Ruling 74-530, 1974-2 CB 188]

There is an interesting difference in wording between the regulation specifying the allocation of depreciation [Reg. §1.167(h)-(1)(c)] and that dealing with the allocation of depletion. [Reg. §1.611-1(c)(5)]. The regulation outlining the allocation of depletion specifically requires the allocation to be made based on income allocated between the estate and beneficiary for the property generating the depletion deduction. However, the regulation on allocation of depreciation makes no such reference to the property generating the deduction.

The Internal Revenue Code provisions providing for the division make no such distinction, referring only to estate income in each instance. But the regulations serve to interpret these provisions, and their use of different language can create a problem.

The matter would be an issue, for instance, if the estate allocated the income from a particular property only to certain heirs who were going to receive the property. Taken literally, a depreciation deduction would be allocated in part to heirs with no rights to the underlying property, but a depletion deduction would be allocated only to those receiving the income from the property.


Trusts

For trusts, the law looks first to the underlying trust document or governing law to determine how depreciation is to be allocated, generally, whether the governing instrument or local law permits or requires the trustee to maintain a reserve for depreciation. If the trustee maintains such a reserve in the trust accounting, the amount of tax depreciation up to the reserve is allocated to the trust. Only after the reserve is allocated to the trust is the balance divided between the trust and the beneficiaries based on the trust's accounting income in excess of the amount set aside for the reserve. [Reg. §1.167(h)-1(b)]

Example 2.3

Two years later, the Harry Jones Heirs Trust now holds the rental property described in the example above for an estate. Under the terms of the trust document, the trust must maintain a reserve of $4,000 [$16,000 × ($10,000/$40,000] = $4,000) out of trust income related to the rental property described above for depreciation. The trust has generated $20,000 of tax depreciation. The trust had total accounting income, before reduction for the reserve, of $44,000. The trust distributed $10,000 of income to its single income beneficiary this year, Mary.

The first $4,000 of tax depreciation is to be allocated to the trust. A portion of the remaining $16,000 of depreciation is allocated to Mary as follows:

The $40,000 represents the trust's accounting income, before reduction for the reserve ($44,000) reduced by the amount of the reserve for depreciation ($4,000), claimed by the trustee in computing trust accounting income.

The trust receives the remaining $12,000 plus the original $4,000 that was part of its reserve, and claims a deduction for $16,000 on Schedule E.


Flow-Through Entities

Officially, as the instructions to Form 1041 note, depreciation flowing through to a trust or estate from a pass-through entity is treated in the same manner by the trust or estate as if the asset generating the depreciation had been held by the trust or estate itself. [Instructions to Form 1041, 2021, page 22]

However, quite often the trust or estate will not receive information from the entity regarding the amount of depreciation allocated to the trust that is included on the Schedule K-1. If such information is provided, the trustee would be able to pass through the depreciation to the beneficiary, assuming the general rules are met.


Entire Course (including glossary) (15)

IRC Section 179

One item deserves special note before moving beyond depreciation for trusts and estates, even though the item is technically not an item of depreciation.

Trusts and Estates

Trusts and estates are not eligible to claim a deduction under IRC §179. [IRC §179(d)(4)] The prohibition is an absolute one, and is not changed by the fact the trust may be actively conducting a trade or business in which it meets the material participation rules under IRC §469, as the law simply tells us that IRC §179 does not apply to trusts or estates. The fact that the trust might argue that since all income went to a beneficiary for the year, that the IRC §179 should pass through to the beneficiary, that logic also does not apply—the trust or estate simply cannot make use of the provision.

Grantor Trusts

Because a grantor trust is ignored as an entity for income tax purposes, it generally is assumed an individual who has assets otherwise eligible for IRC §179 held in a grantor trust (such as most revocable living trusts) is eligible to claim the IRC §179 deduction on his/her personal return. While no authority exists that directly confirms this, the IRS has not challenged that treatment in the many years since IRC §179 (and specifically IRC §179(d)(4)) has been part of the law.

Note

No similar prohibition exists on the use of the bonus depreciation rules under IRC §168(k).

Example 2.4

The Jolly Roger Trust owns and operates a retail sporting goods store. The trust purchased a new asset on the first day of the 2022 tax year with a MACRS life of 5 years at a cost of $20,000. The trust purchased no other assets during the year. The trust had income of $50,000 before considering any cost recovery on the new piece of equipment.

The trust is not able to claim any IRC §179 expense related to the equipment purchase. However, the trust may claim the 100% bonus depreciation ($20,000). As was noted above, this depreciation will be subject to allocation between the beneficiary and the estate.


Entire Course (including glossary) (16)

Entire Course (including glossary) (17)

A special rule applies if a trust or estate is a partner in a partnership or shareholder in an S corporation that wishes to make an IRC §179 election. The partnership or S corporation's basis in the property on which an IRC §179 election is made is not reduced by the trust or estate's allocable share of the IRC §179 amount. Rather, the partnership or S corporation may claim a depreciation deduction on this portion of the asset. [Reg. §1.179-1(f)(3)]

Note

Note that the depreciation occurs at the entity level and not at the level of the trust or estate. In an S corporation, the income allocation rules would generally seem to require that the deduction generated by this depreciation would be allocated to all of the corporation's shareholders, not just the trust or estate. Similarly, the partnership agreement would determine the allocation of the resulting depreciation in a partnership setting, and it is not required to be allocated only to the trust or estate. In that case, the trustee or executor may wish to consider requesting that the partnership agreement specifically allocate such depreciation to the trust or estate if it does not already do so.


Study Question 5

Which of the following most accurately describes the Tax Court's holding in the case of Frank Aragona Trust v. Commissioner?

AA trust cannot qualify as a real estate professional under §469(c)
BThe passive activity loss limitations do not apply to trusts or estates
CThe Tax Court agreed with the analysis found in TAM 201317010 that hours of a trustee performing services as an employee for an activity held by the trust do not count in determining active participation
DThe fact that governing law requires trustees to act in the interests of beneficiaries means all actions of the trustee is taken “as a fiduciary” for purposes of the §469 rules

Trusts and estates report gains or losses and are subject to the same rules as individuals generally. However, it is important to remember that, by default, gains or losses are treated as part of accounting principal, rather than income.

Capital Gains or Losses (Form 1041, Schedule D)

As with dividends and interest, the same rules that apply to an individual's capital gains apply to trusts and estates. The estate or trust reports capital gains and losses on Schedule D, Form 1041 which will flow onto that form either directly from a Form 1099-B on which the basis of securities has been properly reported, or from Forms 8949, Sales and Other Dispositions of Capital Assets, on which the details of sales are reported.

Details do not need to be reported on Form 8949 if both the sales proceeds and basis have been properly reported by the brokerage firm on Form 1099-B and there are no adjustments. Schedule D also picks up items treated as capital gains from Forms 4684, Casualties and Thefts; 6252, Installment Sale Income; 6781, Gains and Losses From Section 1256 Contracts and Straddles; 8824, Like-Kind Exchanges, 4797, Sale of Business Property, as well as capital gains and losses flowing from partnerships and S corporations.

Form 1041, Schedule D also provides a line for the reporting of capital gain distributions.

Note

The instructions to Form 1041 note that a Form 1040 Schedule D is not acceptable as part of a fiduciary return—rather the Form 1041 version must be used.


Basis of Property Acquired from Decedent

Income tax basis of property acquired from the decedent may be determined by reference to the law in effect on the date of death and forms filed by the estate. In most cases that basis will be the fair market value on the date of the decedent's death, but there are a number of special cases where that may not be the case.

Alternate Valuation

For instance, the estate may have used the alternate valuation date and, if so, the value of the asset on that date would be used as the basis of the asset.

Modified Carryover Basis for 2010

Also, advisors must remember the potential for a modified carryover basis if the decedent passed away in 2010. In such a case, an inquiry needs to be made about whether the estate elected to use the no estate tax provisions in place for that year, or whether the estate was taxed under the regular rules and used the traditional fair market value basis rules.

Note

Note that in order to use the alternate valuation date, there must be a reduction in the combined estate and GST tax due by making use of the alternate valuation date, it must reduce the gross estate, and it must be elected on a Form 706, United States Estate (and Generation-Skipping Transfer) Tax Return. [IRC §2032(a) and (c)] Thus, the value at that date cannot be elected if it would serve to increase the value of the estate, something an executor might like to have been able to do for an estate that would have no estate tax liability in either event.


Consistent Basis Rules

As was discussed earlier, Congress enacted a provision aimed at insuring that an heir, with an asset which was reported on a Form 706 with an estate tax liability, will not claim a higher basis for that asset on an income tax return. [IRC §§6015 and 1014] The basis is reported to the party receiving property from the estate on Form 8971 Schedule A.

If the trust or estate has an asset subject to these rules it cannot report a greater basis for any income tax purpose. If the trust or estate does so, the trust or estate will be subject to a 20% accuracy related penalty under §6662 for the taxes related to the “excess” basis reported on the income tax return. [IRC §6662(b)(8)]

Issues Involving Brokerage Firms

One thing to be aware of is that quite often brokerage firms do not properly reflect adjustments to basis of assets that passed through an estate, so some inquiry should be made before relying upon a report from the brokerage regarding computed gains or losses on the sale of various securities. Similarly, such organizations often don't grasp the rules involved if the decedent lived in a community property state at the time of his or her demise.


Entire Course (including glossary) (18)

Holding Period


Entire Course (including glossary) (19)

The holding period of all assets received from decedent is long-term, even if the decedent acquired the assets the day before he or she died and the estate is selling it a day later. [IRC §1223(9)] Normally, your tax software will flag this asset as being an inherited asset. Again, this is an area where at times certain brokerage firm reports will fail to pick up this issue and will treat these sales as short-term.

Example 2.5

The day before her death, Mildred Penny, bought stock in XYZ Corporation for $200,000 which is the value on the date she dies. Two days after her death the company announces a breakthrough in its research which the stock market rewards with an increase in stock price. Ms. Penny's executor sells the stock for $400,000. The estate will report a $200,000 long-term capital gain, even though the shares were only held for four days in total between Ms. Penny and her estate.


Decedent's Personal Residence

A personal residence generally becomes an investment asset to the estate or beneficiaries and will retain that status unless converted to personal use by a beneficiary. [Estate of Pauline Miller, TC Memo 1967-4, and H.V. Watkins, TC Memo 1973-167] As such, the sale of the property, if it occurs at a loss, may generate deductible capital loss.

A 1998 Office of Chief Council Service Center Advice (SCA 1998-012) caused a bit of controversy by asserting that a trust or estate could claim a loss only if it had actively converted the property to an income producing item, such as by renting the property out, if the property had been used for personal purposes by the decedent.

However, the IRS in Publication 559, Survivors, Executors, and Administrators, provided the following description of the proper treatment of the sale of the decedent's residence:

Sale of decedent's residence If the estate is the legal owner of a decedent's residence and the personal representative sells it in the course of administration, the tax treatment of gain or loss depends on how the estate holds or uses the former residence. For example, if, as the personal representative, you intend to realize the value of the house through sale, the residence is a capital asset held for investment and gain or loss is capital gain or loss (which may be deductible). This is the case even though it was the decedent's personal residence and even if you did not rent it out. If, however, the house is not held for business or investment use (for example, if you intend to permit a beneficiary to live in the residence rent-free and then distribute it to the beneficiary to live in), and you later decide to sell the residence without first converting it to business or investment use, any gain is capital gain, but a loss is not deductible.


Decedent's Personal Residence (cont'd)


Entire Course (including glossary) (20)

This guidance appears to be more in line with the court cases that suggests so long as there is no disqualifying use, any loss on disposal would be deductible. While some will complain that an IRS publication is not binding (which it is not), neither is a Service Center Advice binding on either the taxpayers or, importantly, the courts.

Example 2.6

Jean Bach had lived in her home on the beach for 30 years prior to her death. Upon her death, her executor placed the property on the market to be sold. Due to a decline in the market shortly after Jean's death, the property is sold two months later for $20,000 less than its value on her date of death. The estate should recognize a $20,000 long-term capital loss. This loss will not be subject to disallowance due to personal use since the estate is not treated as inheriting the decedent's use of the property.


Other Assets

The nature of an asset other than a residence also may be different for the trust or estate than it was for a decedent. As with the residence, the issue is how this taxpayer (the trust or estate) deals with the asset, not how the decedent dealt with it. As a result, an asset that may have been inventory in the hands of the decedent may be a capital asset when sold by the trust or estate.

Note

In the case of Maley v. Commissioner, 17 TC 260, acq., the Tax Court found that income received from the sale of wine by a cooperative, where the wine was received after the grower's death and where the grower generally sent his crop to the cooperative for processing into wine, was a capital asset for sales occurring after his death. This determination can be complicated because the IRS may attempt to argue some or all of a gain represents income in respect of a decedent. The Courts have been willing to agree with this to some extent. [See Commissioner v. Linde , 45 AFTR 1522 (CA9 1954).]

Tax Rates

Capital gain rates for estates and trusts are the same as for individuals. Thus, for 2022 the maximum rate imposed on long-term capital gains for the trust or estate is set at 20%. For 2022, the 20% maximum rate applies to long-term capital gains for trusts and estates with total taxable income greater than $13,700. The 15% rate applies to long-term capital gains for trusts and estates with total taxable income of at least $2,800 and less than $13,270. For trusts and estates with total taxable income of less than $2,800, the long-term capital gains rate is 0%.


Principal and Income Acts

Under Principal and Income Acts enacted in the various states, capital gains are allocated to trust principal and reportable by the fiduciary unless allocated to income beneficiaries:

  • By will or trust document,

  • By state law, or

  • By actual distribution of proceeds from the capital gains transaction.

Note

As will be discussed in Chapter 5 on the income distribution deduction and distributable net income, this fact means that advisors will most often find that the trust or estate will pay income taxes on any amounts of realized capital gains.

Net capital losses are only reportable by the estate or trust during the period of administration, and are limited to offsetting no more than $3,000 of ordinary income. There is no option to pass those losses through to the beneficiaries during any year except the last, even if the trust document provides that capital gains are considered part of trust income. Unused capital losses are allocated to beneficiaries in the final return of an estate or trust.


Income from rentals, royalties, partnerships, S corporations, and other trusts/estates are reported on Schedule E. The Schedule E used is actually the same as the Form 1040 or Form 1040-SR, Schedule E.

Rental Income

General

Rent and royalty income is reported using the first page of Schedule E, Form 1040 or Form 1040-SR:

  • The advisor should be sure to identify income in respect of decedent. That would include rents that were due but not yet paid on the date the decedent died.

  • An estate is eligible for the $25,000 rental loss exception to passive activity loss limitation rules, while a trust is not.


Real Estate Professional

The passive activity rules, discussed above, apply to trusts and estates. Thus, most trusts and estates, unlike the trust in case of Frank Aragona Trust v. Commissioner, will find that all rental activities will be treated as passive. The Aragona Trust case involved trustees whose activities allowed the trust to be treated as a real estate professional.

For a trust to be treated as a real estate professional, the trust must meet the requirements to be a real estate professional, which will require more than 750 hours of material participation by the trustee in real estate activities and more material participation in such real estate activities than in any other activity in which the trust materially participates. Real estate activities for this purpose include real property trades or businesses involved in:

  • Development,

  • Redevelopment,

  • Construction,

  • Reconstruction,

  • Acquisition,

  • Conversion,

  • Rental,

  • Operation,

  • Management,

  • Leasing, or

  • Brokerage.


Real Estate Professional (cont'd)


Entire Course (including glossary) (21)

As with individuals, if the trust qualifies as a real estate professional, that merely removes the automatic treatment of rental activities as passive. The trust would still have to demonstrate material participation in each rental activity. As with an individual, by default each rental is treated as a separate activity unless the trust elects to treat all rentals as a single activity. [IRC §469(c)(7)]

Entire Course (including glossary) (22)

While it is now possible for a trust to be treated as a real estate professional and have its rental properties not treated as automatically passive, the vast majority of trusts holding rental real estate will not be able to take advantage of this change in the law. The trustees generally will not be able to show sufficient hours of participation in the rental real estate activities for the trust to be treated as a real estate professional.



Pass-through Income

Trusts and estates may hold interests in various pass-through entities. As with an individual, the information contained on the Schedule K-1 received by the trust or estate will flow to various locations on the tax return.

Nonseparately computed income is reported on Schedule E, and after being combined with other items reported on Schedule E for the trust or estate, carried to line 5 of the Form 1041.

Other items, such as interest, dividends, investment interest expense, etc. are to be carried to the appropriate lines either directly on Form 1041 or on the appropriate supporting schedule.

A trust or estate reports income from Schedules K-1 for entities whose tax year ends during the tax year of the trust or estate.

Example 2.7

The Estate of Wilma Wayne owns a 10% interest in the American Drilling Partnership. The estate has a June 30 fiscal year-end, while the partnership reports on a calendar year. For the estate's tax year beginning July 1, 2021, and ending June 30, 2022, the estate reports the amounts reported to it on American Drilling's K-1 for the year ended December 31, 2021. The partnership's activity from January 1, 2022, through June 30, 2022 (the end of the estate's fiscal year) will be reported on the partnership K-1 for the year ended December 31, 2022, and on the estate's tax return for the year ended June 30, 2023.

As is detailed in Chapter 4, trust accounting rules generally define “income” from a partnership or S corporation very differently than the tax rules. Thus, a trust or estate holding an interest in a pass-through entity will often find a significant difference between taxable income flowing to the trust or estate from the pass-through entity and trust accounting income arising from the same entity.


Partnerships

Partnership nonseparately stated income is reported on Schedule E of Form 1041. Other items reported by the partnership will be picked up on the appropriate line of the return.


Entire Course (including glossary) (23)

Basis in Partnership Interest

As with individuals, a trust or estate must track its basis in the partnership in order to ensure that the trust or estate does not deduct losses in excess of its basis in the partnership and also to determine whether any distributions of cash (including deemed distributions of cash from a reduction in debt allocated to the trust or estate by the partnership) would give rise to taxable income. [IRC §§705, 722, and 731]

Note

Often the partnership interest will be held from the inception of the trust or estate, being received either from the grantor or the decedent. If the interest is acquired by transfer from the grantor, the beginning basis of the interest will generally be the grantor's interest in the property. In that case, the trustee will need to obtain information on the grantor's basis from the grantor upon the inception of the trust.

If the property is received from the decedent, then the beginning basis will be the basis of the property as determined under the rules for basis of inherited property pursuant to IRC §1014. Normally, the basis of inherited property will be either the fair market value at the date of death or, if applicable, the alternate valuation date. If the property is inherited, the executor or trustee should inquire whether an IRC §754 election has been or will be made. If such an election is in place, the trustee will need to look for information on the K-1 related to any adjustments that will be made to the income flowing out on the K-1.

Prior to the new rules issued by the IRS, the individual partner (in this case the trust or estate) was responsible for maintaining a calculation of tax basis. Notice 2019-66 provides that the partnership is required to report each partner's basis on the tax basis for taxable years beginning on or after January 1, 2020. The partnership will take responsibility for tracking the basis of each partner and reporting on Schedule K-1 Part L; however, the ultimate responsibility for assuring that the basis calculation is correct will fall on the trustee or executor.


Losses


Entire Course (including glossary) (24)

Similarly, after determining that there is sufficient basis to deduct a flow-through loss, the partnership or estate must test to ensure losses will be allowed under the at-risk rules. [IRC §465] The trust computes this limitation on Form 6198, At-Risk Limitations, if it has any amounts not at risk in the activity.

Finally, if a loss is allowed after both the basis limitations and the at-risk limitations, it is next tested under the passive activity limitations described earlier. [IRC §469] The trust or estate completes Form 8582, Passive Activity Loss Limitations.

If any loss has survived all three levels of tests, then the loss may flow forward onto the Form 1041 or the appropriate schedule.

Unlike S corporations, there is no limitation on the types of trusts or estates that may hold a partnership interest.


S Corporations

S corporation income shown on a Schedule K-1 is reported similarly to the partnership income reporting described above. Thus, the trustee must again be concerned with basis, at-risk, and the passive activity rules, but that is merely the beginning of the issues with S corporations and fiduciary income tax returns.

S corporations pose special problems in the context of a trust or estate since only qualified shareholders may own S stock. If the trust does not qualify to hold S stock, the corporation's S election is terminated and the entity reverts to C corporation status. In some cases, action must be taken in a timely manner by the trustee or beneficiaries in order to qualify the trust to hold S corporation stock.


Entire Course (including glossary) (25)

Estates Holding S Corporation Stock

Generally, estates do pose problems for an S corporation, presuming the stock is acquired from a decedent who was an eligible S shareholder at the date of death. [IRC §1361(b)(1)(B)]

Note

If the IRS determines that the administration of the estate has been unreasonably prolonged, there is a risk, should the assets pass to a trust following the termination of the estate, that the IRS would find the shares had actually been held for tax purposes by a disqualified trust if the shares were there more than two years after the time the IRS finds the estate administration should have terminated. Clearly, keeping an estate around merely because it is an eligible S shareholder while the successor in interest is not generally would not be a valid reason to prolong administration of an estate.

A trust that is qualified to make and does make an IRC §645 election (generally made on Form 8855) to be taxed as part of the decedent's estate would be an eligible shareholder during the period from the date of decedent's death until the earlier of: the date the estate and trust have distributed all of their assets or the day before the applicable date (2 years after the date of decedent's death, unless a Form 706 is filed when the date will be tied to determination of the estate tax). [Reg. §1.645-1(f)]

The estate of an individual in bankruptcy is also considered to be a valid S corporation shareholder. [IRC §1361(c)(3)]


Grantor Trusts as S Shareholders


Entire Course (including glossary) (26)

A trust that is treated under the grantor trust rules IRC §§671-6780 as 100% owned by an individual who is a citizen or resident of the United States is a qualified S corporation shareholder. [IRC §1361(c)(2)(A)(i)] Specifically, this means that a revocable living trust generally will qualify to hold S shares as long as the grantor of the trust meets the requirements to be an S shareholder. [PLR 200027032] This type of trust is the one that advisors most often will encounter where an eligible grantor trust will hold S shares.

Example 2.8

Hazel Wilson forms the Hazel Wilson Revocable Living Trust and transfers shares of ABC Corporation, an S corporation, into the trust. The trust provides that Hazel has the right to withdraw any and all trust assets at any time during her life and to revise or terminate the trust. Under the grantor trust rules, Hazel is treated as the owner of all assets in the trust due to the powers she retained. [IRC §676] The transfer of the shares into the Hazel Wilson Revocable Living Trust does not terminate ABC Corporation's S election.


Grantor Trusts as S Shareholders (cont'd)

The treatment applies to any grantor trust that meets the 100% test, not just revocable living trusts.

Example 2.9

Ben Bishop establishes the Bishop Grandchildren's Irrevocable Trust and places 200 shares of ABC Corporation stock into the trust. Ben has no power to revoke the trust and all assets in the trust will pass to Ben's grandchildren when they attain age 30. Under the terms of the trust, Ben has the power to remove any asset from the trust by substituting another asset of equivalent value. The retained power causes the trust to be treated as a grantor trust with Ben as the owner of the assets of the trust for income tax purposes. [IRC §675(4)] This type of trust is generally referred to as an intentionally defective grantor trust (IDGT) as the trust is recognized for estate and gift tax purposes (thus the assets will not be in Ben's taxable estate) even though Ben is treated as the owner for income tax purposes. This trust would qualify to hold the S corporation stock without terminating the S election, at least so long as Ben is alive.

Conversely, it is important to note that the trust must be treated as 100% owned by the individual under the grantor trust rules. If the trust is only partially treated as a grantor trust under the rules, it would not qualify as an eligible S shareholder under these provisions.


Testamentary Trusts as S Shareholders for Two-Year Period


Entire Course (including glossary) (27)

If a trust receives S corporation shares under the terms of a will, the trust will be considered an eligible S corporation shareholder for a limited period of time. The trust is treated as an eligible shareholder for two years from the date the shares are transferred to the trust by the terms of a will. [IRC §1361(c)(2)(A)(iii)]

Example 2.10

David Rogers's will provides that the shares he holds in the Old Towne S Corporation will pass to the David Rogers Children's Trust. David dies on June 30, 2020, and the shares are transferred to the trust on September 30, 2020, by the estate's executor. The trust will be treated as an eligible shareholder through September 30, 2022. After that date, unless another exception applies, the trust will not be an eligible shareholder. If the trust continues to hold the shares at that point, the S election would terminate.

A similar rule applies if the shares were held by a grantor trust under the rule described earlier after the death of the grantor. In that case, the 2-year period runs from the date of the deemed owner's death if the trust continues in existence following the death of the grantor. [IRC §1361(c)(2)(A)(iii)]

Example 2.11

Hazel Wilson, whose shares had been held in the Hazel Wilson Revocable Living Trust, dies on March 1, 2020. At that point, the trust ceases to be treated as a grantor trust under the grantor trust rules. Because the trust had previously qualified as a grantor trust, the trust is treated as an eligible shareholder until March 1, 2022.

Note

Advisors, especially those who are also involved in the preparation of the S corporation tax return, need to carefully note such deadlines to ensure that S status is not accidentally lost. The rules noted above provide for no special rule that could apply if the shares remain in the trust due to circ*mstances beyond the trustee's control, such as the existence of a legal dispute among the heirs that blocks the trustee from transferring the shares or making certain elections. In such situations, the corporation may be forced, once matters are resolved, to file a private letter ruling request with the IRS for relief from an inadvertent termination.


Voting Trusts as S Shareholders

A qualified voting trust is an eligible S corporation shareholder. Such a trust is one created primarily to exercise the voting power of the stock that is transferred to it. To be a qualified voting trust, all of the following must be true:

  • All beneficial owners of shares must be treated as the owner of the shares under the grantor trust rules.

  • The trust must be created pursuant to a written trust agreement entered into by the shareholders that:

    • delegates to one or more trustees the right to vote;

    • requires all distributions with respect to the stock of the corporation held by the trust to be paid to, or on behalf of, the beneficial owners of that stock;

    • requires title and possession of that stock to be delivered to those beneficial owners upon termination of the trust; and

    • terminates, under its terms or by state law, on or before a specific date or event. [Reg. §1.1361-1(h)(1)(v)]


Voting Trusts as S Shareholders (cont'd)


Entire Course (including glossary) (28)

Each beneficial owner is treated as a shareholder of the S corporation with respect to their proportionate share of the stock held by the trust. [Reg. §1.1361-1(h)(3)(i)(E)]

Example 2.12

Mary, Fred, and Jane each hold 100 shares of ABC Corporation stock, an S corporation. They transfer their shares to a trust. They agree to the transfer provided that James, as trustee of the trust, will exercise the voting power of the ABC Corporation stock. The trust terms provide that the trust will terminate in 10 years, at which time the shares will be returned to Mary, Fred, and Jane. During the term of the trust, all distributions are to be passed directly through to Mary, Fred, and Jane and if the trust is terminated prior to the 10-year period, the stock is to revert to Mary, Fred, and Jane. In this case, the trust is an eligible S corporation shareholder for its entire term.

To view this interactivity please view chapter 2, page 54

Interactivity information:

Qualified Subchapter S Trusts (QSSTs)

There are two general purpose trusts that can hold S corporation stock.

The first one we will consider is a qualified subchapter S trust, or QSST. To be a qualified subchapter S trust, the trust must satisfy two separate tests. First, the trust must have terms that comply with the requirements for terms of a QSST trust. [IRC §1361(d)(3)]

Second, the beneficiary (not the trustee) of the trust must elect to have the QSST provisions apply to the trust in a timely manner. [IRC §1361(d)(2)]

To meet the terms of a QSST, the trust must provide the following:

  • During the life of the current income beneficiary, there shall be only one income beneficiary of the trust,

  • Any corpus distributed during the life of the current income beneficiary may be distributed only to such beneficiary,

  • The income interest of the current income beneficiary in the trust shall terminate on the earlier of such beneficiary's death or the termination of the trust,

  • Upon the termination of the trust during the life of the current income beneficiary, the trust shall distribute all of its assets to such beneficiary, and

  • All of the income is distributed currently to a single individual who is a citizen or resident of the United States. [IRC §1361(d)(3)]

QSST Election


Entire Course (including glossary) (29)

Once it is determined that the trust is eligible to be treated as a QSST, the beneficiary must make an election to consent to having the S corporation interest in the trust taxed under the QSST provisions. [IRC §1361(d)(2)(A)] That election must be made within 2 months and 15 days after the stock is received by the trust (or is deemed received by the trust if the trust was previously an eligible testamentary trust). [IRC §1361(d)(2)(D)]

Note

If a trust fails to make a timely QSST election, the IRS has provided procedures for automatically receiving permission to make a late election in Revenue Procedure 2013-30. This revenue procedure provides relief if the taxpayer satisfies the general requirements of Section 4 and the specific requirements applicable to that taxpayer under Sections 5 through 7 of this revenue procedure. Procedures for obtaining relief for late QSST election are found in Section 6.


QSST Election (cont'd)


Entire Course (including glossary) (30)

Once made the election is irrevocable without the consent of the IRS. [IRC §1361(d)(2)(C)] If there is a successor income beneficiary, the election will apply to that successor beneficiary and all future beneficiaries—thus, only the initial income beneficiary is the one who is able to decide whether the QSST treatment will apply to the trust. [IRC §1361(d)(2)(B)(ii)]

Example 2.13

Wanda is a beneficiary of the NewStock Trust that has all of the terms required for the trust to be treated as a QSST. Wanda makes a timely election for the trust to be treated as a QSST. Under the terms of the trust, at Wanda's death the interest passes to her son, Renaldo. Renaldo does not need to make, nor is he allowed to revoke, the QSST treatment of the trust's ownership of the S corporation stock.

If the trust owns shares in more than one S corporation, the beneficiary must make a separate election for each S corporation's shares to be treated under the QSST rules if the beneficiary wishes the rules to apply to all of the holdings. [IRC §1361(d)(2)(B)(i)]


Election Requirements

If a corporation is making its S election and has a trust shareholder where the beneficiary is eligible to make a QSST election, the QSST election can be made in Part III of the same Form 2553 of the corporation making the S election.

Otherwise, the election is made by the beneficiary of the trust signing and filing an election with the IRS Service Center where the S corporation files its income tax return. That election must:

  • Contain the name, address, and taxpayer identification number of the current income beneficiary, the trust, and the corporation;

  • Identify the election as an election made under IRC §1361(d)(2);

  • Specify the date on which the election is to become effective (not earlier than 15 days and two months before the date on which the election is filed);

  • Specify the date (or dates) on which the stock of the corporation was transferred to the trust; and

(continued on next page)


Election Requirements (cont'd)

  • Provide all information and representations necessary to show that:

    • Under the terms of the trust and applicable local law—

      • During the life of the current income beneficiary, there will be only one income beneficiary of the trust (if husband and wife are beneficiaries, that they will file joint returns and that both are U.S. residents or citizens);

      • Any corpus distributed during the life of the current income beneficiary may be distributed only to that beneficiary;

      • The current beneficiary's income interest in the trust will terminate on the earlier of the beneficiary's death or upon termination of the trust; and

      • Upon the termination of the trust during the life of such income beneficiary, the trust will distribute all its assets to such beneficiary.

    • The trust is required to distribute all of its income currently, or that the trustee will distribute all of its income currently if not so required by the terms of the trust.

    • No distribution of income or corpus by the trust will be in satisfaction of the grantor's legal obligation to support or maintain the income beneficiary. [Reg. §1.1361-1(j)(6)(ii)]


Election Requirements (cont'd)


Entire Course (including glossary) (31)

Once the election takes effect, the beneficiary will treated as the owner of the shares for income tax purposes under the grantor trust rules of IRC §678(a) and as the shareholder of the shares for S corporation purposes. [IRC §1361(d)(1)]

Note

The Qualified Subchapter S Trust (QSST) passes all of the income of S corporation through to the beneficiary for tax purposes and all distributions from the S corporation must be paid out to the beneficiary. What the QSST does is control the disposition of the shares, as the beneficiary does not need to have any rights to dispose of the shares, including via the beneficiary's will. Thus, the QSST makes sense where there is concern regarding the disposition of the shares, including the shares potentially falling into the hands of a creditor. But the trust is not appropriate if the trust is meant to keep cash out of the hands of the beneficiary while allowing the S corporation to continue to make distributions that other shareholders receive or if the trustee is to have discretion regarding the beneficiaries to receive any distributions from the trust.


Electing Small Business Trusts (ESBTs)

The other general purpose trust election available to allow S corporation stock to be held by a trust without terminating the S election is the electing small business trust (ESBT) election under IRC §1361(e).

Unlike the QSST, the ESBT trust does not need to currently distribute any amounts received by the S corporation, the trustee makes the election without any action on the part of the beneficiary, the trust can have multiple beneficiaries, and can provide for a sprinkle power where the trustee can exercise discretion in determining the beneficiaries who are to receive any distributions.

As with the QSST, an ESBT both has to meet specific requirements based on its terms [IRC §1361(e)(1)] and an election must be made in a timely manner (though by the trustee and not the beneficiary). [IRC §1361(e)(3)]


Entire Course (including glossary) (32)

Electing Small Business Trusts (ESBTs) (cont'd)

To be able to make the election the trust must meet all of the following requirements:

  • The trust:

    • Does not have as a beneficiary any person other than:

      • An individual,

      • An estate,

      • Certain tax-exempt organizations described in paragraph (2), (3), (4), or (5) of IRC §170(c), or

      • An organization described in IRC §170(c)(1) (A State, a possession of the United States, or any political subdivision of any of the foregoing, or the United States or the District of Columbia) which holds a contingent interest in such trust and is not a potential current beneficiary.

    • Has no interest that was acquired by purchase (any acquisition of property where basis is determined under IRC §1202 is considered a purchase for these purposes). Regulation §1.1361-1(m)(1)(iii) notes that a purchased interest would include an interest received by a beneficiary where the person receiving the beneficial interest pays the gift tax.


Electing Small Business Trusts (ESBTs) (cont'd)

  • The trust is not:

    • A qualifying subchapter S trust (a trust where the beneficiary has made a valid QSST election that remains in force),

    • A trust that is exempt from tax, or

    • A charitable remainder trust (either a charitable remainder annuity trust or charitable remainder unitrust). [IRC §1361(e)(1)]

The election is to be made by a trustee with the ability to legally bind the trust who must sign the election statement. The election statement must contain the following items:

  • The name, address, and taxpayer identification number of the trust, the potential current beneficiaries, and the S corporations in which the trust currently holds stock;

  • An identification of the election as an ESBT election made under IRC §1361(e)(3);

  • The first date on which the trust owned stock in each S corporation;

  • The date on which the election is to become effective (not earlier than 15 days and two months before the date on which the election is filed); and

  • Representations signed by the trustee stating that—

    • The trust meets the definitional requirements of IRC §1361(e)(1); and

    • All potential current beneficiaries of the trust meet the shareholder requirements of section 1361(b)(1). [Reg. §1.1361-1(m)(2)(ii)]


Electing Small Business Trusts (ESBTs) (cont'd)

If the trust owns shares in multiple S corporations that all file their returns with the same IRS Service Center, only a single election needs to be filed with that Center. If the trust owns shares in S corporations that file with multiple Service Centers, the initial election must be filed with each Center. However, if the trust later acquires shares in an S Corporation that files with a Service Center that no election was filed with, the trust is not required to send a new election to that Service Center. [Reg. §1.1361-1(m)(2)(i)]

An ESBT trust takes on a dual personality, with the S corporation income and expenses being isolated from the other assets of the trust. However, the trust still has only a single employer identification number. [Reg. §1.1361-1(m)(3)(ii)]

Each potential current beneficiary is treated as a shareholder of the S corporation for purposes of determining the S corporation's eligibility to maintain its S status (such as for the maximum number of shareholders.) [IRC §1361(c)(2)(B)(v)] A potential current beneficiary for any period is any person who at any time during such period is entitled to, or at the discretion of any person may receive, a distribution from the principal or income of the trust (determined without regard to any power of appointment to the extent such power remains unexercised at the end of such period).

So far, the trust appears to be far superior to the QSST in virtually every respect, being a much more flexible vehicle. That is true, but there is one very significant downside to the ESBT when it comes time to compute taxes due on trust income.

IRC §641(c) provides the tax treatment for ESBTs. The portion of the ESBT which consists of stock in one or more S corporations is treated as if it were a separate trust. [IRC §641(c)(1)] The tax will be imposed at the highest applicable tax rate (that is, the trust does not get the benefit of the lower tax brackets) although it will be able to make use of lower capital gain tax rates (though again only at the highest rate). [§641(c)(2)(A)].


Electing Small Business Trusts (ESBTs) (cont'd)

The taxable income of the ESBT is computed as follows:


Entire Course (including glossary) (33)
Entire Course (including glossary) (34)

Items flowing from the S corporation's K-1s are taken into account.

Entire Course (including glossary) (35)

Gain or loss from the disposition of S corporation stock.

Entire Course (including glossary) (36)

State or local income taxes allocated to the prior two items.

Entire Course (including glossary) (37)

Interest expense paid or accrued on indebtedness incurred to acquire S corporation stock.


Electing Small Business Trusts (ESBTs) (cont'd)

No other deductions not allowed for the above will be included in the calculation of taxable income and none of the above amounts will be apportioned to any beneficiary of trust. As well, no deductions will be allowed for capital losses in excess of capital gains arising from the S corporations. [IRC §641(c)(2)]

The AMT exemption shall be set to zero when computing any minimum tax liability for this portion of the trust.

The S portion of an ESBT is the portion of the trust that consists of stock in one or more S corporations and is not treated as a grantor type trust. The Form 1041 instructions provide that the tax on the S portion must be figured separately from the tax on the remainder of the ESBT (if any) and attached to the return. It is entered on Schedule G, Part I, line 4. [2020 Form 1041 Instructions, page 14-15]

The S Corporation items included in computing the tax on the ESBT trust are to be excluded from determination of the tax for the remainder of the trust and in computing the distributable net income of the trust. [IRC §641(c)(3)]

Note

ESBT elections are generally made for nontax reasons, as it will be rare that the total federal income tax imposed on the S corporation income will be less under the ESBT taxation regime than it would be if the income was taxed at the beneficiary level. However, there may be compelling non-tax reasons why there is a need to hold shares in the trust, retain S status, allow for distributions to be made to other shareholders but not require such funds to flow out to trust beneficiaries. However any decision to make use of such a trust should be made with a full understanding of the likely significant tax cost of doing so.


Distributions of Passive Activities

Quite often a trust holding a partnership or S corporation will end up with losses from the partnership or S corporation being suspended under the passive activity loss rules of IRC §469. If the trust or estate distributes that interest to a beneficiary or heir, the basis of the property is increased by the amount of the passive activity loss allocable to the interest. However, the losses will not be deductible, either by the trust/estate or by the beneficiary/heir. [IRC §469(j)(12)]

The basis is increased because basis is first reduced before applying the passive activity loss rules while the asset is held by the trust. This is the same rule that applies to an individual. The application of this provision is illustrated by the following example, adapted from the passive activity loss regulations, but revised for a trust as the holder of the interest.


Entire Course (including glossary) (38)

Example 2.14

In 2021, Abby Trust, a calendar year trust, acquires a partnership interest in Racing Partnership, a calendar year partnership. Racing Partnership's only activity is a trade or business activity in which Abby Trust materially participates for 2021. Racing Partnership incurs a loss in 2021. Abby Trust's distributive share of Racing Partnership's 2021 loss is $1,000. However, Abby Trust's basis in the partnership interest at the end of 2021 (without regard to Abby Trust's distributive share of partnership loss) is $600; accordingly, IRC §704(d) disallows any deduction in 2021 for $400 of Abby Trust's distributive share of Racing Partnership's loss. The remainder of Abby Trust's distributive share of Racing Partnership's loss would be allowed as a deduction for 2021 if the taxable income for all taxable years were determined without regard to IRC §§469, 613A(d), and 1211.

Abby Trust does not materially participate in Racing Partnership's activity for 2022. In 2022, Racing Partnership again incurs a loss, and Abby Trust's distributive share of the loss is again $1,000. At the end of 2022, Abby Trust's basis in the partnership interest (without regard to Abby Trust's distributive share of partnership loss) is $2,000; accordingly, in 2022 IRC §704(d) does not limit Abby Trust's deduction for either Abby Trust's $1,000 distributive share of Racing Partnership's 2022 loss or the $400 loss carried over from 2021 under the second sentence of IRC §704(d). These losses would be allowed as a deduction for 2022 if taxable income for all taxable years were determined without regard to IRC §§469, 613A(d) and 1211.

(continued on the next page)

Example 2.14 (cont'd)

Under these facts, only $400 of Abby's distributive share of Racing Partnership's deductions from the activity are disallowed under IRC §704(d) in 2021. Abby Trust's remaining deductions from the activity are treated as deductions that arise in connection with the activity for 2021 under paragraph (d)(8) of this section. Because Abby Trust materially participates in the activity for 2020, the activity is not a passive activity (within the meaning of Reg. §1.469-1T(e)(1)) of Abby Trust for such year. Accordingly, the deductions that are not disallowed in 2020 are not passive activity deductions.

Abby Trust does not materially participate in Racing Partnership's activity for 2021. Accordingly, the activity is a passive activity of Abby for such year. No portion of Abby Trust's distributive share of Racing Partnership's deductions from the activity is disallowed under section 704(d). Accordingly, Abby Trust's distributive share of Racing Partnership's deductions for 2021 and the $400 of deductions carried over from 2021 are both treated under paragraph (d)(8) of this section as deductions that arise in 2022. Since the activity is a passive activity for 2022, such deductions are passive activity deductions.

Since the loss will never be deducted under the distribution rule noted previously, the law provides that the basis that was reduced by the suspended passive loss is to be restored before it is distributed to the beneficiary/heir.

Example 2.15

The Wayne Dexter Irrevocable trust distributes its interest in the XYZ Partnership to Karen Dexter, a trust beneficiary. At the time the interest was distributed, the trust had a basis in the interest of $10,000 and had suspended passive losses related to XYZ Partnership of $3,000. The basis of the partnership interest is increased to $13,000 ($10,000 + $3,000) immediately before the distribution to Karen. Karen's basis when she receives the interest is $13,000. However, the $3,000 loss will not be deductible even if Karen has passive income in future years or disposes of her interest in the partnership. Similarly, the trust does not claim that loss either.


Entire Course (including glossary) (39)

Study Question 6

Which of the following is not true regarding a Qualified Subchapter S Trust (QSST)?

ADuring the life of the current income beneficiary, there must be only one income beneficiary of the trust.
BThe trustee must elect for the trust to be classified as a QSST.
CAny corpus distributed during the life of the current income beneficiary must be distributed to that income beneficiary.
DIf the trust terminates during the life of the current income beneficiary, all assets of the trust must be distributed to that current income beneficiary.

Form 4797 is used to report ordinary gain or loss from the sale of property other than capital assets, using the same rules that apply to individuals. The recapture provisions of IRC §§1245, 1250, 1252, 1254, and 1255 apply to dispositions of business assets by estates and trusts.

Recapture reported to an estate or trust on Schedules K-1 is to be reported through Form 4797 into Form 1041.

Example 2.16

The Robert Allen Trust sells a rental real estate property for $200,000. The property has an unadjusted basis of $350,000 and there is accumulated depreciation of $75,000 through the date of sale. The sale results in a loss of $75,000 ($200,000 – ($350,000 – $75,000)). That loss is treated as an IRC §1231 loss. The sale is reported on line 2, Part I of the Form 4797. Assuming the trust has no other IRC §1231 transactions, the loss would be carried to line 7 of Form 4797, and after following instructions for all other lines on that form will then be carried to line 7 of Form 1041 and be reported as an ordinary loss.

Example 2.17

Six years later, the Robert Allen Trust sells a piece of equipment used in a business owned by the trust. The equipment, which had a five-year life under MACRS, had an original unadjusted basis of $10,000 and was fully depreciated. The equipment was sold for $12,000.

As the equipment is IRC §1245 property, the sale is reported on page 2, Part III of the Form 4797. The description is entered on line 19 along with the date acquired and date sold. The sales price is reported on line 20, with the accumulated depreciation on line 22. A gain of $12,000 is reported on line 24.

The depreciation allowed or allowable of $10,000 is reported on line 25a. As $10,000 is less than $12,000, $10,000 is reported on line 25b. The $10,000 represents the IRC §1245 recapture.

With no other items reported on Form 4797, the $12,000 gain would be carried to line 30, with the IRC §1245 recapture reported on line 31. The recapture is also carried to line 13 of Part II of Form 4797 (the Ordinary Gain or Loss Section).

The excess of the total gain over the recapture of $2,000 is entered on line 32. This gain represents an IRC §1231 gain. The number is carried to line 6 of Part I of Form 4797, where it is included in the calculation of net IRC §1231 gain. Assuming no other IRC §1231 gains or losses and no unrecaptured IRC §1231 losses, the net IRC §1231 gain of $2,000 would be carried to Schedule D as a long-term capital gain.


General

Form 1041 line 8 is where other items of income are reported on the trust tax return. Generally, items should be reported on this line only if there is no specific line provided for an entry. The return should contain a schedule indicating the items that are included in line 8 unless the description will fit on the line provided next to line 8. Two items of income included on line 8 are unpaid compensation received by the decedent's estate that is income in respect of decedent and any part of a total distribution shown on Form 1099-R, Distributions From Pensions, Annuities, Retirement or Profit-Sharing Plans, IRAs, Insurance Contracts, etc., that is treated as ordinary income.


Entire Course (including glossary) (40)

Compensation of the Decedent

The instructions for Form 1041 note that compensation of the decedent that represents income in respect of a decedent should be reported on line 8.

Any compensation for the services of a cash-basis employee received by the estate after the death of the decedent is treated as income in respect of a decedent (IRD). These items to be reported on line 8 include:


Entire Course (including glossary) (41)
  • Wages earned during an employee's last illness paid to the estate [Revenue Ruling 59-64]

  • Payment of accrued but unpaid leave held by the employee at his/her death paid to the estate [Revenue Ruling 55-229]

  • Deferred compensation earned by the employee prior to the date of death that is paid to the estate after the date of death of the employee [Reg. §1.691(a)-2(b), Example 1]


Annuities and Retirement Accounts

Line 8 also contains amounts received that are reported on a Form 1099-R as ordinary income from an annuity or retirement account.

Trusts and estates are subject to tax on annuities in the same manner as are individuals. Quite often these accounts comprise the largest single source of income in respect of a decedent making up an estate or a trust.

Income in respect of a decedent is taxable to the estate or another party succeeding to the income in the same manner as it would have been had it been received by the decedent prior to his/her death. [IRC §691, Reg. §1.691(a)-3(a)]


Entire Course (including glossary) (42)

Annuities and Retirement Accounts (cont'd)

Income in respect of a decedent consists of the following:

Entire Course (including glossary) (43)

All accrued income of a decedent who reported his or her income by use of the cash receipts and disbursem*nts method;

Entire Course (including glossary) (44)

Income accrued solely by reason of the decedent's death in case of a decedent who reports his or her income by use of an accrual method of accounting; and

Entire Course (including glossary) (45)

Income to which the decedent had a contingent claim at the time of his or her death. [Reg. §1.691(a)-1(b)]

The practical impact of these rules for income in respect of a decedent is that the provisions that generally apply to step up the basis of assets held by a decedent at death (IRC §§1014(a) and 1022) will not apply to increase basis in an annuity or retirement account that is transferred at death. [Proposed Reg. §1.691(a)-3] Thus, for purposes of computing the taxable portion of an annuity or retirement account, the decedent's basis will be treated as the estate or trust's basis in the account.


Chapter 3. Deductions

This chapter provides an overview of deductions available on Form 1041. Included in the discussion are interest, taxes, fiduciary fees, and charitable deductions.


Entire Course (including glossary) (46)

As is true for income, deductions are also generally based on the rules that apply to individuals. However, there are certain treatments and items that are unique to trusts and estates.


Entire Course (including glossary) (47)

Deductions are claimed on page 1 of Form 1041. Per the 2021 Draft Form 1041, deductions are reported on the following lines of the form:

10. Interest

11. Taxes

12. Fiduciary fees

13. Charitable deduction (from Schedule A on page 2 of Form 1041)

14. Attorney, accountant, and return preparer fees

15a. Other deductions

15b. Net operating loss deduction

18. Income distribution deduction

19. Estate tax deduction including certain generation-skipping transfer taxes

20. Qualified business income deduction. Attach Form 8995 or Form 8995-A

21. Exemption

In this chapter, we will look at most of these deductions. The income distribution deduction is a specialized topic that will be discussed in Chapter 5 since it involves a number of specialized trust topics.


General


Entire Course (including glossary) (48)

IRC §163 governs the deduction of interest for a trust or estate, just as it does for individuals.

Note

Generally, the same interest tracing rules found at Reg. §1.168-8T that apply for individual taxes will apply for an estate or trust to determine the classification of most interest. An advisor preparing a trust or estate income tax return where the trust or estate has incurred interest expense should approach the classification of interest expense in the same manner as the preparer has done for preparing individual income tax returns. Thus, all of the various optional rules for tracing the use of funds that apply on individual returns will apply for the trust or estate.

As with an individual, the default rule under IRC §163 for a trust or estate is that all interest expense represents nondeductible personal interest, unless it properly falls into one of the following categories:

  • Interest paid or accrued on indebtedness properly allocable to a trade or business (which will generally be reported on Schedule C),

  • Investment interest (reported on Form 4952, Investment Interest Expense Deduction, and deductible only to the extent of net investment income),

  • Interest which is taken into account under IRC §469 in computing income or loss from a passive activity (which generally would include rental properties),

  • Qualified residence interest,

  • Interest paid on estate taxes in the limited cases of taxes deferred under IRC §6163 (value of reversionary or remainder interest in property) and IRC §6166 (payment of estate tax where estate consists largely of interest in closely held business), and

  • Interest allowable as a deduction under IRC §221, relating to interest on educational loans. [IRC §163(h)(2)]

Note

Interest incurred on a loan taken out to pay estate taxes generally will not be deductible since it will fail to fall into one of the above categories.

Example 3.1

Ronald Tori operated an ice cream shop prior to his death. His estate continues to operate the ice cream shop. Ronald had a credit line that was used exclusively for the ice cream shop and the estate has continued to use that line. The estate paid $15,000 in interest on the credit line.

The $15,000 will be reported on Schedule C as a deduction against the income of the ice cream shop. The amount will not be reported on line 10 of Form 1041.


Entire Course (including glossary) (49)

Qualified Residence Interest

Generally, the rules that provide for the deduction of interest as qualified residence interest for an individual require the residence to be either the principal residence of the taxpayer or another residence of the taxpayer that is actually used as a residence by the taxpayer. [IRC §163(h)(4)(A), Reg. §1.163-10T(p)(3)(i)(B)]

Since a trust or estate cannot actually reside in a property, it's not surprising that the IRC provides for a modification of this rule for determining whether interest is qualified residence interest of the trust or estate.


Entire Course (including glossary) (50)

Qualified Residence of Beneficiary

In the case of a trust or estate, interest paid or incurred by an estate or trust on indebtedness secured by a qualified residence of a beneficiary of the trust or estate is treated as qualified residence interest if the residence would be a qualified residence if owned by the beneficiary. The beneficiary must have either a present or residuary interest in the trust or estate. [IRC §163(h)(4)(D)]

Example 3.2

Christina Zane's estate includes Christina's residence. Katy, Christina's daughter, is the only heir of Christina's estate and per the will is to receive all assets in the estate once the administration is complete. Katy intends to move into the residence and use it as her principal residence. Interest paid by the estate on the mortgage will be deductible on Form 1041.

Decedent's Residence

A property that was used as the decedent's residence prior to death, unless used by a beneficiary as a residence as described above, will not be a qualifying residence and the interest would not be deductible as qualified residence interest.

Note

If the decedent's former residence is rented out or otherwise used for income producing purposes, the trust or estate may be able to claim deductions under other provisions (interest on rental properties or investment interest). But merely holding the property prior to distributions to heirs may lead to the treatment of the interest on the mortgage as nondeductible personal interest.


Requirements

The same requirements for properties that qualify as a principal residence that apply for individuals will apply to the trust or estate. For tax years beginning after 2017 and before 2026, the Tax Cuts and Jobs Act of 2017 provides that home acquisition indebtedness is limited to $750,000 for home acquisition indebtedness incurred after December 15, 2017. The previous $1,000,000 limitation remains for acquisition indebtedness incurred on or before December 15, 2017. Interest on home equity indebtedness is not allowed after 2017 and before 2026, without regard to the date the home equity loan was incurred. Note however that home equity indebtedness that qualifies as acquisition indebtedness because the funds were used to substantially improve the residence, is allowed subject to the $1,000,000 or $750,000 limits.

Example 3.3

Assume that Christina died during 2021 and the balance of the first mortgage on the property, which was used to acquire Christina's home was $300,000. There was a $200,000 interest only home equity credit line outstanding on Christina's home that she had used to pay personal expenses during her life.

The estate would be entitled to deduct the interest paid on the $300,000 first mortgage in full as acquisition debt, deductible both for regular taxes and in computing the alternative minimum tax. However, none of the interest on the credit line would be deductible for regular tax or computing the alternative minimum tax.


Investment Interest

The investment interest rules of IRC §163(d) apply to estates and trusts in the same manner as they apply to individuals. The amount deductible as investment interest expense by the trust or estate is limited each year to the net investment income of the trust or estate, with any excess carried forward to succeeding years.

Investment Income

Investment income includes the following items:

Entire Course (including glossary) (51)

Portfolio income under the passive activity rules (generally interest, dividends, etc.) reduced by qualified dividends, unless the estate or trust elects to forgo the lower tax rate on such income and include the amount in investment income pursuant to IRC §163(d)(4)(B).

Entire Course (including glossary) (52)

Short-term gains on the sale of property held for investment. However, net long-term capital gains on such dispositions will not be considered investment income unless the estate makes the election to forgo the lower capital gain tax rates pursuant to IRC §163(d)(4)(B).

Entire Course (including glossary) (53)

Income from a working interest in oil and gas held in an entity that does not limit the taxpayer's liability if the taxpayer does not materially participate in the activity. [IRC §163(d)(5)(A)(ii)]


Investment Interest (cont'd)

Note

When Congress expanded the lower capital gain rate to include qualified dividends, that income, like capital gains, was removed from investment income by default. Thus, trusts that had borrowed funds to invest in stocks found that they often had little or no investment income that could be used to enable the deduction of investment interest. In such cases, unless the trust or estate elected to waive the lower tax rates and include these items in investment income, much or all of the investment interest would be nondeductible. Thus, normally the election is made to waive the lower rate since by doing so the trust gets a deduction that will offset the amount now being excluded from the lower rate, effectively giving a zero tax rate.

If the trust has reason to believe that it will have significant investment income in the future that will not qualify for the lower rates, it may make sense to skip making the election and allow the investment interest to flow forward to the future year where it will offset income otherwise taxed at ordinary rates even without the election. However, in making that determination, the trust should not lose sight of the present value of a current tax reduction versus the discounted value of a reduction in the future.

An advisor should discuss the matter with the trustee to come to a decision regarding whether this election should or should not be made.


Deductions

Investment income is then reduced by deductions directly related to the production of investment income to compute net investment income. [IRC §163(d)(4)(C)] If such a deduction is limited due to the 2% of adjusted gross income limitation, it reduces investment income only to the extent it is actually allowed as a deduction.

The Tax Cuts and Jobs Act of 2017 provides that all miscellaneous itemized deductions subject to 2% of AGI are suspended for tax years beginning after 2017 and before 2026.

Note

Tax advisors need to understand how their tax software approaches the election to waive the lower tax rates and claim the amounts as investment income. Some software defaults to making the election to include the amount in investment income in order to maximize the investment interest deduction. Some other software does not automatically make the election, a specific entry must be made.

In either event, the tax advisor should review options whenever investment interest expense would otherwise be limited. The tax advisor should ensure the trustee/executor agrees with the election, if made.


Reporting

Investment interest is reported by the trust or estate on Form 4952, where the deductible portion is computed.

Example 3.4

In 2021, the ABC Trust incurs $10,000 of interest on a loan it took out to purchase a portfolio of securities. The trust has interest income of $4,000, and dividend income of $15,000 (of which $10,000 is qualified dividend income). The trust paid an investment advisory fee of $10,000 for management of its investment properties.

The trust initially computes its investment income as follows: $4,000 interest income + ($15,000 dividends – $10,000 qualified dividends) for total investment income of $9,000. The trust would be allowed an investment interest deduction of $9,000, with the $1,000 disallowed being carried forward to the following year.

However, the trust could elect to treat $1,000 of the qualified dividends as investment income. If the trust makes the election to do so (by simply including the amount on Form 4952 as part of investment income as provided in the instructions to the return), then the entire $10,000 of investment interest expense is allowed as a deduction. However, $1,000 of the qualified dividends will be subject to ordinary tax rates and not be eligible for the reduced tax rate on such income.


Study Question 7

Which of the following would not be considered in determining the deductibility of interest paid by a trust or estate on a mortgage used to acquire the decedent's residence?

AThe residence is being used as a qualified residence by a beneficiary with a present income interest in the estate.
BThe trust rents out the property to an unrelated party on a five year lease.
CThe residence was used as the principal residence of the decedent for five years prior to the date of death.
DThe residence is used as a qualified residence by an heir named in the will who will receive a distribution of his share of the residual estate.

General

As with individuals, some, but not all, taxes are allowed as deductions by IRC §164.


Entire Course (including glossary) (54)

Deductible Taxes

Except as otherwise provided in IRC §164, the following taxes are allowed as a deduction for the taxable year within which paid or accrued. IRC §164(a) includes:

  • State and local, and foreign real property taxes.

  • State and local personal property taxes.

  • State and local, and foreign income, war profits, and excess profits taxes.

  • The generation-skipping transfer tax imposed on income distributions.

Note that foreign income taxes are deductible only if the taxpayer has not elected to claim the foreign tax credit in lieu of the deduction for such taxes.

The Tax Cuts and Jobs Act of 2017 limits the total deduction for taxes to $10,000, effective for tax years beginning after 2017 and before 2026. The $10,000 limitation rule does not apply if the taxes are paid or accrued in carrying on a trade or business or incurred in an activity described in IRC Section 212 (production of income).

Foreign real property taxes shall not be taken into account under subsection (a) for these same years.


Nondeductible Taxes

Key nondeductible taxes for the trust include:

  • Federal income, duties, and excise taxes

  • Federal estate and gift taxes, as well as state estate, legacy, inheritance, succession or gift taxes

  • Generation-skipping transfer taxes not imposed on income distributions

  • Foreign real property taxes

Reporting

Taxes will generally be reported on line 11 of Form 1041 and will be limited to $10,000 effective for tax years after 2017 and before 2026. However, in the following cases the taxes should be reported elsewhere on the return and are not limited by the $10,000 limitation:

  • Taxes related to a trade or business of the trust will be reported on Schedule C.

  • Taxes related to a rental property will be reported on Schedule E for the appropriate rental.

  • Taxes related to a farm being operated as a business (as opposed to a rental) will be reported on Schedule F.


State and Local Income Taxes

The estate or trust is eligible to claim a deduction for state and local income taxes. [IRC §164(a)(1)]

A deduction is claimed by an estate for state and local income taxes on the decedent's income tax returns that are paid after the date of death as deductions in respect of a decedent. [IRC §691(b)] Such taxes may be concurrently claimed as a deduction on the decedent's Form 706 estate tax return.

State and Local Sales Taxes

A trust or estate, like an individual, has the right to elect to claim a deduction for state and local sales taxes in lieu of claiming a deduction for state and local income taxes. [IRC §164(b)(5)]

However, the trust or estate is not eligible to use the optional sales tax tables as they are limited by the IRS solely to individuals. Thus, a trust or estate would need to calculate the state and local sales paid by the trust.

As a practical matter, most trusts and estates are not likely to incur large amounts of sales taxes, so this election will primarily be useful for trusts and estates in states without a state income tax.


Property Taxes


Entire Course (including glossary) (55)

Property taxes paid by a trust or estate are deductible in computing the trust or estate's regular tax liability. [IRC §164(a)(1), (2)] The deduction is available for property taxes imposed on either personal or real property.

To be a deductible personal property tax, the tax must be an ad valorem tax imposed annually on personal property. An ad valorem tax is one based on the value of the property.

Example 3.5

The Estate of Hazel Chance paid $200 state motor vehicle tax on a car Hazel owned at her death that the estate still owns. The tax is imposed each year at a rate of 1% of the value of the vehicle ($17,500, resulting in a tax of $175), plus a registration fee of $25. Only $175 of the tax is deductible as a personal property tax, while the $25 registration fee is not deductible.


Property Taxes Related to Trade or Business


Entire Course (including glossary) (56)

If the property tax relates to a trade or business of the trust or estate, it is reported on Schedule C or Schedule F, respectively. If the tax relates to property held for rental, it is reported on Schedule E. These taxes would be deductible in computing both the regular income tax and the alternative minimum tax. However, while other property taxes are deductible in computing the regular income tax, they may not be deducted in computing alternative minimum taxable income.

Decedent's Accrued Property Taxes

Accrued property taxes that were a decedent's obligation at the time of his/her death represent a deduction in respect of a decedent. [IRC §691(b)] As such, the amount is deductible both on the Form 706 (estate tax return) of the decedent (as a liability due at death) and the Form 1041 (as a deductible tax). Taxes accruing after the date of death are deducted only on the Form 1041.


Study Question 8

Which of the following most accurately describes the deduction for taxes paid by a trust or estate?

ANo deduction may be claimed by the trust or estate for any generation-skipping transfer (GST) taxes paid by the trust or estate.
BSince the foreign tax credit is not available to a trust or estate, the trust or estate must claim a deduction for any foreign taxes paid.
CAn estate may claim an income tax deduction for state inheritance taxes paid on property received from the decedent.
DAn estate may claim a deduction for state income taxes due on the decedent's final income tax return both on the Form 706 and Form 1041.


Entire Course (including glossary) (57)

Generally, a trust may claim a deduction for fiduciary fees paid to the trust fiduciary as a cost of administering the trust. Fiduciary expenses include probate court fees and costs, fiduciary bond premiums, legal publication costs such as notices to creditors or heirs, the cost of certified copies of the decedent's death certificate, and other costs related to fiduciary accounts. Fiduciary fees are allowable under IRC Section 67(e) if they are incurred for the administration of the trust or estate, if they would not have been incurred if the property was not held in an estate or trust. These fees are deducted on line 12 of Form 1041.

Fiduciary fees paid are also referred to as executor fees when paid in the administration of an estate. Those fees deducted on Form 706 cannot also be deducted on Form 1041. The tax advisor must elect to deduct such administrative expenses on Form 1041 in lieu of claiming a deduction for such fees on the Form 706. A discussion of this election is found later in this section.


Fiduciary Fees (cont'd)

For small trusts and estates, the fee is often paid to a person who is not a professional fiduciary, but rather a relative or friend of the trustor/decedent. Such fees are taxable to the fiduciary, but generally they will not be subject to self-employment tax. That is because the self-employment tax applies only to a trade or business carried on by an individual. A non-professional fiduciary is generally not found to be carrying on such a trade or business, unless all of the following conditions are met:

  • There is a trade or business among the assets of the estate,

  • The fiduciary actively participates in the operation of this trade or business, and

  • The fees of the fiduciary are related to the operation of the trade or business. [Revenue Ruling 58-5]


Entire Course (including glossary) (58)

General

Charitable deductions of trusts and estates are subject to restrictions and special rules that are unique to the Form 1041. For this reason, such deductions are claimed on Schedule A of Form 1041, with the deductible balance carried forward to line 13 of Form 1041.

One key difference is that, unlike individuals (who face a 50% of adjusted gross income limitation (increases to 60% after 2017 and before 2026) [IRC §170(b)(1)]) and C corporations (which face a 10% of taxable income limitation [IRC §170(b)(2)]), there is no percentage of income limitation on charitable contributions of an estate or trust, unless the estate or trust has unrelated business income under IRC §512. IRC §642(c)(1), while imposing other conditions that are discussed later, has no reference to a similar percentage of income limitation and the IRC §170(b) limitation provisions make no references to trusts or estates.


Entire Course (including glossary) (59)

Authorization to Make Charitable Contribution

Unlike individuals, it is not enough for the trust to make a contribution to a qualified charity for which no benefit is received. Rather, the tax law looks for specific authorization for the trust or estate to make such a contribution, as well as imposing requirements for the source of the deduction.

  • A charitable contribution will potentially be allowed to a trust or estate if and only if the contribution is made pursuant to the terms of the governing instrument of the estate or trust. [IRC §642(c)]

  • In Old Colony Trust v. Commissioner, 19 AFTR 489 (S.Ct. 1937), the United States Supreme Court ruled that the provision can be one that authorizes the trustee to make such contributions at his/her discretion. That is, the trust or estate document does not have to mandate that a charitable contribution be made in order to qualify under the IRC §642(c) terms of the instrument rule.

  • The IRS allowed that payments made to a charity to settle a will contest were also treated as made pursuant to the governing instrument. [Revenue Ruling 55-122]


Entire Course (including glossary) (60)

Contributions Made From Amounts Included in Gross Income

Another limitation that must be met for charitable contributions to be deducted by a trust is that they must be made out of amounts included in gross income. [IRC §642(c)] The fact that the gross income was recognized in an earlier year does not bar the deduction, so long as no deduction had previously been allowed for such contribution. [Reg. §1.642(c)-1(a)(1)]

Example 3.6

The Mary Jones Trust provides that, at the discretion of the trustee, a particular piece of real estate may be distributed to a charity. The trustee distributes that real estate to the charity at a time when the property was worth $700,000. The basis of the property in the hands of the trust immediately before the distribution was $300,000. Had the property been sold by the trust, the gain would have been taxable as a long-term capital gain.

While an individual would be allowed a deduction of $700,000 (subject to adjusted gross income limitations) for the contribution of the property, the trust is allowed no deduction as the contribution is not being made out of the gross taxable income of the trust.

Assume the trust allows the trustee to sell the property and distribute the proceeds to the charity. If the trust sold the property and then made a contribution of the proceeds of $700,000, it would recognize a long-term capital gain of $400,000, but be allowed a charitable contribution deduction of $400,000.


Gross Income Defined

Note that the test above relates to the tax concept of gross income, rather than the fiduciary accounting income concept of gross income. As a result, a contribution out of trust principal could give rise to a charitable contribution deduction, but only to the extent the principal from which the gift was made was included gross taxable income.

Note

For instance, under the provisions of the uniform principal and income act, discussed elsewhere, distributions from an IRA to a trust would generally consist largely or entirely of principal. However, if those funds are then contributed to charity after being paid to the trust, the payment would still qualify as coming out of gross income for tax deduction purposes under IRC §642(c), even though the payment may be deemed to come entirely out of trust accounting principal.

Tax-Exempt Income

No deduction will be allowed for contributions made from tax-exempt income, since the amounts would not be included in gross income of the trust or estate. [Reg. §1.642(c)-3(b)]


Election to Deduct in Year Prior to Contribution

A trust or estate can elect to accelerate a deduction made out of gross income in the year prior to when the contribution is made. [Reg. §1.642(c)-1(b)] The election is available so long as the gross income on which the contribution is to be made has been recognized by the end of the year prior to actual payment of the contribution. If the election is made to claim the deduction in the preceding year, no deduction is allowed for the year the contribution is paid.

Election Deadline

The election must be made by date prescribed for filing the tax return (including extensions) for the succeeding taxable year. If the trustee has already made the election on the prior year return, the election can be revoked without the consent of the IRS if the revocation is made before the same due date for the return for the succeeding year. However, after that date, the election becomes irrevocable. [Reg. §1.642(c)-1(b)(2)]

Note

What this means is that the trustee/executor will have an absolute right to claim a deduction in the most favorable year, but the final decision must be made on or before the date for filing the return for the year in which the contribution is made. Tax advisors who see a contribution being made out of gross income where the income had been recognized in an earlier year should consider advising the trustee/executor of the existence of this option.


Election Procedure


Entire Course (including glossary) (61)

The election is made by filing a statement with the original or amended return that:

  • States the name and address of the fiduciary,

  • Identifies the estate or trust for which the fiduciary is acting,

  • Indicates that the fiduciary is making an election under IRC §642(c)(1) in respect of contributions treated as paid during such taxable year,

  • Gives the name and address of each organization to which any such contribution is paid, and

  • States the amount of each contribution and date of actual payment or, if applicable, the total amount of contributions paid to each organization during the succeeding taxable year that is to be treated as paid in the preceding taxable year. [Reg. §1.642(c)-1(b)(3)]

Example 3.7


Entire Course (including glossary) (62)

The Johan Bass Trust (a calendar year trust) provides that the trustee shall make contributions to the XYZ Charity (a §501(c)(3) organization) from the interest received on a particular corporate bond. The bond pays interest of $10,000 that is received by the trust on December 31 of Year 1. In January of Year 2, the trustee sends XYZ Charity a check for the $10,000.

The trustee of the Johan Bass Trust would generally claim a deduction for the contribution in Year 2. However, if the trustee elects, the deduction can instead be claimed in Year 1. The trustee has until the due date (including extensions) for filing the Year 2 return to make the final decision on which year the deduction will be claimed.

Since the trustee may revoke the election, the trust could claim the deduction on the Year 1 return initially. If it is determined when preparing the Year 2 return that it would be preferable not to have elected, the trustee may prepare an amended return for Year 1 revoking the election and then claim the deduction on the Year 2 return.


Estate Permanent Set Aside Provision

A second acceleration of deduction provision applies to an estate if certain conditions are met. The rule will also apply for a trust that has made the election to be treated as part of the decedent's estate under IRC §645.

Note

The same rule described below applies to certain trusts established on or before October 9, 1969 that provided for qualified irrevocable remainder interests to be left to a charity. [IRC §642(c)(2)(A), (B)] Obviously, the number of such qualifying trust is small and continually decreasing, so as a practical matter, most advisors will see this permanent set aside rule only in the context of an estate. Thus, our discussion of this issue will refer to it only in the context of an estate.

An estate is allowed a deduction for amounts that are permanently set aside for charitable purposes. [IRC §642(c)(2)] It is important that the amounts be permanently set aside, which generally will require that the estate transfer the amount out of estate income and into principal specifically restricted to distribution to the charity or charities in question. However, unlike the election to deduct in the year prior to the contribution, in the case of a permanent set aside by an estate, there is no deadline by which the amounts must be transferred to the charity or charities.

Example 3.8

The Estate of Mildred Pennywise left all assets to five charities. In the current tax year, the estate received interest of $80,000. The estate set aside the $80,000 on its books for distribution to the charities. The estate will be allowed a deduction in the current year for amounts permanently set aside for charity.

The deduction will not be allowed unless, based on the facts and circ*mstances, the possibility that the amounts will not be distributed to the charity is so remote as to be negligible. [Reg. §1.642(c)-2(d)]

Note

The case of the Estate of Belmont, 144 TC No. 6 (2015), is a case where the possibility was found to be not so remote as to be negligible. The estate consisted of a residence, a condominium in which the decedent's brother had been allowed to reside, and an IRA. The estate provided that $50,000 was to be left to the brother and all other assets of the estate would go to a charity. The estate claimed a charitable deduction after the IRA proceeds were received, claiming the amount had been permanently set aside. However, the brother claimed to have been promised a life estate and filed suit against the estate. Legal expenses related to dispute depleted the estate's assets, eating into the funds that had been claimed to be set aside.

The Tax Court found that the legal dispute was reasonably foreseeable as of the end of the tax year in which the estate claimed the deduction. The estate was aware that the brother's claim of a life estate was a serious one, and it was reasonably possible that significant legal expenses would need to be incurred to resolve the matter.


Special Final Year of Trust or Estate Rule

A charitable contribution deduction cannot create or increase the excess deductions on termination available to be passed out to beneficiaries under IRC §642(h), O'Bryan v. Commissioner, 75 TC 304 (1980).

Reporting Distributions to Charitable Beneficiaries

A distribution to a charitable beneficiary is reported on Schedule A and no Schedule K-1 is issued to the charity. The payment to the charity does not qualify for a distribution deduction under IRC §661, but rather is treated solely under the charitable contribution rules of IRC §642(c). With no distribution deduction related to the charity, there is no item that would be reported on Schedule K-1 to the charity. [Revenue Ruling 68-667, 1968-2 CB 289]


Entire Course (including glossary) (63)

Study Question 9

Which of the following is true regarding charitable contributions from trusts and estates?

ATrusts and estates are subject to a limitation of 60% of adjusted gross income for charitable contributions.
BA charitable contribution from a trust must be authorized pursuant to the terms of the governing documents.
CA trust or estate must record the deduction in the year that the charitable contribution is made.
DAn estate permanently sets aside funds for charitable purposes. The estate will be able to take a deduction if, based on the facts and circ*mstances, the distributions will more likely than not be distributed to charity.

General

The net operating loss deduction of IRC §172 is available to trusts and estates under the provisions that apply to individuals. That means a net operating loss is available generally if the trust or estate has losses from a trade or business it operates or it incurs a casualty loss.

Carryovers

A trust or estate is subject to the same carryback or carryforward time periods as individual income taxpayers, including the various special carryback periods. Prior to the Tax Cuts and Jobs Act of 2017, for most losses that meant the loss may be carried back two years and then carried forward 20 years. Like an individual, any loss must have been carried back, unless the trust or estate timely elected to waive the carryback period and carry the loss forward.

The Tax Cuts and Jobs Act of 2017 (TCJA) provides that for tax years after December 31, 2017, a net operating loss cannot be carried back, but must be carried forward. The carry forward is not limited to 20 years (as it was under previous legislation) and is instead allowed to be carried forward indefinitely. Note, however, that farming losses arising in tax years beginning in 2021 or later may be carried back 2 years and carried forward indefinitely. The TCJA also provides that net operating loss carryforwards are limited in that they cannot offset more than 80% of taxable income. The Coronavirus Air Relief Act (CARES Act) temporarily modified these rules, allowing a 100% deduction for NOLs arising in taxable years 2018, 2019, and 2020.


Carryovers (cont'd)

Thus, under current law taxpayers may claim a deduction limited to 80% of the NOL for the current years and any NOLs carried forward from years after 2017. A 100% deduction is permitted with regard to NOLs arising in tax years 2017 and earlier. [IRC §172]

The TCJA also introduced an “excel business loss limitation” in Code section 461(l) that applies to noncorporate taxpayers, including trusts and estates. Under that provision, a business loss exceeding $270,000 for trusts and estates cannot be claimed for the tax year 2022 and must instead be carried over to subsequent years as an NOL. The limitation amount is indexed annually. The CARES Act temporarily suspended this limitation, but it is again applicable for tax years beginning on or after January 1, 2021.

[IRC §461(l)]

Note

In practice, most trusts or estates do not hold operating business interests or incur casualty losses, making trust or estate net operating losses a relatively rare event in practice. Even if a trust or estate has an interest in a trade or business, the loss still must clear the passive activity rules under IRC §469 before it could lead to a net operating loss.


NOL Reduces Distributable Net Income

One unique feature of net operating loss carrybacks or carryforwards as they apply to trusts and estates is the potential impact on beneficiaries. The net operating loss reduces the amount of the trust's distributable net income (DNI). Thus, the Schedules K-1 for beneficiaries are to be recomputed for carryback years for each beneficiary. Affected beneficiaries must file their return within three years of due date of the taxable year that generated the net operating loss. [Revenue Ruling 61-20, 1961-1 CB 248]

Example 3.9

In 2017, the James Henderson Irrevocable Trust generates a net operating loss that is carried back two years. The loss reduces the trust's distributable net income to zero in that year. During that year, the trust had made a distribution from income to Harry Henderson and Harry received a Schedule K-1 reporting his portion of the income distribution deduction. Following the carryback, as DNI is reduced to zero, the income distribution deduction is also reduced to zero.

The trust should prepare a revised Schedule K-1 for Harry showing no income and marked as an amended Schedule K-1. The revised Schedule K-1 is to be submitted with the amended trust return carrying back the net operating loss and provided to Harry for use in preparing his own amended tax return for the year in question.


Procedure

As with individuals, the trust or estate recomputes all items that are limited based on the trust or estate's adjusted gross income in computing tax for the year to which the loss is carried.

  1. The net operating loss carried to a tax year is reduced by modified taxable income to determine the amount of net operating loss (if any) that is available to be carried to the next available year.

  2. Modified taxable income is the taxable income of the trust or estate subject to the following adjustments:

    • The net operating loss for the tax year or any later year is added back to taxable income; [IRC §172(b)(2)]

    • Any net capital loss deduction is added back to taxable income; [IRC §§172(b)(2) and (d)(2)]

    • The charitable contribution deduction is removed from the computation of taxable income; [Reg. §1.642(d)-1]

    • The allowed income distribution deduction is added back to taxable income; [Reg. §1.642(d)-1] and

    • The deduction for the personal exemption is added back to taxable income.

(continued on next page)


Procedure (cont'd)

  1. Modified taxable income must be adjusted to take into account the deductions that would be allowed for miscellaneous itemized deductions, casualty losses, and percentage depletion after making the adjustments above that would impact the trust's adjusted gross income. [Reg. §1.172-5(a)]

  2. If the net operating loss being carried to the year exceeds the modified taxable income computed above, the excess is carried to the next available year.

  3. The estate or trust can file the net operating loss refund claim by either filing Form 1045, Application for Tentative Refund, or preparing an amended Form 1041 for the year in question.

  4. The statute of limitations for the year the loss is carried to is three years from the date prescribed for filing the tax return (including extensions) for the year generating the net operating loss. [IRC §§6511(d)(2) and (4)]

Note

An estate does not succeed to the decedent's net operating loss. Rather, any net operating loss remaining unused that would have carried forward from the decedent's final Form 1040 is lost. [Revenue Ruling 74-175, 1974-1 CB 52]


NOL – Excess Deductions on Termination of an Estate or Trust

Upon termination of a trust or estate, a beneficiary succeeding to its property is allowed to deduct any unused NOL carryover for regular and alternative tax NOL for AMT purposes if the carryover would be allowable to the estate or trust in a later tax year but will not be used because it is the final year of the estate or trust. The unused carryover amounts of NOL and ATNOL are reported to the beneficiary on Schedule K-1 box 11, using codes D and E in the final year.


Entire Course (including glossary) (64)

General

As was discussed earlier, the rules related to disallowance of expenses related to tax-exempt income under IRC §265 apply to trusts and estates in the same manner as they apply to individuals. However, given that generally a much larger percentage of a trust's income arises from investment assets and much of the trust's expenses incurred are related to the trustee dealing with that income, the issue tends to take on a greater importance in a trust or estate.


Entire Course (including glossary) (65)

Expenses Directly Allocable to Income

The first step in handling such expenses is to determine if an expense is directly allocable to a specific type of income. Expenses directly allocable to taxable income, unless limited by another provision of the tax law, will be allowed in their entirety. Similarly, expenses that are directly allocable to exempt income are not allowed as a deduction. [Reg. §1.265-1(c)]

Note that in the following examples and discussions, that although the miscellaneous itemized deductions subject to the 2% of AGI limitation are suspended for tax years after 2017 and before 2026, the allocation methods and examples are important to understand for the years when the deduction is available.

Example 3.10

In 2017, the Mary Jones Irrevocable Trust pays $1,000 in investment advisory fees to Investment Firm A. That firm manages an account that invests solely in taxable bonds and performs no other services for the trust. The trust also pays $1,500 in investment advisory fees to Investment Firm B which manages a portfolio consisting of municipal bonds and also performs no other services for the trust.

The $1,000 in fees paid to Firm A will not be limited by the rules of IRC §265 in any way. However, the entire $1,500 of fees paid to Firm B will be wholly non-deductible on the trust's federal income tax return.


Expenses Not Directly Allocable to Income

For expenses that are not clearly allocable to only taxable or nontaxable income, Reg. §1.265-1(c) requires that a reasonable proportion of the expenses be allocated to exempt income (and thus not be treated as deductible) based on the facts and circ*mstances of the specific situation of the trust.

Note

It is important to note that the regulation does not prescribe a specific method that must be used to allocate these expenses, only requiring that a “reasonable” portion must be treated as allocable to the exempt income. Advisors should be aware, however, that tax software does generally have a “standard” method the software will use to allocate such expenses. However, the advisor is responsible to decide whether the tax software's method is appropriate, as well as to advise the trust about other methods that may be reasonable in light of the facts.

In most cases, there will be more than one reasonable method that could be used and, in some cases, a particular method that generally might be deemed reasonable may, in fact, not be reasonable. The latter case can include the method that is used by default by the advisor's tax preparation software. As well, a method other than the default method used by the advisor's tax software may be more advantageous to the trust.


Allocation Based on Gross Income

A method quite often used as the default by tax software programs is to allocate expenses based on the ratio of total taxable and tax-exempt income. This is done by default for the simple, practical reason that the software generally has access to this information, or at least what it believes is such information.

This method is also popular because IRS Publication 550, Investment Income and Expenses, provides that is one such acceptable method. The 2020 edition, on page 34, provides the following example:

Example 3.11

You received $6,000 interest; $4,800 was tax-exempt and $1,200 was taxable. In earning this income, you had $500 of expenses. You cannot specifically identify the amount of each expense item that is for each income item, so you must divide your expenses. 80% ($4,800 tax-exempt interest divided by $6,000 total interest) of your expenses is for the tax-exempt income. You cannot deduct $400 (80% of $500) of the expenses. You can deduct $100 (the rest of the expenses) because they are for the taxable interest.

Because it is found in an IRS Publication, the method is less likely than others to be challenged by the IRS, but that does not mean it cannot be challenged. Similarly, the method is not required to be used. [Revenue Ruling 63-27, clarifying Revenue Ruling 59-32]


Allocation Based on Value of Assets Generating Taxable and Tax-Exempt Income

When an investment advisor's fee was based on a percentage of assets under management, the IRS convinced the Tax Court that a reasonable allocation was one based on the relative value of the taxable and tax-exempt securities on which the fee was based. [Alt v. Commissioner , TC Memo 1969-292]

While in that case the taxpayer was arguing that the advisor actually spent far more time on the taxable securities, and thus was not allocating the fees based on relative amounts of gross income, arguably the same challenge by the IRS could prevail if there was a significant discrepancy between the income ratio and the method upon which an expense has been calculated.


Entire Course (including glossary) (66)

Allocation Based on Income Types Over the Life of the Trust


Entire Course (including glossary) (67)

The fact that the Publication's suggested reasonable method is not always going to be deemed to be reasonable was pointed out by the IRS in Revenue Ruling 77-466.

In that ruling, the IRS held that a trustee's termination fee, based on service over the entire term of the trust, could not be allocated solely based on the current year's income. In the situation described in the Revenue Ruling, the trustee had disposed of all tax-exempt investments immediately prior to the final year of the trust.

The trust attempted to claim a deduction for the entire termination fee paid to the trustee based on the fact that the trust had no tax-exempt income during the year in question. The IRS ruled that such an allocation method was not reasonable in this case. Rather, the IRS found the proper method for allocation was to look at the trust's income over the entire life of the trust.

To view this interactivity please view chapter 3, page 47

Interactivity information:

Tax Return and Recordkeeping Requirements

Trusts and estates that receive tax-exempt income are required to submit an itemized statement with their tax return showing in detail the amount of each class of exempt income, the amount of expenses directly allocable to the exempt income, and the amount allocated to the exempt income by an allocation method. The statement must disclose the method used to allocate the expenses. A listing of expenses that are claimed to be not in any way allocable to an exempt class of income must also be attached to the return. [Reg. §1.265-1(d)(1)]

The taxpayer is required to maintain all records necessary to make allocations required by this rule. [Reg. §1.265-1(d)(1) and IRC §6001]

Generally, the estate has the right to claim deductions for expenses that arise due to the existence of the trust or estate (fees for trust/estate accountings, legal fees related to the trust, etc.) and expenses for the generation of income.

Like individuals, some itemized deductions are allowed only if they exceed 2% of adjusted gross income under IRC §67. The question of which expenses were subject to the 2% limitation had been a matter of dispute. In 2008, the Supreme Court attempted to provide a test to be used to determine which expenses would and would not be subject to this limitation in the case of Knight v. Commissioner, 101 AFTR 2d 2008-544.

However it wasn't until the issuance of final regulations implementing the provisions of the Knight decision that detailed guidance was provided on those deductions incurred by a trust or estate that are or are not subject to the 2% limitation. [Reg. §1.67-4(d), TD 9664, 2014]

As previously mentioned, the deductions subject to the 2% of adjusted gross income are suspended by the Tax Cuts and Jobs Act of 2017 for tax years beginning after 2017 and before 2026. The TCJA added new IRC Section 67(g) which disallows those deductions. The following discussion of those deductions is for the understanding of the deduction for years when it was or will be available.


Adjusted Gross Income of Trust or Estate

There is no line on Form 1041 that is labeled adjusted gross income like there is on Form 1040. Nevertheless, the tax advisor preparing a Form 1041 needs to compute this number in order to apply the 2% of adjusted gross income limitation.

Per IRC §67(e) the adjusted gross income of a trust or estate is computed in the same manner as it is for an individual, except that additional categories of items will be allowed in computing adjusted gross income. These items are:

  • The deductions for costs which are paid or incurred in connection with the administration of the estate or trust and which would not have been incurred if the property were not held in such trust or estate,

  • The income distribution deduction, and

  • The trust or estate's personal exemption.

The first category of expenses is where the controversy arose since the 2% “haircut” imposed by IRC §67 does not apply to items that are deductible in computing adjusted gross income.


Adjusted Gross Income of Trust or Estate (cont'd)

The Knight case involved the question of just what made up these expenses. Specifically, the question arose about investment advisory fees. The trustee claimed that he had incurred such fees solely because of the duty imposed by the relevant trust law to show he had exercised prudence in handling the investments of the trusts or estates and, therefore, had properly discharged his fiduciary duty. Thus, in the trustee's view, the expenses would not have been incurred had the property not been held in a trust or estate.

The Supreme Court in the Knight decision, to the contrary, held that the test should be whether such expenses are of a type that are ordinarily only incurred in the context of a trust or estate. Since individual taxpayers routinely pay for investment advice, such advice will be subject to the 2% generally even if paid by a trust or estate. So, the Court reasoned, expenses commonly or customarily incurred by an individual would not be included in computing a trust's adjusted gross income under IRC §67(e).

Note

Such miscellaneous itemized deductions, in addition to being subject to the 2% limitation of IRC §67 are also not deductible in computing the trust's alternative minimum taxable income. Since they are not used in computing the trust's adjusted gross income, they will not reduce adjusted gross income for purposes of the excess adjusted gross income calculation for the net investment income tax under IRC §1411.

Fees such as fiduciary or executor fees, in the alternative, clearly would not be incurred outside the context of a trust or estate and, thus, would be included in the computation of the trust's adjusted gross income.


Expenses Customarily or Ordinarily Incurred by an Individual

While the Supreme Court's test sounded simple, in practice it proved much more difficult to implement. Immediately following the publication of the Knight decision, commentators began wondering about how to deal with situations, common when using a bank or other corporate trustee, where the trustee charges a single fee that includes both trustee services and investment advice. The trustee's fee is an expense that clearly would not be incurred if the assets were not held in trust, but the Supreme Court made clear the investment fees needed to be treated as miscellaneous itemized deductions.

The IRS applied a temporary relief provision for such bundled corporate trustee fees in Notice 2008-32 that was in effect until final regulations were issued. Six years later, in 2014, the IRS finally was able to publish the final regulations (found at Reg. §1.67-4(d)) in this area dealing with the matter of bundled fiduciary/investment fees, as well as giving clarification in other areas.

The general rule, found at Reg. §1.67-4(b)(1), provides initially that for the analysis of whether a cost is “commonly” or “ordinarily” incurred by an individual, the fiduciary must look at the type of product or service rendered to the trust or estate and not the label applied to the fee.

The regulation provides that generally costs incurred in defense of a claim against the trust or estate (or the grantor or decedent) would be a cost commonly and ordinarily incurred by an individual. However, it notes that this would not be true if the claim related to the trust's existence, validity, or administration.


Expenses Customarily or Ordinarily Incurred by an Individual (cont'd)

While the regulation does not say so, it should be noted that such expenses would not automatically be deductible or, if deductible, subjected to the 2% limit. For instance, if the trust is involved in the conduct of a trade or business and paid legal fees directly related to a claim related to the business, such legal fees would, if they met the general rules for a deductible business expense, be deductible in computing the trust's adjusted gross income and not treated as a miscellaneous itemized deduction.

Remember that not all deductions allowed to an individual are subject to the 2% haircut and those exceptions will carry over into the treatment of the expenses when paid by a trust or estate.


Entire Course (including glossary) (68)

Specific Costs Addressed in Reg. §1.67-4(d)

The IRS outlined five specific types of costs the IRS believed required specific guidance indicating the costs that would or would not be allowable in computing the trust's adjusted gross income.

Where the IRS has not provided specific guidance, the tax advisor is to apply the general test regarding whether the expense is of a type customarily or ordinarily incurred by an individual to determine whether the expense can be deducted in computing the trust or estate's adjusted gross income.

Ownership Costs

Ownership costs are costs that are incurred simply by being the owner of a piece of property such as condominium fees, insurance premiums, maintenance and lawn services, automobile registration fees and auto insurance costs. Such expenses are commonly or ordinarily incurred by individual owners of such property. [Reg. §1.67-4(b)(2)]

Note

Thus, if they are not includable in the computation of adjusted gross income for an individual (which would be true generally if not incurred in a trade or business, as part of a rental activity, etc.), they would not be deductible in computing adjusted income. Generally, if the expenses did not relate to a business, rental, etc., they would be deductible only if incurred for the production of income as provided for in IRC §212.


Tax Preparation Fees

An item of special interest to most readers is the treatment of tax preparation fees incurred by a trust or estate. The regulation provides that expenses incurred for the following tax returns will be treated as costs not subject to the 2% floor:

  • Estate tax returns

  • Generation-skipping transfer tax returns

  • Fiduciary income tax returns

  • Decedent's final individual income tax returns

The costs of preparing all other returns are specifically subject to the 2% floor. The most prominent type of return not excluded from the 2% limit is gift tax returns, the regulation noting that the cost of preparing such is a cost commonly and customarily incurred by individuals. [Reg. §1.67-4(b)(3)]

Allowable attorney, accountant, and return preparer fees are reported on line 14 of Form 1041.


Investment Advisory Fees

The regulation deals directly with the Knight issues at this point. While the Court held that investment fees are customarily and ordinarily incurred by individuals, the Court found that in unique situations (not present in the Knight case), there could be investment fees triggered solely by unique facts of the matter.

The regulations deal with this issue by providing that investment advisory costs are generally subject to the 2% floor. However, in the special case situations discussed by the Court, the regulation will allow the excess portion of the fee to be treated as not subject to the 2% floor.

The regulation limits this treatment to incremental costs of investment advice beyond those that would normally be charged to an individual investor. Such advice is defined as an additional charge added solely because:

  • The advice is being rendered to a trust or estate, and

  • This is due to either:

    • An unusual investment objective (presumably provided by the trust or estate documents), or

    • There is a need for a specialized balancing of the interests of the parties (beyond the usual balancing of the interests of current beneficiaries and remaindermen), such that a reasonable comparison with individual investors would be improper.


Investment Advisory Fees (cont'd)

Even in those cases, only the amount of the fee in excess of what would normally be charged to an individual will escape the 2% haircut. [Reg. §1.67-4(b)(4)]

Note

As this special rule was added to the final regulation primarily to deal with a rather fuzzy statement made by the Supreme Court regarding a theoretical “special” situation that, while not defined, was held not to be the case in Knight, it seems reasonable to expect the IRS will be unlikely to find many cases they will deem to reasonably allow this splitting of investment fees. More likely, the IRS will argue that the trust or estate in front of them is “just like” the Knight situation in this regard.

Appraisal Fees

Like tax preparation fees, the IRS decided to provide a specific list of appraisal fees not subject to the 2% limit, with all other appraisal fees being considered of a type normally and customarily incurred by an individual. Those fees specifically exempted from the 2% limit are:

  • Fees incurred to determine the fair market value of assets at the decedent's date of death or alternate valuation date,

  • Fees incurred to determine value for purposes of making trust distributions, and

  • Fees incurred to determine values required to prepare the trust or estate's tax return or GST return (such as might need to be done to prepare a split-interest trust return for a charitable remainder unitrust).

All other appraisal fees are deemed to be of a type ordinarily and customarily incurred by an individual, with the regulation specifically providing that appraisals incurred for insurance purposes are of the not excepted type. [Reg. §1.67-4(b)(5)]


Other Costs Identified by the IRS as Not Subject to the 2% Limit

Reg. §1.67-4(b)(6) lists the following additional expenses that will not be treated as subject to the 2% limit:


Entire Course (including glossary) (69)
  • Probate court fees and costs,

  • Fiduciary bond premiums,

  • Legal publication costs of notices to creditors or heirs,

  • Cost of certified copies of the decedent's death certificate, and

  • Costs related to fiduciary accounts.

To view this interactivity please view chapter 3, page 58

Interactivity information:

Bundled Fees

If the estate or trust pays a single fee or expense that covers both costs subject to the 2% limitation and those not subject to such a limit, the fee must be allocated between the costs subject to the limit and those that are not. [Reg. §1.67-4(c)(1)]

Note that this does reach beyond simply corporate trustee fees. The IRS specifically lists the following types of fees as examples of types that may end up with a “mixed” treatment:

  • Fiduciary's fee,

  • Attorney's fee, and

  • Accountant's fee.

The regulation then goes on to explain specific situations.

Hourly Versus Non-Hourly Fees in Bundled Fees

The regulation technically only discusses non-hourly fees at Reg. §1.67-4(c)(2), but by implication it also contains guidance for fees charged on an hourly basis. Specifically, the regulation provides that if a fee is not charged on an hourly basis, only the portion of the expense attributable to investment advice is subject to the 2% floor. Presumably, this means that if an hourly fee is charged, a more comprehensive split of the fee is required, and the issue is not limited to searching for investment advice.

Expenses Specifically Excluded From Allocation

Even though a fee may be paid to a party charging a bundled fee, specific items of that fee will not be subject to allocation. Such fees not subject to allocation include:

  • Any payments made to third parties out of the bundled fee that would have been subject to the 2% floor if paid directly by the estate or trust, and

  • Any expenses separately assessed by payee of the bundled fee for services that are commonly or customarily incurred by an individual.

So, for instance, it would appear that if a corporate trustee were to outsource the investment advice to a third party, that fee paid to the third party would be treated as subject to the 2% limit entirely, even though the trust may have just paid a single fee to the corporate entity. [Reg. §1.67-4(c)(3)]


Reasonable Method of Allocation

The regulation concludes the discussion of bundled fees by providing that any reasonable method may be used to allocate the fees between those subject to the limit and those not subject to the limit. [Reg. §1.67-4(c)(4)]

While giving that broad authority, the regulation then goes on to provide certain factors that can be considered in making the allocation. The regulation does provide that this is not an exclusive list of factors, though clearly drawing from this list may eliminate having to give a detailed defense of the method used if an examination takes place. The listed factors are:

  • Percentage of the value of the corpus subject to investment advice,

  • What a third party advisor would have charged for similar advisory services, and

  • Amount of fiduciary's time devoted to investment advice as opposed to dealing with beneficiaries and distribution decisions or other fiduciary matters.

The provision goes on to remind readers who may somehow have missed the provision found at Reg. §1.67-4(c)(3) described above that they cannot allocate those fees for which allocation is prohibited.


Election to Deduct Administrative Expenses on Form 1041 for an Estate

Generally, administration expenses for a trust would be deductible in computing the trust's taxable income, assuming the expense meets IRC §67(e)'s requirements that the amount:

  • Is paid or incurred in connection with the administration of the trust, and

  • Would not have been incurred if the property were not held in such trust.

Notice 2018-61 issued by the IRS confirmed that expenses allowable under IRC Sec 67(e) remain deductible and it intends to issue regulations clarifying the effect of IRC Section 67(g) on the deductibility of certain expenses incurred by estates and nongrantor trusts.

For an estate there is an additional caveat. The amount may not be deducted in computing the estate's estate tax on Form 706. Similarly, amounts claimed as a casualty loss on the estate tax return are not eligible to be deducted on Form 1041. [IRC §642(g)]

By default these expenses are not allowed to be deducted on the Form 1041 for an estate. However, the executor may make an election under Reg. §1.642(g)-1 by filing a statement with the Form 1041 or a separate statement with the appropriate IRS Service Center for association with the return. The election statement may be filed at any time prior to the expiration of the statute of limitations for the return where the deduction is claimed. [Reg. §1.642(g)-1]


Election to Deduct Administrative Expenses on Form 1041 for an Estate (cont'd)

The statement must provide that the items claimed as deductions on the Form 1041 have not been allowed as deductions from the gross estate of the decedent under IRC §2053 (related to administrative expenses) or IRC § 2054 (related to casualty losses) and that all rights to have such items allowed at any time as deductions under those sections are waived.

Note

Note that this is not an all or nothing election. The executor may choose to claim none of the expenses, all of the expenses, or only certain items on the Form 1041 in lieu of claiming them on the Form 706. [Revenue Ruling 70-361, 1970-2 CB 133] The decision will generally be made by looking at the relative tax benefit from claiming the deduction against each tax. Note, though, that the election is irrevocable once filed, so quite often it makes sense to delay filing the election until the Form 706 is audited by the IRS.


Entire Course (including glossary) (70)

Qualified Business Income Deduction

As previously mentioned, IRC Section 199A allows for a special deduction. In general, a trust or estate computes its deduction based on the qualified business income (QBI), W-2 wages, unadjusted basis immediately after acquisition (UBIA) of qualified property, qualified REIT dividends and qualified Publicly Traded Partnership (PTP) income that are allocated to the trust or estate. Form 8995 or Form 8995-A will be used to calculate the estate's or trust's qualified business income deduction.

The allocation is based on the relative proportion of the trust's or estate's distributable net income (DNI) for the tax year that is distributed or required to be distributed to the beneficiary or is retained by the trust or estate. The allowable deduction will be reported on line 20 of Form 1041. The beneficiaries' apportioned share of the QBI, W-2 wages, UBIA, qualified REIT dividends and PTP income will be reported on Schedule K-1, box 14, code I.

Estates and trusts may deduct up to 20% of their allocated QBI. The calculation and complexity of the Section 199A deduction is beyond the scope of this course, but it is important for advisors to be aware of its availability. The QBI deduction applies only for income tax purposes and it does not reduce the 3.8% net investment income tax.


Chapter 4. Accounting Income and the Applicable Principal and Income Act

This chapter covers the concept of accounting income. When working with trusts and estates, accounting income is governed by state law and the governing documents of the trust or estate rather than Financial Accounting Standards Board (FASB).


Entire Course (including glossary) (71)

Accounting Income

One of the key concepts that advisors must deal with when handling tax returns for trusts is that of accounting income and its computation. As we shall discuss in this chapter, the term has a specific meaning that is defined both by the trust document and by the applicable law governing the trust.

Accounting income is part of the determination of various tax-related matters, including being used in the determination of whether a trust is a simple or complex trust for a particular tax year, and serves as one factor to limit the amount of the trust or estate income distribution deduction for the year. Specifically, the completion of one of two lines on Schedule B of Form 1041 for a trust is generally going to require the knowledge of what is the accounting income of the trust. Line 8 specifically asks for the amount of accounting income for any trust that is classified as a complex trust for the year in question.

If the trust is a simple trust, it may appear that you will not need to provide the IRS with this number on the return, but that is not going to be correct. Line 9 of Schedule B asks for the amount required to be currently distributed under the terms of the trust. By definition, a simple trust is one that requires a distribution of accounting income to be made for the year and does not make any distribution from principal. Thus, line 9 will be the amount of accounting income for a simple trust. Trust accounting is primarily concerned with separating the net assets of the trust into one of two categories—that of trust principal (also referred to as corpus) and that of trust income.

To view this interactivity please view chapter 4, page 3

Interactivity information:

Accounting Income versus Taxable Income

Most advisors in the United States are used to having income for accounting purposes in other contexts determined under the set of standards known as generally accepted accounting principles issued by the Financial Accounting Standards Board (FASB). The standards issued by the FASB provide for reporting on the accrual basis where revenue is recognized when earned and expenses are recognized when incurred, even if the actual receipt or payment of cash has not yet taken place.

However, what is referred to on Form 1041 and in the instructions to Form 1041 as accounting income is neither governed by reference to standards issued by the FASB nor is such income generally determined on an accrual basis.

Rather, accounting income is determined by reference to the state law governing the trust or estate. These laws, based on the legal concepts underlying the governance of trusts, normally require most items to be accounted for on a purely cash basis of accounting.

Not only is this accounting different than advisors are used to when preparing financial reports for business entities, but the concepts are also radically different from those underlying the computation of taxable income under the tax law. Many items that are income under accounting principles are not income for tax purposes and, conversely, a number of items that are income for tax purposes will not be considered part of accounting income.

Complicating matters even more is the concept of “distributable net income” (DNI) that will be discussed in the next chapter. DNI provides yet another income definition, used in computing the income distribution deduction and the nature of income reported to beneficiaries on Schedules K-1, which is also different from other categories of income.

It is crucial that advisors understand the proper computation of each of these types of income, what is included in each, and the impact of the differences on the preparation of a Form 1041, amounts to be reported on Schedules K-1, and the computation of the income tax of the trust or estate.

It's also important to remember that accounting income is computed because it affects the differing rights to distributions of the various beneficiaries. Ultimately, the computation of accounting income tends to eventually lead to a determination of the amount of cash distributions or assets that the various beneficiaries will receive. As such, an error in computing accounting income will quite often lead to one party receiving less than he or she should have received under the terms of the trust or will.

Each jurisdiction has its own law that is meant to address the treatment of trusts. Technically, each state could create a full system of accounting rules for trusts without reference to what any other state has done or what has historically been the treatment of trusts.

In reality, there is an attempt to both keep trust law generally consistent with what has been done previously and to coordinate the statutes between the states so that there is a reasonable level of consistency in the various state laws.

Definition

The Uniform Law Commission of the National Conference of Commissioners on Uniform State Laws has published and updated various proposed uniform laws that are meant to harmonize the various state laws. However, the mere fact the commission proposes a uniform act or revises the act does not make it law—rather, each state legislature must take up and adopt the act. In doing so, the legislatures can, and often do, make some revisions to the proposed text.

One of the uniform acts exists in the area of trust accounting—the Uniform Principal and Income Act. The Act was fully revised in 2000, with significant amendments proposed in 2008. There is a newer version of the act, Fiduciary Income and Principal Act of 2018. As of November 2022 only Arkansas, Colorado, Kansas, Utah, Virginia, and Washington have enacted the 2018 version, although legislation to do so has been proposed in California, Missouri, and Tennessee.


States That Have Not Yet Adopted

While most states have adopted the 2000 version of the Uniform Principal and Income Act along with the 2008 amendments, some states have not. Per the Uniform Law Commission website, as of November 2022, the following states and territories have not yet adopted the base 2000 Uniform Principal and Income Act:

  • Georgia

  • Illinois

  • Louisiana

  • Puerto Rico

  • Rhode Island

  • U.S. Virgin Islands

Move on to the next page to see a graphical representation of the states that have not yet adopted the 2000 version.


States That Have Not Yet Adopted (Cont'd)


Entire Course (including glossary) (72)

States That Have Not Yet Adopted 2008 Amendments

Among those states that have adopted the 2000 Act, a number have not adopted the 2008 amendments. Those states, per the Uniform Law Commission website, include:

  • Alaska

  • Florida

  • Hawaii

  • Massachusetts

  • Minnesota

  • New Hampshire

  • New York

  • Pennsylvania

  • Wyoming

The next page contains a graphical representation of the states that have not yet adopted the 2008 amendments.


States That Have Not Yet Adopted 2008 Amendments (cont'd)


Entire Course (including glossary) (73)

States That Have Adopted With Unique Amendments

While the remaining states, which cover the vast majority of the U.S. population have adopted the 2000 Act and the 2008 amendments, many have made their own unique amendments to various portions of the Act. Thus, the advisor will need to directly consult the appropriate governing state law to determine the proper treatment for a particular trust.


Entire Course (including glossary) (74)

Focus On Uniform Act and 2008 Amendments

This chapter will discuss the provisions of the 2000 Act, after incorporation of the 2008 amendments to the Act. References to provisions of the Act will be to the section numbers found in the Uniform Act. However, advisors should be aware that when the statute is actually enacted by a legislature, the numbering of specific sections is generally modified to conform with the statutory organization structure of the particular state. Thus, an advisor will need to map references given to Uniform Act provisions to the equivalent section in a particular state's statutes.


Entire Course (including glossary) (75)

Fiduciary Income and Principal Act of 2018

The following information is provided on the Uniform Law Commissions website:

The Uniform Fiduciary Income and Principal Act (UFIPA) is a revision of the former Uniform Principal and Income Act with a new name to differentiate the act from its three predecessor versions. While older trusts often had clear delineation between income and principal interests, modern trust accounting requires flexibility. Trustees now tend to invest for the greatest total return, and then adjust between interest and principal to produce a fair result for all the beneficiaries. UFIPA recognizes this trend toward total-return investing and includes unitrust conversion rules to allow even older trusts to take advantage of modern investment trends. UFIPA gives estate planning attorneys additional flexibility to tailor a trust for each client's needs and includes a new governing law section to help avoid jurisdictional disputes.

As of November 2022, Arkansas, Colorado, Kansas, Utah, Virginia, and Washington have adopted the UFIPA. California, Missouri, and Tennessee have introduced the legislation.


Superior Status of Definitions in Trust

One of the important concepts found in the Uniform Act is that the trust document can override any particular treatment specified in the state law. Section 103(a) of the Uniform Act generally provides for the following order that an advisor should consult in determining the proper treatment of an item for purposes of the accounting.

  1. If the trust document requires a particular accounting treatment for an item, that treatment must be respected in the accounting, even if it is contrary to a provision found in the principal and income act. [Uniform Act Section 103(a)(1)]

  2. If the trust grants the trustee discretion to determine the accounting treatment of a particular item, the trustee's determination of the treatment must be followed even, if it is contrary to a provision in the Uniform Act. [Uniform Act Section 103(a)(2)]

  3. If the trust is silent with regard to the treatment of a particular item, then the provisions governing that item in the Uniform Act must be followed. [Uniform Act Section 103(a)(3), (4)]

To compute accounting income for a trust or estate, the advisor will need access to the governing document (the trust or will) along with the applicable state act. To the extent the trust document or applicable act grants the trustee discretion in determinations on accounting issues, the advisor will also need to inquire about the decisions the trustee had made or plans to make in administering the trust or estate.

To view this interactivity please view chapter 4, page 13

Interactivity information:

Applicable State Act

Since the actual statutes found in various jurisdiction are not identical, an advisor will need to become aware of which state or jurisdiction's laws are to be used when administering the trust.

The first place to start in making this determination is to read the trust document. Most trust documents will have a clause that specifies which law is to be used to administer the trust or estate and normally that provision will be binding on the trustee.

However, the actual determination of jurisdiction is a legal matter outside the general realm of taxation. For that reason, confirmation of the proper governing law with the trust's legal counsel is a prudent step for the advisor. That will be especially true if the trust document is silent as to the applicable governing law.

Trust Principal

Trust principal represents, generally, the assets transferred to the trust by the trustor or other transferees. Technically, per the Uniform Act, it represents “property held in trust for distribution to a remainder beneficiary when the trust terminates.” [Uniform Principal and Income Act Section 102(10)]

If trust property is converted into another asset, for instance, into cash by being sold, generally that converted asset remains as part of principal. In addition some cash receipts received by the trust may actually represent just such a conversion of principal.

For instance, if the trust is holding land on which there is a significant oil deposit, payments received as royalties from an entity that contracts to drill and extract the oil, some of those payments would clearly represent the conversion of that oil into cash. By having the oil extracted, the land (which is principal) is being made less valuable.

Trust accounting rules attempt to capture that sort of conversion of principal and ensure that those funds remain available for the principal beneficiary and are not simply distributed out to the income beneficiary as part of trust income.


Trust Income

Trust income, per the Uniform Act, generally means “money or property that a fiduciary receives as current return from a principal asset.” This return is not, under trust accounting principles, any longer a part of the trust principal, but is rather an item of income that is reserved for the benefit of those beneficiaries that hold an interest in trust income.

An obvious example of such an item of income would be interest paid on a trust's savings account at a bank. The cash payment of interest is a return being paid to the trust on the principal that the trust has on deposit with the bank (and thus has loaned to the bank). In this case, the balance in the savings account will become divided between a portion that represents principal (the original funding amount) and income (the interest income that has been deposited back into the account).

Note

Credit shelter trusts used in estate planning traditionally give a surviving spouse a mandatory current income interest, but have restrictions on the spouse's ability to obtain a distribution out of principal, reserving that most often for the decedent's children. Since the tax advisor needs to know trust income to properly complete the Form 1041, the advisor may find himself facing claims if the trust does not properly handle insuring that the proper amount of income has been distributed and that either no distribution was made in excess of that amount or the trustee made the proper determination that the surviving spouse qualified for a distribution out of principal. The exposure can be especially acute in second marriage situations where the principal beneficiaries are not the children of the surviving spouse.


Study Question 10

Which of the following would be the first provision to be consulted when determining whether a cash receipt should be allocated to principal or income in the trust?

AThe treatment specified for the item in the applicable principal and income act.
BA provision in the trust document requiring the cash receipt to be allocated to principal.
CA provision in generally accepted accounting principles.
DThe trustee has historically allocated 50% of this receipt to income and 50% to principal.

Study Question 11

Which of the following does not accurately describe the relationship of the trust document with the applicable principal and income act per the Uniform Principal and Income Act?

AThe provisions of the applicable principal and income act only apply to the extent the trust document provides the terms of the act are to be used.
BIf a trust document requires treatment of an item as income that the act requires to be treated as principal, the trustee must follow the treatment specified in the trust.
CIf a trust document is silent with regard to the treatment of a particular item received by the trust where a treatment is provided by the act, the trustee must follow the treatment specified in the act.
DIf the trust grants the trustee discretion with regard to the treatment of an item where the applicable principal and income act requires a particular treatment, the trustee may determine a treatment other than that required by the act.

General

The accounting provisions of the Uniform Act are principally concerned with the classification of cash receipts and disbursem*nts, dividing each receipt or disbursem*nt between principal and income. Generally, no notice is given of an economic event until such time as it leads to a cash transaction.

Conceptually, this is justified when remembering that the law on trusts is concerned primarily with making distributions to income beneficiaries of current returns and holding principal assets for distribution to principal beneficiaries at the end of the trust. When viewed in that manner, it makes sense not to credit or charge an item to income until such time as it either leads to an influx of cash (from which a distribution can be made) or an outflow of cash (which will reduce the distributions that can be made).

But this does result in some quirks in the accounting that may confuse many advisors when they first encounter trusts and estates. For instance, generally, a trust or estate takes no notice of what takes place inside an entity in which it owns an interest.

So the fact that a partnership in which the trust is a 50% interest holder might have reported $50,000 in interest income allocable to the trust during the year is not a matter of interest for trust accounting. The partnership's share of that interest which may be reported to it on a Schedule K-1 does not lead to trust income that is available for distribution to the income beneficiary.


Default Allocation


Entire Course (including glossary) (76)

The Act provides an overriding default allocation of receipts and disbursem*nts to principal absent a provision either in the trust document or the Act that provides some or all of a particular receipt or disbursem*nt is to be charged to income. [Uniform Principal and Income Act Section 103(a)(4)]

Note

As a consequence of this rule, the trustee must be able to point to specific authority in the trust or governing law to classify a particular receipt or disbursem*nt as allocable to income. Unless such authority exists to treat the item as wholly or partially allocable to income, the item must be accounted for as a principal transaction.

Allocation of Receipts

Article 4 of the Uniform Principal and Income Act contains provisions instructing the trustee, in the absence of contrary provisions in the trust document, on the proper allocation of receipts received by the trust between principal and income.


Funds Received from Entities

Section 401 of the Uniform Principal and Income Act deals with receipts from entities in which the trust holds an interest. Entities encompass a broad range of different types of interests an estate or trust might hold. The Act provides that the term entities includes an interest held in a:

  • Corporation,

  • Partnership,

  • Limited Liability Company (LLC),

  • Regulated Investment Company (Mutual Fund),

  • Real Estate Investment Trust (REIT), and

  • Common Trust Fund. [Uniform Principal and Income Act Section 401(a)]


Exclusions

A few holdings are excluded from entity treatment as provided in Section 401. Those include most interests the trust or estate may hold in a trust or estate, business interests where the trustee had elected to handle the interest under the special rules discussed below, or asset-backed securities governed by their own provisions in the Act. [Uniform Principal and Income Act Section 401(a)]

General Rule

The general rule is that any cash received from the entity is allocated entirely to income (Uniform Principal and Income Act Section 401(b)) while distributions of property are to be allocated entirely to principal. [Uniform Principal and Income Act Section 401(c)(1)]

Example 4.1

The Harry Jones Bypass Trust owns a 10% interest in a partnership that invests in stocks and bonds. The trust's 10% share of interest for the year is $10,000 and its 10% share of qualified dividends is $30,000. The partnership makes no distributions during the year. Because there are no distributions made, the trust does not add any amount to trust accounting income for the year.

The following year, the partnership incurs a net capital loss that offsets its interest and dividend income in its entirety, resulting in a net flow-through income of $0 reported on the K-1. The same year it makes a $30,000 distribution to the trust. In this year, the trust had $30,000 of accounting income from the partnership distribution.


Mutual Funds

A special rule that is tied to the income tax treatment of mutual fund distributions applies to capital gain distributions of a mutual fund. Any distribution treated as a capital gain distribution under federal tax law for a year is treated as allocable to trust principal. [Uniform Principal and Income Act Section 401(c)(4)]

Note

Mutual fund capital gain distributions for federal tax purposes are limited to a fund's net long-term capital gain distributions in excess of short term capital losses the fund may have incurred for the year. While some mutual funds report on statements distributions that arise from short-term capital gains incurred by the fund, such distributions are accounted for as income distributions under the standard rule since they are not capital gain distributions as defined by the Internal Revenue Code.

The commentary included as part of the Uniform Principal and Income Act notes that if a trustee allows a mutual fund to reinvest dividends, this treated as transferring an item out of income and transferring it to principal pursuant to the power to adjust under Act Section 104. The commentary notes that if the trustee is allowing this to happen for a reason other than use of the power to reallocate (for instance, to save brokerage commissions that would otherwise be due from making investments), the trustee should transfer an equivalent amount of cash from principal to income. [Commentary on Act Section 104, Uniform Principal and Income Act]

To view this interactivity please view chapter 4, page 23

Interactivity information:

Liquidations of Trust's Interest in Entity

Distributions in exchange for all or part of the trust's interest in the entity or in full or partial liquidation of the trust's interest in the entity are not allocated to income, but instead are treated as principal receipts. [Uniform Principal and Income Act Section 401(c)(2), (3)]

A payment in exchange for the trust's interest in the entity can be received as a series of liquidations, so long as they represent related distributions for all or a portion of the interest. [Uniform Principal and Income Act Section 401(c)(2)]

Tests for Partial Liquidation

There are two tests that are applied to determine if a distribution from an entity is to be treated as a partial liquidation of the trust's interest in the entity:

Entire Course (including glossary) (77)

The entity indicates, at or near the time of the distribution, that the distribution is intended to be a distribution in partial liquidation of the trust's interest, or

Entire Course (including glossary) (78)

The total amount of money received in the distribution or series of distributions exceeds 20% of the entity's gross assets, as shown by the entity's year-end financial statements initially preceding the initial distribution. [Uniform Principal and Income Act Section 401(d)]

A distribution is not treated as received in partial liquidation, nor is the distribution taken into account for the 20% test cited above, to the extent it does not exceed the income tax that the trustee or beneficiary must pay on the taxable income of the entity. [Uniform Principal and Income Act Section 401(e)]


Distributions from Trusts or Estates

While the Uniform Principal and Income Act does not respect the flow-through of treatment for pure tax pass-throughs like partnerships, it does have its own pass-through treatment for distributions from most interests the trust or estate may have in a trust or estate.

As long as the interest in the trust or estate is not a purchased interest, the trustee or executor must inquire of the trustee or executor of the distributing trust or estate to determine whether the distribution is being made out of principal, income, or both. The trustee or executor allocates the distribution to principal or income based on the source of the distribution from the originating trust or estate. [Uniform Principal and Income Act Section 402]

Example 4.2

The Jones Family Trust receives a $35,000 distribution from the Harry Smith Trust. The trustee of the Harry Smith Trust indicates that $28,000 of the distribution is coming from the principal of the trust, while $7,000 is coming from trust income. The trustee of the Jones Family Trust will treat $7,000 of the distribution as allocable to income and $28,000 as allocable to principal.


Distributions from Trusts or Estates (cont'd)

The commentary notes an exception to this rule if the governing documents of the distributing trust or estate provide for the classification by the receiving trust or estate. If, for instance, the distributing trust has a provision in its governing documents that distributions from it are to be added to principal of the receiving trust, the trustee of the receiving trust is to add the distribution to principal even if the distribution came from the income of the distributing trust. [Commentary on Section 402, Uniform Principal and Income Act]

Example 4.3

The Mary Jones Grandchild Educational Trust is a beneficiary of Mary Jones's estate. Mary's estate provides that this trust is to receive 10% of the residual of Mary's estate which will include an interest in any income earned on estate principal during the administration of estate. The estate's final distribution contains both 10% of the principal that became part of the estate at Mary's death and the trust's 10% share of the income remaining in the estate at that time. Despite the fact that a portion of the distribution arises from income, the provision in Mary's will directing that the trust treat the entire distribution as principal will take precedence and none of the distribution will be allocated to income in Mary Jones Grandchild Educational Trust.

If the trustee has purchased an investment entity that is organized as a trust, these rules do not apply. Rather, distributions from that entity are taxed under the general entity rules or, if applicable, the asset-based security rules in the Act.


Business Conducted by Trust

If a trust owns all or part of an operating business, the traditional trust accounting rules and trust terms may pose significant problems for the business. Accounting for the business activity, whether as part of an entity or as an organization operated directly by the trustee, is likely to result in a computation of income that is very different from the traditional GAAP accounting income. Provisions of the trust that require distributions of trust income could very well place the business in a position of being able to retain far too little cash to be able to continue to operate.


Entire Course (including glossary) (79)

Option to Maintain Separate Accounting Records

Recognizing the issues created by businesses and trust accounting rules, the Uniform Act provides an option for certain interests held by the trust that qualify as a business. If the trustee determines it is in the best interests of all beneficiaries to account separately for the business or activity instead of treating it as part of the trust's general accounting records, the trustee may maintain separate accounting records for the business's transactions. [Uniform Principal and Income Act Section 403(a)]

If the trustee maintains such separate accounts, then the trustee is given the power to determine a reasonable cash accumulation to be retained in the business and not made subject to distribution. In making that determination, the Act provides that the trustee may consider:

  • Cash that must be retained for working capital,

  • Cash retained for the acquisition or replacement of fixed assets, and

  • Cash retained for other reasonably foreseeable needs of the business or activity. [Uniform Principal and Income Act Section 403(b)]

The Act also allows the trustee who has opted to use this separate accounting method to determine the extent to which any other cash receipts or disbursem*nts are to be divided between principal and income. [Uniform Principal and Income Act Section 403(b)]

To view this interactivity please view chapter 4, page 29

Interactivity information:

Eligible Activities

The Uniform Act allows this treatment for activities in the following areas:

  • retail, manufacturing, service, and other traditional business activities;

  • farming;

  • raising and selling livestock and other animals;

  • management of rental properties;

  • extraction of minerals and other natural resources;

  • timber operations; and

  • derivative and option activities as described in the Act. [Uniform Principal and Income Act Section 403(c)]

The commentary provides some additional insight into this provision. The commentary cautions that the provision is not meant to allow a trustee to treat a traditional securities portfolio under these rules to bypass the treatment of such items provided for in the Act.

However, the commentary indicates the provision is meant to give quite a bit of flexibility to the trustee to operate a business that is being run as a proprietorship or a single member LLC. The commentary notes specifically that the trustee could account for this simply as a sub-trust or continue to use the business and record-keeping methods used by the transferor prior to the time the assets came into the estate or trust. [Commentary on Uniform Principal and Income Act]


Entire Course (including glossary) (80)

Items Received from Trustor or Decedent

Assets received from the trustor or decedent are to be classified as additions to principal. [Uniform Principal and Income Act Section 404(a)] The same is true of receipts from a trust with a terminating income interest or a payor under contract naming the trustee as beneficiary. However, there are some important special rules about assets transferred to the trust that may not initially seem reasonable until you remember that trust and estate accounting is very strongly on a cash basis.

Payments Received

A payment received by the trust or the estate related to an asset should be classified wholly as income or principal based on the due date of the payment. The due date for these purposes is the date the payor was obligated to make the payment, not the day the payment is actually made. [Uniform Principal and Income Act Section 302]

Example 4.4

One of the assets transferred by Joe to a trust for his daughter is a bond with interest due on July 1 and January 1. The trust provides that his daughter will receive the income of the trust each year. He transfers the bond to the trust on June 15th. The entire July 1 payment, and not just the portion representing interest earned for the period from June 15th to July 1, will be included in the trust income that is distributed to his daughter.

Similarly, if the bond is transferred on July 2 but the payment is made on August 1, none of the payment will be included in trust income for the year. The payor was obligated to make the payment on July 1, prior to the bond being transferred into the trust.


Income Interest Received from Estate


Entire Course (including glossary) (81)

Another related issue arises when an asset passes to a trust based on a provision in a decedent's will. In that case, the income beneficiary's income interest begins on the day the decedent dies even though the asset may remain part of the estate during a period of administration. [Uniform Principal and Income Act Section 301(b)(2)]

Example 4.5

Under Denise's will, a trust for her daughter in which her daughter has a mandatory income interest, is to be left all of the shares of XYZ Company that Denise held on the day she died. The shares remain in the estate for a year while the executor wraps up Denise's affairs and ensures that enough assets exist to pay off all creditors. Despite the fact that the asset is being held by the estate for that year, any income produced by that asset (such as the issuance of dividends) is to be treated as allocated to trust income.


Proceeds from Sale of Assets

Generally, any proceeds from the sale of assets are to be treated as trust principal. That includes any gain or loss based on the difference between the cost of the asset and what is obtained as part of the sale.

Such a transaction would include a sale, exchange, liquidation, or other change in form of a principal asset. [Uniform Principal and Income Act Section 404(2)]

Note

The fact that appreciation is, by default, given entirely to the principal beneficiary while cash dividends and interest are allocated to income creates issues for investment management in a trust. Strict adherence to these rules made it difficult for a trustee to manage trust investments in a fashion that would be the most prudent, since normally a type of investment selected would cause its benefits to go primarily to either the income or principal beneficiaries. The trustee's power to reallocate has been added to the Act to allow for investing using current investment methods that emphasize diversification of asset types by allowing a trustee to “move” receipts to make up for biases in the traditional allocation methods.


Rental Property

Receipts from the rental of trust property generally are allocated to income, including payments received for the cancellation of a lease or to renew a lease. However, deposits received from tenants are classified as principal and are not released to income until the trust's contractual obligations are satisfied as that amount. [Uniform Principal and Income Act Section 405]

Note

The commentary to the Act points out that this provision only applies if the trustee does not separately account for management of rental properties under the business provisions (found in Act Section 403) discussed earlier.


Entire Course (including glossary) (82)

Receipts from Bonds and Other Types of Debt Instruments

Payments received by a trust under most debt instruments, aside from those acquired less than a year before the instrument matures, are accounted for under a pure cash basis. The Uniform Principal and Income Act's default provisions do not take into account either premiums or discounts that existed when the instrument was acquired by the trust.


Entire Course (including glossary) (83)

Receipts from Bonds and Other Types of Debt Instruments (cont'd)

Thus, any payment received on the debt obligation, whether determined by a fixed, variable, or floating rate, must be added to income without taking into account any amortization of premium. [Uniform Principal and Income Act Section 406(a)]

Similarly, the entire amount received from the sale or other disposition of a debt is added to principal, as well as the funds from any redemption of the debt by the debtor, without regard to any discount, unless the instrument was originally acquired by the trust or estate within one year of its maturity. [Uniform Principal and Income Act Section 406(b)]

If the obligation does mature within one year of when it is acquired by the trust or estate, the excess received by the trustee for the obligation over the amount paid for the obligation is allocated entirely to income. [Uniform Principal and Income Act Section 406(b)]

The commentary included with the Act notes that the allocation to principal rule applies even to inflation indexed bonds. If a bond provides that at maturity the balance due will be an initial face value portion plus an addition to the face balance to account for inflation, that addition will be allocated to principal if the bond was acquired more than one year prior to maturity and all to income if acquired within the one-year window. [Commentary to Uniform Principal and Income Act Section 406]

The commentary also directs the trustee to take into consideration the impact of bonds that do not pay interest currently on his or her decision to exercise the power to adjust between principal and income. [Commentary to Uniform Principal and Income Act Section 406]


Proceeds of Insurance Policies and Similar Contracts

A trustee generally allocates to principal the proceeds of any life insurance policy or other contract that names the trust as a beneficiary, including insurance policies that insure the trust against loss or damage to, destruction of, or loss of title of a trust asset. [Uniform Principal and Income Act Section 407(a)]

If the policy insured the trust against loss of occupancy or other use by an income beneficiary, loss of income from the property, or loss of profits from a business, then the proceeds are to be allocated to income. However, if the policy insuring against the loss of profits from a business relates to a business for which the trustee is using the separate accounting method, then the proceeds will be accounted for under that special provision and not this one. [Uniform Principal and Income Act Section 407(b)]

If the trustee receives a payment of dividends on the insurance policy, the dividends are allocated to the same category from which the expense for payment of premiums is charged. That is, if premiums are paid from income, the dividends are allocated to income, and if the premiums are paid from principal, the dividends are allocated to principal. [Uniform Principal and Income Act Section 407(a)]


Deferred Compensation

Payments covered by this provision are payments to be made over a period of years or during the life of one or more individuals due to services rendered or property transferred to the payor. Programs covered by this rule would include:

  • Employer retirement plans,

  • Individual retirement accounts and annuities, and

  • Annuity contracts. [Uniform Principal and Income Act Section 409(a)]

If the payment received on such an arrangement is characterized as representing interest, dividend, or a substitute payment for either, that amount is allocated to income, with any additional payment received in the same accounting period (which is defined as one year) being allocated to principal. [Uniform Principal and Income Act Section 409(b)]

The commentary clarifies that this provision is not meant to apply to individual retirement accounts or any program with payment provisions similar to an IRA. Rather, as the commentary notes, this provision is meant to deal with programs like phantom stock plans, where a distribution is made to be equivalent to that which would have been received had the trust actually owned the stock in question.

If no part of the payment is characterized as representing interest, dividends, or equivalent and the payment is one required to be made under the terms of the arrangement, 10% of the amount received will be allocated to income and 90% to principal. If no part of the payment is required to be made or the trustee receives the entire balance due, the entire payment will be allocated to principal. A trustee exercising a right to withdraw is taking a distribution that is treated as not required to be made. [Uniform Principal and Income Act Section 407(c)]


Special Rules

Special rules apply if the trust meets one of two tests based on the Internal Revenue Code. [Uniform Principal and Income Act Section 407(d)]

Qualified Terminable Interest Property (QTIP)

If the trust in question has made an election under IRC §2056(b)(7) for a qualified terminable interest (a QTIP election), that election causes property in the trust in which the spouse has only a qualified life estate, nevertheless, to be treated as includable in that spouse's estate when he/she dies and qualifies for the unlimited marital deduction. However, such a trust must meet detailed requirements, including the right of the spouse to all income from the trust.

Power of Appointment

If the trust is one in which the spouse retains a power of appointment under IRC §2056(b)(5) in which the spouse retains an income interest for life, again, such a trust is treated as passing to the surviving spouse and deemed eligible for the unlimited marital exclusion. Again, like the QTIP trust, the spouse must have an absolute right to all income for life. [Uniform Principal and Income Act Section 407(d)]

The rules apply to “separate funds” which the Act defines as:

  • Private or commercial annuities,

  • Individual retirement accounts, and

  • Pension, profit-sharing, or stock bonus plans.

The special provisions apply only if the trust would not otherwise qualify for the unlimited marital deduction unless the special “look through” rules are applied. In that case, the trustee must first attempt to determine the internal income of each separate fund. If the trustee receives a demand from the surviving spouse, the trustee shall demand that the entity administering the separate fund distribute the internal income to the trust.

As the commentary notes, the IRS has ruled in Revenue Ruling 2006-26 that the default provisions of the Uniform Act's treatment would cause a trust not to qualify for the QTIP safe harbors, since the plan interest must separately meet the QTIP requirements.

The spouse can also require the trustee to make a transfer from principal to income if the distributions from the separate fund are less than the internal income of the fund. [Uniform Principal and Income Act Section 407(f)]

If the trustee cannot obtain the internal income of the separate account, a percentage provided for by the applicable state act multiplied times the fund's value will be treated as income. The Uniform Act provides a suggested percentage range from 3% to 5% that a legislature should adopt.

If the trustee cannot determine either the internal income of the separate fund or its value (for instance, in the case of payments coming from a defined benefit arrangement), the internal income of the fund is the specified rate noted above times the value of expected future payments computed under the provisions of IRC §7520 for the month preceding the accounting period for which the computation is made. [Uniform Principal and Income Act Section 407(g)]

Note

Note that these rules produce income numbers that are different than the taxable income arising from distributions from these arrangements, most often resulting in substantially lower amounts of accounting income from the distribution than the amount treated as the taxable portion of the distribution.

The commentary notes that these assets share characteristics generally with liquidating assets, covered in the next section. However, even though these distributions would meet the requirements to be classified as liquidating assets, they are given their own specific treatment, rather than being handled in the same manner as other assets that meet the definition of a liquidating asset.


Liquidating Assets

Some assets are expected to decline in value over time. These assets are referred to as “liquidating assets” under the Uniform Act which specifies that they include:

  • Leaseholds,

  • Patents,

  • Copyrights,

  • Royalty rights, and

  • Right to receive payments over a period of more than one year that does not provide for payment of interest on the unpaid balance. [Uniform Principal and Income Act Section 410(a)]

The Act specifically excludes certain assets from this classification. That includes:

  • Deferred compensation described in the previous section,

  • Timber (which has its own rule described later), and

  • Mineral, water, and other natural resources (the treatment of which is described in the following section). [Uniform Principal and Income Act Section 410(a)]

For items covered by this rule, the trustee is to allocate 10% of the receipts to income and 90% to principal. [Uniform Principal and Income Act Section 410(b)]

The commentary with the Act notes that this provision is meant to cover lottery rights.

To view this interactivity please view chapter 4, page 43

Interactivity information:

Interests in Natural Resources

The Act provides rules for different types of payments received with regard to natural resources. For interests in a mineral or other natural resource (except water which will be discussed below), the trust must allocate:

  • Receipts received as a nominal delay rental or nominal annual rent on a lease entirely to income. Note that if the amount is more than nominal, this particular provision will not apply.

  • If a payment is received as a production payment, if the agreement creating the production payment provides an amount designated as interest or an equivalent, that amount is allocated to interest. Any other amounts received on such a payment must be allocated to principal. Thus, if there is no interest provision, the entire payment would be allocated to principal.

  • If the amount is received as a royalty, shut-in payment, take-or-pay payment, or a bonus, 90% is allocated to principal and 10% to income. The same treatment will apply to a delay rental that is more than nominal.

  • Any amount received as a working interest or other payment not specifically provided for in the previous points, the payment is to be allocated 90% to principal and 10% to income. [Uniform Principal and Income Act Section 411(a)]

If the payments relate to an interest in water, then the accounting depends on whether the water rights are renewable. Water rights that are renewable are allocated entirely to income. If the rights are not renewable, then 90% must be allocated to principal and the remaining 10% to income. [Uniform Principal and Income Act Section 411(b)]

A special rule is found in this provision that states if a trust had an interest in such an item prior to the effective date of the Act (which will vary from state to state), the trustee may allocate under the rules in place before the adoption of the Act or by using the above provision. If the interest was acquired after the effective date of the Act for the governing state, then the Act's rules are to be used. [Uniform Principal and Income Act Section 411(c)]

As with other rules for activities that are also included in the business section, this rule does not apply if the trustee is accounting for this activity under the business rule.


Entire Course (including glossary) (84)

To view this interactivity please view chapter 4, page 45

Interactivity information:

Interests in Timber

Timber is subject to another special set of rules for the allocation of receipts between principal and income. The rule takes into account the fact that timber grows, so that additional timber becomes available from the land as time passes.

  • To the extent that the amount of timber does not exceed the rate of growth during the accounting period in which a beneficiary has a mandatory income interest, net receipts are allocated to income. [Uniform Principal and Income Act Section 412(a)(1)] The method for determining the rate of growth is to be selected by the trustee, taking into account the methods customarily used for the kind of timber involved. [Act Commentary]

  • Receipts are to be allocated to principal to the extent the amount of timber removed exceeds the rate of growth or the net receipts are from the sale of standing timber. [Uniform Principal and Income Act Section 412(a)(2)]

  • Receipts are to be allocated between principal and income using the rules in the prior two bullet points if the net receipts are from the lease of timberland or from a contract to cut timber from land owned by the trust. [Uniform Principal and Income Act Section 412(a)(3)]

  • Finally, receipts from advance payments, bonuses, and other payments not allocated under the earlier rules are to be allocated entirely to principal. [Uniform Principal and Income Act Section 412(a)(4)]

Interests in Timber (cont'd)


Entire Course (including glossary) (85)

However, before applying these rules, the trustee will need to transfer to principal a reasonable amount for depletion. This is the reason all of the above rules refer to net receipts in their description. [Uniform Principal and Income Act Section 412(b)]

As with the provision on natural resources, this provision also has a special date of enactment interest rule that gives the trustee the discretion to select either the prior method or the method on this Act for any interest held on the effective date of the Act. [Uniform Principal and Income Act Section 412(d)]

However, any other interest will be subject to the rules of the Act, even if the timber was held by the grantor or decedent previously, unless the trust document provides differently or the trustee is eligible to use and does use the business interest accounting rules for the timber. [Uniform Principal and Income Act Section 412(c) and Act Commentary]


Property Not Productive of Income

Section 413(a) of the Uniform Principal and Income Act provides for the right of a surviving spouse to force a trustee to make the property more productive of income if it is necessary to obtain the estate tax marital deduction.

As the commentary notes, this provision was added to eliminate tax concerns surrounding removing a provision in the 1962 Act that allowed the income beneficiary the right to force the trustee to receive a portion of unproductive property as “delayed income” for these purposes. The old provision put the principal and income act into conflict with the Uniform Prudent Investor Act which required evaluating a trustee's investment and management decisions based in the context of the portfolio as a whole.

Under the old act, this test was applied on an asset-by-asset basis, thus there was no “credit” given for the fact that the trustee might have invaded principal for the benefit of that beneficiary under other provisions of the trust.

Under the revised act, consideration was given to the return as a whole along with the trustee's power to adjust. However, under the regulations governing valuation of interests using actuarial values under IRC §7520 (found in Regs. §1.7520-3 and 25.7520-3), such methods could not be used if the beneficiary did not have the right to compel the trustee to make the property productive.


Property Not Productive of Income (cont'd)

Under the rule in the current Act, the trustee is not simply required to sell the property (the only option under the prior Act), but rather can:


Entire Course (including glossary) (86)
  • Make the property productive of income,

  • Convert the property,

  • Transfer funds from principal to income under the power to adjust, or

  • Take some combination of those actions. [Uniform Principal and Income Act Section 413(a)]

If such action is not required to maintain the marital deduction (for instance, for a trust where both principal and any accumulated income will be part of the surviving spouse's estate), then these rules do not apply and any sale of the asset will be treated as consisting of principal. [Uniform Principal and Income Act Section 413(b)]


Derivatives and Options

If a trustee does not account for derivatives under the trade or business rule (either because there is no trade or business or the trustee does not decide that method is appropriate in the situation), then all receipts and disbursem*nts from derivative transactions are allocated to principal. [Uniform Principal and Income Act Section 414(b)]

For purposes of this section, a derivative is defined to mean a contract or financial instrument (or a combination of such contracts and instruments) that gives the trust the right to participate in some or all of the changes in:

  • The price of a tangible or intangible asset or group of assets,

  • A rate, an index of prices or rates, or

  • Another market indicator for an asset or a group of assets. [Uniform Principal and Income Act Section 414(a)]

Options to acquire or purchase property are to be allocated to or taken from principal if paid for or allocated to principal if received by the trust. The treatment is the same regardless of whether the trust owns the property in question when the option is granted. Any gain or loss realized upon the exercise of an option, including an option granted to the settlor of the trust for services rendered, must be allocated to principal. [Uniform Principal and Income Act Section 414(c)]

Note

The commentary for this section points out that the rule will not apply if the trustee does not directly invest in derivatives and options, but rather holds an interest in an entity (such as a partnership) that invests in such items. In that case, the standard rules for funds received from entities would apply. Similarly, the fact that the trustee might be using a “mark-to-market” treatment for other purposes (tax, for instance) will not create adjustments to principal or income for trust accounting purposes. However, if the trustee receives property other than cash to settle the derivative transaction, that property becomes principal.


Asset-Backed Securities

An asset backed security is defined as an asset whose value is based upon the right it gives the trust to receive distributions from the proceeds of financial assets that serve as collateral for the security. [Uniform Principal and Income Act Section 415(a)] As the commentary notes, such assets generally include packages of the following types of debt that are sold to investors:

  • Real estate mortgages,

  • Credit card receivables, and

  • Auto loans.


Entire Course (including glossary) (87)

Asset-Backed Securities (cont'd)

Such securities are also sometimes sold in a manner so that an investor has rights only to the interest or principal of the underlying basket of instruments. It also includes securities that allow the trust to participate only in the capital appreciation of an underlying security or the interest or principal return (the commentary cites as an example Primes or Scores issued by Americus Trust). [Commentary for Section 415]

If the trust receives a payment from both interest or other current return and other proceeds from the financial instrument, the trustee is to allocate to income the portion the payor identifies as related to interest or current return, and allocate all remaining payments to principal. [Uniform Principal and Income Act Section 415(b)]

If a trust receives one or more payments in a single accounting period (one year) in exchange for its entire interest in the asset-backed security, the trustee allocates the payment to principal. [Uniform Principal and Income Act Section 415(c)]

Note

If a payment comes from an instrument that does not meet the definition of a derivative or any other type of instrument specifically defined in the Act, the commentary to Section 415 reminds us that any amounts received from that instrument would be allocated to principal under the default rule. In such a case, the commentary continues, the trustee would then consider whether a discretionary allocation of some portion of that receipt to income would be proper use of the trustee's power to reallocate.


Insubstantial Allocations

The detailed rules can create a significant administrative burden with a trust holding a variety of assets where the trustee would need to apply various special rules to holdings of the trust that will have little impact on final result in determining trust income for the year in question. In recognition of this issue, the Act does provide the trustee with the option of allocating such receipts entirely to principal if certain tests are met.

  • This rule can be used for otherwise required allocations of deferred compensation, mineral, water, and other natural resources, timber, and asset-backed securities which meet the insubstantial test described below.

Note that derivative transactions are not included in that list. The insubstantial allocation rule may not be used if the trustee would be prohibited from making a discretionary adjustment due to the circ*mstances outlined in the section discussing the trustee's general power to reallocate. [Uniform Principal and Income Act Section 408]

To use this section the trustee must determine that the allocation is insubstantial. The Act provides an allocation is presumed to be insubstantial if:

  • The allocation would serve to increase or decrease net income (as determined before taking the allocation being tested into account) by less than 10%; or

  • The value of the asset generating the receipt that leads to the allocation makes up less than 10% of the trust's assets at the beginning of the accounting period (normally the beginning of the year). [Uniform Principal and Income Act Section 408]


General

Just as receipts must be divided between principal and income in performing an accounting for the trust, disbursem*nts also must be charged to one or the other category, or split between the two.

As with receipts, the default treatment for disbursem*nts is to treat them as coming from trust principal. [Uniform Principal and Income Act Section 103(a)(4)] The trust document can, as with receipts, override the treatment specified under the applicable Principal and Income Act. [Uniform Principal and Income Act Section 103(a)]


Entire Course (including glossary) (88)

Trustee, Investment Advisory, and Custodial Service Compensation

Regular Compensation

The regular compensation paid to any person providing trustee services provided by the trustee, investment advisory services, and custodial services to the trust is to be divided evenly between principal and income. [Uniform Principal and Income Act Sections 501(1), 502(a)(1)]

A trustee's regular compensation includes amounts computed as a percentage of the trust's principal, income, or both. [Commentary to Act Section 501]

All of a trustee's compensation calculated on principal as a fee for acceptance, distribution, or termination along with any payments to the trustee to prepare property for sale are to be allocated wholly to principal. [Uniform Principal and Income Act Section 502(a)(2)]

Accountings, Judicial Proceedings, and Other Matters

One half of expenses for accountings, judicial proceedings or other matters that involve both the principal and income interests of the trust are to be charged to income, with the remainder going to principal. [Uniform Principal and Income Act Section 501(2)]

Conversely, expenses of a proceeding to construe the trust, or to protect the trust or its property are to be charged entirely to principal. [Uniform Principal and Income Act Section 502(a)(4)]


Expenditures for Expenses Incurred in Administration, Management, or Preservation of Trust Property

General expenses incurred in the administration, management, or preservation of trust property are charged to income. Also charged to income are any expenses related to the distribution of trust income. Expenses covered include:

  • Interest,

  • Ordinary repairs,

  • Regularly recurring taxes assessed against principal, and

  • Expenses of a proceeding that concerns primarily the income interest. [Uniform Principal and Income Act Section 501(3)]

The regularly recurring taxes would include item such as property taxes.


Insurance

Insurance can end up being charged against either principal or income, depending on the nature of the premium.

Recurring premiums on insurance covering the loss of a principal asset, or the loss of the income or use of an asset is charged against income. [Uniform Principal and Income Act Section 501(3)] The commentary points out that the recurring requirement exists to distinguish things such as annual premiums for insurance, properly chargeable to income, from one-time payments such as title insurance. Even though title insurance does serve to protect against a loss of an asset, the fact that it is not a recurring charge means that it does not fall under this income rule. [Commentary on Section 501]

Rather premiums paid on any policy not meeting the requirements of Section 501(3) of the Act are charged against principal pursuant to Section 502(a)(5).

Transfer Taxes

A charge is made to trust principal for the payment of any estate, inheritance, or other transfer taxes (such as gift taxes), as well as any penalties that are apportioned to the trust. [Uniform Principal and Income Act Section 502(a)(6)]

To view this interactivity please view chapter 4, page 57

Interactivity information:

Debt Payments

Payments on debt are divided between principal and interest. As was noted in an earlier discussion, interest paid by the trust is charged against income. [Uniform Principal and Income Act Section 501(3)] But payments on the principal of the debt are to be charged against trust principal. [Uniform Principal and Income Act Section 502(a)(3)]

Similarly, if a trust principal asset is encumbered with a debt where the trust has agreed that income from the property will be paid directly to the creditor, the trust must transfer an amount from principal to income in the amount of that diverted income that was used to reduce the principal of the debt. [Uniform Principal and Income Act Section 502(b)]

Environmental Expenses

Disbursem*nts related to environmental matters are charged against trust principal. Such expenses include the following items specifically listed in the Act:

  • Reclamation,

  • Assessing environmental conditions,

  • Remedying and removing environmental contamination,

  • Monitoring remedial activities and the release of substances,

  • Preventing future releases of substances,

  • Collecting amounts from persons liable or potentially liable for the costs of those activities,

  • Penalties imposed under environmental laws or regulations and other payments made to comply with those laws or regulations,

  • Statutory or common law claims by third parties, and

  • Defending claims based on environmental matters. [Uniform Principal and Income Act Section 502(a)(7)]

The commentary justifies this treatment of environmental expenses as being charged against principal by noting that such expenses would generally be an unusual event. If the trust is conducting a business where such expenses would regularly occur, the commentary notes that the trustee deciding to account for the business activity separately would not apply this provision.


Income Taxes

Income taxes, despite their name, are not automatically allocated to income. Rather, the law reflects the reality that income tax rules for what is income differ significantly from those used for trust accounting purposes. Thus, the law looks to the nature of receipts that give rise to an income tax and allocates the tax between principal and income based on the treatment of the underlying receipt under the trust document and/or principal and income act.

If the tax arises from receipts allocated to income, the tax is allocated against income. [Uniform Principal and Income Act Section 505(a)] Alternatively, if the tax arises from receipts allocated to principal, then the tax is allocated to trust principal. [Uniform Principal and Income Act Section 505(b)]


Entire Course (including glossary) (89)

Taxes Related to Pass-Through Income

One of key differences between taxable income and trust accounting principles relates to the taxation of entities, especially pass-through entities. The receipts from the S corporation or partnership generally don't trigger the income tax at the entity level—rather it is the flow-through income of the entity that triggers the tax to the partners or S corporation shareholders.

The Uniform Principal and Income Act modified this in the most recent amendments to the Act. Under the most recent Act, the tax must be paid:

  • From income to the extent that receipts from the entity are allocated only to income;

  • From principal to the extent that receipts from the entity are allocated only to principal;

  • Proportionately between principal and income to the extent that the receipts from the entity are allocated to both principal and income; and

  • Entirely to principal to the extent the tax exceeds the receipts from the entity. [Uniform Principal and Income Act Section 505(c)]

Finally, after applying these rules, the trust shall adjust principal or income receipts to the extent that the trust's taxes are reduced due to a distribution to a beneficiary. [Uniform Principal and Income Act Section 505(c)]


Adjustments Based on Taxes

The Act allows a trustee to make adjustments between principal and income for amounts to adjust for the shift in economic interests that result from:

  • Elections and decisions (aside from certain deductions related to estate tax issues) that the trustee makes regarding tax matters,

  • A tax imposed upon the fiduciary or beneficiary as a result of a transaction involving the estate or trust (including a distribution), or

  • The holding of a pass-through entity that is taxed to the estate or trust without regard to whether a distribution is made to the estate or trust. [Uniform Principal and Income Act Section 506(a)]

If there is a reduction in the estate marital deduction or charitable contribution deduction because the trustee elected to treat a distribution from trust principal as a deduction for income tax rather than estate tax purposes, the trustee shall reimburse principal from each estate, trust, or beneficiary that benefits from the decrease in tax. [Uniform Principal and Income Act Section 506(b)]


Depreciation


Entire Course (including glossary) (90)

For trust accounting purposes, the term depreciation refers to a reduction in value due to wear, tear, decay, corrosion, or gradual obsolescence of a fixed asset. [Uniform Principal and Income Act Section 503(a)] Thus, it is not like either tax depreciation (which is based on simply spreading the cost of an asset over a specified period of time) or traditional business generally accepted accounting principles depreciation which looks to match the cost of the property, less an amount the property is expected to be worth at the end of its useful life, against income generated by that property.

The trustee may transfer to principal a reasonable amount of the net cash receipts from a principal asset that is subject to depreciation (that is, a decline in value described above), except that no amount may be transferred for depreciation on:

  • Real property used or made available for use by a beneficiary as a residence, or

  • Tangible property held or made available for the personal use of a beneficiary, during the administration of a decedent's estate.


Trustee' Power to Reallocate

Generally, if a trustee invests and manages trust assets as a prudent investor, the terms of the trust describe the amount that may or must be distributed to a beneficiary by referring to trust income. The trustee may reallocate between principal and income as the trustee considers necessary. [Uniform Principal and Income Act Section 104(a)]

In making this decision, the trustee is directed to consider:

  • The nature, purpose, and expected duration of the trust;

  • The intent of the settlor;

  • The identity and circ*mstances of the beneficiaries;

  • The needs for liquidity, regularity of income, and preservation and appreciation of capital;

  • The assets held in the trust; the extent to which they consist of financial assets, interests in closely held enterprises, tangible and intangible personal property, or real property; the extent to which an asset is used by a beneficiary; and whether an asset was purchased by the trustee or received from the settlor;

  • The net amount allocated to income under the other sections of the Act and the increase or decrease in the value of the principal assets, which the trustee may estimate as to assets for which market values are not readily available;

  • Whether and to what extent the terms of the trust give the trustee the power to invade principal or accumulate income, or prohibit the trustee from invading principal or accumulating income, and the extent to which the trustee has exercised a power from time to time to invade principal or accumulate income;

  • The actual and anticipated effect of economic conditions on principal and income and effects of inflation and deflation; and

  • The anticipated tax consequences of an adjustment. [Uniform Principal and Income Act Section 104(b)]


Study Question 12

Which of the following most accurately describes the treatment of deferred compensation under Act Section 409?

ANonqualified annuities held in investment accounts are not subject to its provisions.
BA distribution of a “dividend” received under a phantom stock program by an estate would be treated as 10% allocated to income and 90% to principal.
CIf a trustee takes a fully taxable distribution from an IRA when there is no mandatory distribution, allocate the entire distribution to income.
DIf a trustee receives a full distribution of the decedent's balance in a supplemental employee pension under terms of the program that mandate full distribution in the year following the death of the employee, allocate 100% to principal.

Chapter 5. Income Distribution Deduction (Sch. B)

This chapter discusses the distribution deduction available for income distributions to beneficiaries. It walks the learner through the calculation of distributable net income and the distribution deduction.


Entire Course (including glossary) (91)

Entire Course (including glossary) (92)

One final deduction needs special consideration. This deduction is computed on Schedule B of Form 1041, found on page 2 of the form. The income distribution deduction allowed for amounts paid, credited, or required to be distributed to beneficiaries is limited to DNI, a key concept in trust accounting. This amount is also used to determine how much of an amount paid, credited, or required to be distributed to a beneficiary will be includible in his or her gross income.

Distributable Net Income (DNI)

The maximum income distribution deduction available to an estate or trust will be determined by the computation of the distributable net income of the trust or estate. DNI is a tax concept whose definition is found in IRC §643(a). The definition is a purely mechanical one tied to the definition of taxable income and is based principally on the tax treatment of items as opposed to their accounting income treatment.


Calculation of Distributable Net Income

Distributable net income is the taxable income of the trust or estate for the tax year adjusted as follows:

  • No deduction is allowed for distributions.

  • No deduction is allowed for a personal exemption to the trust or estate.

  • Capital gains/losses are generally excluded from DNI, except for the following:

    • Capital gains allocated to accounting income by the terms of the trust document or local law which the trustee accounts for as part of accounting income.

    • Capital gains contributed to charity for which a charitable deduction is allowed.

    • Capital gains used in determining the amount to be distributed to the beneficiary under the terms of the trust document or in accordance with the practice of the fiduciary.

    • Capital gains that are allocated to principal but are distributed to beneficiaries in full or partial liquidation of the beneficiary's interest. [Reg. §1.643(a)-3(e), Examples 7, 9]

    • Capital losses to the extent they are taken into account in determining the amount of gain of capital assets paid, credited, or required to be distributed to the beneficiary during the tax year.

    • Short-term capital gain dividends from mutual funds. (For federal tax purposes, these amounts are treated as dividends and not capital gains. However, long-term capital gain dividends are treated like any other capital gain for these purposes.)


Calculation of Distributable Net Income (cont'd)


Entire Course (including glossary) (93)
  • No deduction is allowed for estate taxes paid on income in respect of a decedent. [Reg. §1.691(c)-2(a)(2)]

  • The gain on the disposition of small business stock allowed under IRC §1202 is added back to taxable income in the computation of DNI.

  • Tax-exempt interest reduced by expenses not allowed in computing the estate's taxable income because they were attributable to tax-exempt interest and the portion of tax-exempt interest deemed to have been used to make a charitable contribution deduction is includible in DNI.

  • In the case of a simple trust only, extraordinary dividends are excluded from the calculation of DNI if the fiduciary, in good faith, does not pay or credit them to any beneficiary by reason of his or her determination that such dividends are allocable to principal under the terms of the governing instrument and applicable local law.

  • The section 199A deduction is not included in distributable net income.

The DNI of the trust or estate is calculated on Schedule B, lines 1-7 on page 2 of Form 1041.

Example 5.1

The Maroon Trust has adjusted total income on line 17 of Form 1041 of $20,000 for the year. Included in that income is $5,000 of long-term capital gains that were allocated to principal in the trust accounting and reported on line 4, page 1 Form 1041. The trust also received $3,000 of tax-exempt interest but had no Section 212 expenses to offset this income.

Schedule B, line 1 adjusted total income is generally line 17 from Page 1 of the 1041.

Schedule B, line 2, will report $3,000 for adjusted tax-exempt income. Adjusted tax-exempt income considers any Section 212 expenses that are directly allocable to tax-exempt income or indirect expenses that are reasonably allocated to tax exempt income. The expenses are subtracted from the tax-exempt income to report an adjusted tax-exempt income figure.

Schedule B, line 6, will report as a negative number the net capital gain, ($5,000).

The trust's distributable net income, Schedule B line 7 combines line 1 through 6 of Schedule B. It is $20,000 (adjusted total income) + $3,000 (tax-exempt income) + ($5,000) (long-term capital gains entered as a negative number on line 6 of Schedule B), or $18,000.

Note

The calculation of distributable net income generally is not affected by what is or is not included in accounting income of the trust or estate with the notable exception of capital gains. As will become clear later, this can create surprising results as the trust document may limit distributions to being made from accounting income, but the Schedule K-1 will tax the beneficiary on his or her share of DNI, including items of DNI that the beneficiary may not have a right to receive and which may be taxed less favorably than the type of income included in accounting income. Conversely, since there is no adjustment for deductions that are allocable to principal, the beneficiary also may end up effectively getting a tax benefit from deductions that have no impact on the amounts the beneficiary will eventually receive so long as trust accounting income ends up being greater than DNI.


Study Question 13

Which of the following is not correct regarding the computation of the distributable net income of a trust or estate?

ADistributable net income includes the deduction for a personal exemption.
BThe amount of any IRA distribution allocable to principal under the terms of the trust is removed from distributable net income.
CNo deduction is allowed for estate taxes paid on income in respect of a decedent.
DCapital losses in excess of capital gains are added back to adjusted total income in the computation of distributable net income.

The trust's income distribution deduction is computed differently depending on whether the trust is treated as a simple trust or complex trust during the tax year in question.

Income Distribution Deduction for Simple Trusts

IRC §651 provides for the income distribution deduction for a simple trust. A simple trust is one that is required to distribute accounting income currently to the trust's income beneficiary or beneficiaries and that does not make a distribution from corpus during the tax year.

Note

As was noted earlier, the status of a trust can and does change from year to year. The fact that a trust was a simple or complex trust in a prior year does not mean the trust will be treated the same in the following year. Rather, the tax advisor must confirm the proper classification of the trust for the tax year being prepared.

For such a simple trust the income distribution deduction is the lesser of:

  • The amount of accounting income (computed under the terms of the trust and/or applicable state law) required to be distributed currently, or

  • The trust's distributable net income, reduced by net tax-exempt income (that is, tax-exempt income reduced by nondeductible expenses related to such income) for the tax year.

Because the trustee is required to distribute the income, the tax law does not require that actual payment be made prior to year end. [Reg. §1.651(a)-2(a)] Similarly, the beneficiaries will need to pay taxes on the amounts that flow to the K-1 in the tax year even if the income distribution is not made by year-end.

To view this interactivity please view chapter 5, page 8

Interactivity information:

Example 5.2

The Carson Dexter Trust is a simple trust that is required to annually distribute income to Bill Dexter, Carson's son. The trustee determines the trust income distribution deduction as follows:

  • The trustee determines the trust's accounting income for the year. In this case, the trustee determines that income was $15,000.

  • The trustee then computes the trust's distributable net income. In this case; the trustee determines that amount is $12,000.

  • The trustee compares the DNI to the amount of income required to be distributed, with the lesser of the two to be the income distribution deduction. In this case, that is $12,000.

  • The trustee will then analyze the nature of the items that will be used to determine the character of income to be distributed to the beneficiaries on the K-1.

Income Distribution Deduction for Estates and Complex Trusts

IRC §661 governs the income distribution deduction in the case of a decedent's estate or a complex trust.

Note

Recall that any trust that does not meet the definition of a simple trust will be treated as a complex trust. Thus, the advisor may view the rules found in IRC §661 as the “default” rules for the income distribution deduction with the provisions at IRC §651 being the “special” rules that apply only in the limited case of a simple trust.

  1. A complex trust has a two tiered distribution system. The tier a distribution comes from will be important in determining how income is allocated on the beneficiary's K-1.

  2. The income distribution is computed as the total of:

    • Any amount of income for such taxable year required to be distributed currently, including any amount required to be distributed which may be paid out of income or corpus to the extent such amount is paid out of income for such taxable year (Tier I); and

    • Any other amounts properly paid or credited or required to be distributed for such taxable year (Tier II).

  3. The income distribution deduction for the complex trust, as is the case with the simple trust, is limited by distributable net income. If DNI is less than the amount computed under the basic distribution rule, the income distribution deduction will be limited to DNI. [IRC §661(a)]

  4. The deduction is further limited by removing any tax-exempt income included in the amount, reduced by any nondeductible expenses allocable to that tax-exempt income. [IRC §661(c)]

  5. If DNI limits the distribution deduction, it is first absorbed by Tier I. If any DNI remains after the first tier amount is deducted from DNI, the remaining balance is treated as part of the Tier II distribution. If there are charitable beneficiaries, a distribution to those beneficiaries is deemed to fall between the Tier I and Tier II distributions.

To view this interactivity please view chapter 5, page 10

Interactivity information:

Example 5.3

The Mollie Aster Trust is required to distribute income up to $12,000 to Max Aster each year. At the trustee's discretion amounts may be distributed to Max's sister, Maureen, each year to provide for her needs. To compute the income distribution deduction for the Mollie Aster Trust the trustee takes the following steps.

  • The trustee determines the trust's accounting income was $15,000.

  • The trustee determines the Tier I amount is $12,000, since trust income was in excess of that amount.

  • The trustee had made additional distributions of $5,000 to Maureen under his discretionary power during the year. The $5,000 represents the Tier II distributions.

  • The trustee computes the trust's distributable net income. The trustee determines that DNI of the trust is $16,000.

  • The trustee determines the income distribution is $16,000, which is the lesser of the DNI ($16,000) or the total of the Tier I and Tier II distributions.

  • The trustee then determines the nature of the items making up DNI to determine the character of the income passed out on each beneficiary's K-1. Note that all of Max's distribution is deemed to come from DNI. However, only $4,000 of Maureen's $5,000 distribution will carry out DNI.

The allocation of DNI between the first and second tier will have an impact if individual beneficiaries' ratios of Tier I and Tier II distributions differ. We will discuss this matter in more detail in the following chapter.

Study Question 14

Which of the following does not accurately describe a trust or estate's income distribution deduction?

AFor a simple trust, actual distribution of trust income before the year-end is not required to claim an income distribution deduction.
BDistributable net income, reduced by net tax-exempt income, serves as the maximum amount a trust may claim as an income distribution deduction.
CThe same income distribution deduction rules apply in the case of both an estate and a complex trust.
DDistributable net income for a complex trust is deemed to first attach to distributions other than those for amounts required to be distributed currently out of the income of the trust.

Chapter 6. Taxation of Distributions to Beneficiaries (Schedule K-1)

This chapter discusses the taxation of distributions to beneficiaries, carefully examining the flow through aspects of trusts and estates.


Entire Course (including glossary) (94)

Trusts and estates are unique in that while they are generally considered taxable entities in their own right, they also flow through income net of deductions that is tied to distributions. The income that is flowed to beneficiaries on the Schedule K-1 is tied to distributions received by the beneficiaries, and the trust receives an income distribution deduction related to income that flows out to the beneficiaries.

Thus, the preparation of a Form 1041 most often includes a requirement to determine what distributions have been made (or were required to be made, even if not made) to each beneficiary and the maximum amount of the taxable income that can be deducted as distributions to a beneficiary (otherwise known as distributable net income). Note that new Schedules K-2 and K-3 regarding international transactions are not applicable to Form 1041.

Generally, the trust or estate will receive a deduction for the lesser of:

  • Amounts either required to be distributed or actually distributed to beneficiaries, or

  • The distributable net income of the trust or estate for the tax year.

Just as the deduction for the trust or estate is limited to the distributable net income (DNI) of the trust or estate for the year, the amount that the beneficiaries must report as taxable income is also capped at that amount. [IRC §§651, 652, 661, and 662]

To view this interactivity please view chapter 6, page 3

Interactivity information:

Differences between Trust K-1s and Other Types of K-1s

The preparer of a Form 1041 must be aware of the fact that although the trust/estate income tax return contains a Schedule K-1, the preparation of that form is significantly different from the preparation of a Schedule K-1 for a partnership or S corporation.

  1. Not all of the items of the trust or estate that are recognized during the year will necessarily flow to the beneficiaries. Generally, a beneficiary is taxed on no more than amounts that he or she is required by the trust document to receive (for a simple trust) or the total of amounts he or she is required to receive plus other distributions (for a complex trust).

  2. On the other hand, a partner or S corporation shareholder is taxed on his or her share of the items recognized by the entity, even if the party received no distribution from the entity during the year and had no right to demand such a distribution.

  3. Trusts and estates generally do not pass out items of loss, except for limited circ*mstances involving depreciation or certain deductions remaining unused in the final year of the trust or estate.

  4. The trust starts out as the taxable entity, and it can only escape tax on items if it qualifies for an income distribution deduction.

A proper calculation of distributable net income and determination of distributions to the beneficiaries (either those required by the trust document or those actually made during the year) is crucial for the proper preparation of a Form 1041.

Distributions to Beneficiaries

The amount that is considered distributed to beneficiaries includes amounts required to be distributed annually to the beneficiary, plus other amounts properly paid, credited, or required to be distributed. [IRC §§661(a), 662(a)] However, certain amounts are excluded from that definition.

Amounts Required to Be Distributed to the Beneficiary

The amount of income required to be distributed currently is determined by the terms of the trust document. As was discussed earlier, this amount is treated as distributed whether or not the amount is actually distributed by year-end.

Amounts Otherwise Properly Paid, Credited, or Required to Be Distributed

Most other payments made to the beneficiary will fall into this second category, unless they are found not to be in accordance with the terms of the trust or estate as interpreted by the applicable state law.


Exception for Specific Gifts and Bequests

A key exception to the treatment of distributions arises if the item or amount distributed is a specific bequest. For purposes of computing the amounts distributed to a beneficiary, a gift or specific bequest is not treated as a distribution that carries out income and thus cannot be included in the amount of distributions used to compute the distribution deduction or taken into account in allocating distributed DNI to beneficiaries. [IRC §663(a)]

A gift or specific bequest must provide that the amount of money or the identity of the specific property must be ascertainable:

Entire Course (including glossary) (95)

Under the terms of decedent's will as of the date of his/her death, or

Entire Course (including glossary) (96)

Under the terms of an inter vivos trust instrument (that is, a trust established during the life of the grantor) as of the date of the inception of the trust. [Reg. §1.663(a)-1(b)(1)]

To view this interactivity please view chapter 6, page 6

Interactivity information:

Exclusions

The following items are specifically excluded from treatment as a specific bequest or gift:

  • An amount which can be paid or credited only from the income of an estate or trust, whether from the income for the year of payment or crediting, or from the income accumulated from a prior year;

  • An annuity or periodic gifts of specific property in lieu of or having the effect of an annuity;

  • A residuary estate or the corpus of a trust (that is, what is left over in an estate or trust after all expenses have been paid and all other required distributions made); or

  • A gift or bequest paid in a lump sum or in not more than three installments, if the gift or bequest is required to be paid in more than three installments under the terms of the governing instrument. [Reg. §1.663(b)-1(b)(2)]

Study Question 15

Which of the following payments would be excluded from treatment as a distribution to a beneficiary?

AA payment made from the estate in the form of an annuity to be paid over 6 years
BA fixed amount to be paid to a beneficiary only from the income of the estate
CA single payment of $20,000 provided for in the decedent's will that may be made to the beneficiary either out of principal or income at the discretion of the trustee
DA lump payment of $50,000 that the beneficiary agrees is in full satisfaction of a 10 year annuity he was to be paid

Election to Treat Distributions as Made in Preceding Year

A special election is available for a complex trust or estate that allows the trustee or executor to elect to treat distributions that would qualify under the rules above (that is, not treated as a specific bequest) that are made during the 65 days following the end of the trust or estate's tax year as if they were made during that preceding tax year. [IRC §663(b)]

Note

Because the marginal income tax rate on trusts and estates is imposed at the highest individual rates on relatively low amounts of trust income, the potential income tax savings of having income taxed at the beneficiary's rate is often a key factor in the decision to make an IRC §663(b) election. However, since an actual distribution must be made to make the election, the trustee must also consider other, non-tax factors (including the terms of the trust and the grantor's reason for establishing the trust) in determining whether such a distribution is either allowed or prudent based on these non-tax considerations.


Entire Course (including glossary) (97)

Election to Treat Distributions as Made in Preceding Year (cont'd)

The amount elected cannot exceed:

  • The amount of accounting income of the trust for the year the election is being made, or

  • The amount of the distributable net income (DNI) of the trust for the year of the election, reduced by distributions treated as made under the rules prior to the effect of the election (after reduction for any amounts paid in the year of election that were treated as made in the prior year under an IRC §663(b) election in that prior year). [Reg. §1.663(b)-1(a)(2)]

Example 6.1

The Robert Watson Trust, a calendar year trust, has $1,000 of accounting income and $800 of distributable net income in Year 1. The trust properly pays $550 to Aaron, a beneficiary, on January 15, Year 1, which the trustee elects to treat under IRC §663(b) as paid on December 31 of the prior year (Year 0). The trust also properly pays to Aaron $600 on July 19, Year 1, and $450 on January 17, Year 2. For Year 1, the maximum amount that may be elected under this subdivision to be treated as properly paid or credited on the last day of Year 1 is $400 ($1,000 – $600). The $550 paid on January 15, Year 1, does not reduce the maximum amount to which the election may apply, because that amount is treated as properly paid on December 31 of the preceding year. [Adapted from Reg. §1.663(b)-1(a)(2)]


Election to Treat Distributions as Made in Preceding Year (cont'd)

The amount elected is limited to those amounts actually paid or credited during that 65-day period and which the trustee or executor actually designates in the election as being subject to the election. That is, the executor must specify the payments in question that are to be treated as paid on the last day of the prior year. [Reg. §1.663(b)-1(a)(2)(ii)]

The election is made on a timely filed (including extensions) Form 1041 for the year in question. The box next to Question 6 on page 3 of the Form 1041 in the Other Information section must be checked to indicate the election is being made. The election is irrevocable once it is made. [Reg. §1.663(b)-2(a)(1)]

Example 6.2

The trustee in Example 6.1 decides to make an IRC §663(b) election for the Year 1 return. The trustee checks the box next to Question 6 on page 3 of Form 1041 to indicate the election is being made and files the Form 1041 by the due date of the return for the year in question. The trustee designates that $400 of the $450 paid on January 17, Year 2 is to be treated as paid on December 31, Year 1. The remaining $50 of the January 17, Year 2 payment will be treated as a Year 2 distribution.

Note

Note that because the election is limited to the lesser of the amounts actually distributed during the 65-day period or the remaining undistributed DNI of the trust, a trustee must ensure an actual distribution at least equal to the amount that the trustee plans to elect is made by the 65-day deadline.

To view this interactivity please view chapter 6, page 11

Interactivity information:

Allocation of Distribution Deduction to Beneficiaries

The amounts treated as distributed to a beneficiary from an estate or trust will retain its character when reported to the beneficiaries on their Schedules K-1. [IRC §§652(b) and 662(b)] That character is determined by looking at what items are deemed to make up the trust or estate's distributable net income (DNI).

  • Except for specific allocations of income that are treated as having economic effect aside from their tax effect, distributions deemed to be made up of a pro rata share of each type of income included in DNI.

  • If not all of the DNI is distributed to beneficiaries, then the trust or estate is treated as retaining a pro rata portion of each type of income contained in DNI.

  • As DNI is a net income calculation, the amounts treated as distributed are net of allowable expenses.

Specific Allocations of Trust/Estate Income

If an economic effect test is met, the allocation will respect a trust document that assigns a specific type of income to a beneficiary. But the allocation must have an economic effect independent of any tax effect. [Reg. §1.652(b)-2(b)]

  • A provision allowing the trustee to specify the allocation of income among various beneficiaries would not meet this requirement, since there would only be a tax effect. Similarly, a provision that required the trust to treat the first $10,000 of income payable to a beneficiary as coming from tax-exempt income would also fail this test.

  • However, a provision that provided that one-half of tax-exempt income would be distributed to one beneficiary, with accounting income in excess of that amount distributed to another beneficiary would have an economic effect independent of the tax effect.

Note

Readers who have handled partnership returns will likely note this test is virtually identical to the “economic effect” test used in the regulations under IRC §704(b) as part of testing to determine if special allocations under a partnership agreement must be respected by the IRS. As is true there, if the only “loser” under the arrangement is the government, the allocation will not be respected.


Allocation of Deductions to Types of Income

All deductions that are included in the calculation of DNI must be allocated to the various types of income included in the DNI calculation when determining the classes of net income to be reported on the Schedules K-1 of the beneficiaries. That would include deductions allocated to principal under terms of the trust document.

Subject to the caveat that all expenses included in the calculation of DNI must be allocated in determining net income that is passed out to the beneficiaries, the expenses are to be allocated as follows:

  • All deductible items directly attributable to a specific type of income must be applied against that type of income up to the amount of that type of income. If deductions directly attributable to a specific type of income exceed that type of income, the excess is allocated as part of the other expenses, as described below. [Reg. §1.652(b)-3(a)]

Note

For instance, all rental expenses for a particular property would be used first to offset the rental income from that piece of property. If the allowed expenses exceeded the rental income of the property, the excess would then be treated as a “general” deduction in computing DNI.


Allocation of Deductions to Types of Income (cont'd)

  • Deductions not directly allocable to a specific class of income may be allocated to any item of income included in computing distributable net income, after taking into account an appropriate allocation of expenses to tax-exempt income under IRC §265 if the trust has such income. [Reg. §1.652(b)-3(b)] Expenses specifically cited as examples of expenses not directly allocable to a specific class of income by the regulations include trustee's commissions, the rental of safe deposit boxes, and state income and personal property taxes. [Reg. §1.652(b)-3(c)]

Example 6.3

If the income of a trust is $30,000 (after direct expenses), consisting equally of $10,000 of dividends, tax-exempt interest, and rents, and income commissions amount to $3,000, one-third ($1,000) of such commissions should be allocated to tax-exempt interest, but the balance of $2,000 may be allocated to the rents or dividends in such proportions as the trustee may elect. [Adapted from example in Reg. §1.652(b)-3(b)]

Note

Tax advisors' tax preparation software will generally include a default method of allocating such expenses that will be used when preparing beneficiaries' Schedules K-1. The advisor should review that allocation and consider whether another allocation would be preferable or advisable for the trust since the regulation grants the trustee wide discretion in allocating such expenses. The trustee should make the final decision on how expenses will be allocated after obtaining advice about potential alternatives from the tax advisor.


Special Rules for Allocation of Charitable Contributions

A special rule applies to complex trusts where a charitable contribution is allowed (if a charitable contribution is claimed, a trust will not be a simple trust). Generally, the charitable contribution must be allocated ratably among all items of income included in computing DNI, including tax-exempt income, unless the governing instrument requires otherwise. [Reg. 1.643(a)-5] This allocation must be done before the allocation of any other expenses. [Reg. 1.662(b)-2]


Entire Course (including glossary) (98)

Two-Tier Allocation for Complex Trusts and Estates

Complex trusts and estates are subject to a two-tier system of allocation of income items between beneficiaries. This two-tiered system is required because DNI serves as a limit on the total amount of income required to be reported by beneficiaries.

Tier 1 Mandatory Distributions

Under this system, DNI is deemed to be first directed to beneficiaries who, under the terms of the trust, are to receive a mandatory distribution of trust accounting income for the tax year. If any DNI remains after that first allocation, the remainder is then allocated to beneficiaries receiving other distributions.

  • Amounts deemed to be required to be distributed currently out of income include any amounts that the trust document provides are to be paid in all events, either out of income or principal, to the extent there is income available after accounting for amounts required to be paid to other beneficiaries out of trust income. [Reg. §1.662(a)-2(c)]

  • The first tier allocation will be handled in the same manner as the allocation of DNI to beneficiaries in a simple trust. If the total amount required to be distributed is in excess of DNI (computed without regard to any charitable contribution), the beneficiary's K-1 will simply report the DNI, divided into the proper class of income based on the allocation rules described above.

  • If the mandatory income distribution amount is less than DNI, then the mandatory income distribution amount will be deemed to consist proportionately of the amounts included in DNI and that portion of the DNI items will be reported on the beneficiary's K-1. The remaining DNI will be used in the second tier calculation described later.

  • If there are multiple beneficiaries who are receiving mandatory income distributions, their shares of the first tier distribution is determined based on the ratio of their mandatory distribution to the total mandatory distributions to all beneficiaries. [Reg. §1.662(a)-2(a)]


Tier 2 Distributions

If no DNI remains at this point, no income will be reported by beneficiaries receiving a distribution of amounts otherwise properly paid, credited, or required to be distributed (a second tier distribution), nor will the fact that a beneficiary who received a first tier distribution also received a second tier distribution cause that beneficiary to be required to report any additional income.

  • However, if there is DNI left after the allocation of DNI to first tier beneficiaries, the remaining DNI serves as a cap on the maximum amount taxable to the second tier beneficiaries. If total second tier distributions are greater than the remaining DNI, then each recipient of a second tier distribution will receive an allocation of the items making up that DNI in proportion to the beneficiary's share of total second tier distributions.

  • If the remaining DNI is greater than the total second tier distribution, then each beneficiary's distribution will be treated as being made up proportionately of the ratio of each class of net income making up DNI. [Reg. §1.662(a)-2(b)]

Example 6.4

Under the terms of the May Hays Trust, $5,000 is to be paid to Crossroads (a charity) out of income each year; $20,000 of income is currently distributable to Andrew; and an annuity of $12,000 is to be paid to Betty out of income or corpus. All expenses are charges against income and capital gains are allocable to corpus. During the taxable year, the trust had income of $30,000 (after the payment of expenses) derived from taxable interest and made the payments to Crossroads charity and distributions to Andrew and Betty as required by the governing instrument.

The amounts treated as distributed currently for Tier 1 purposes total $25,000 ($20,000 to Andrew and $5,000 to Betty). Since the charitable contribution is out of income, the amount of income available for Betty's annuity is only $5,000. The distributable net income of the trust computed under IRC §643(a) without taking into consideration the charitable contributions deduction of $5,000 is $30,000. Since the amounts treated as distributed currently of $25,000 do not exceed the distributable net income (as modified) of $30,000, Andrew is required to include $20,000 in his gross income and Betty is required to include $5,000 in her gross income under the first tier distribution rules.

Example 6.5

The facts are the same as Example 6.4, except that the trust has $10,000 of administration expenses, commissions, etc., chargeable to corpus. The amounts treated as distributed currently for Tier 1 purposes total $25,000 ($20,000 to Andrew and $5,000 to Betty), since trust accounting income remains the same as before. Distributable net income of the trust computed without taking into account the charitable contributions deduction of $5,000 is only $20,000. Since the amounts treated as distributed currently of $25,000 exceed the distributable net income of $20,000, Andrew is required to include $16,000 (20,000/25,000 × $20,000) in his gross income and Betty is required to include $4,000 (5,000/25,000 × $20,000) in her gross income under the first tier distribution rules. Because Andrew and Betty are beneficiaries of amounts of income required to be distributed currently, they do not benefit from the reduction of distributable net income by the charitable contributions deduction. [Adapted from examples in Reg. §1.662(a)-2]


Study Question 16

Which of the following is not specifically excluded from treatment as a specific bequest?

AAn amount which can only be paid from income of a trust or estate accumulated in the current year
BA gift paid out as a lump sum in lieu of a 10 year annuity payment required under terms of the trust
CAn annuity provided to an individual under the trust
DReceipt of all shares of stock in a specific corporation held by the decedent under a specific provision of the will

The amounts reported on various lines of the Schedule K-1 reflect the amounts of each type of net income to be reported by the beneficiary after determining the amount and nature of each item allocated to the beneficiary under the rules described above.

Final Year Deductions


Entire Course (including glossary) (99)

Generally, with the exception of depreciation discussed earlier, a trust or estate does not pass out losses or deductions via 2021 Schedule K-1. (The 2022 Schedule K-1-Draft version of this form is also provided in the Course References for this course, within the Toolbox which can be accessed from the menu bar.)

Special rules allow some, but not all, unused deductions to be passed out of the estate or trust in its final year.

Both the 2021 Sch K-1 Instructions and the 2022 Sch K-1 Instructions draft version are provided in the Course References for this course, which may be accessed within the Toolbox on the menu bar.


Excess Deductions on Termination

A trust or estate that has nonbusiness deductions in excess of its gross income, aside from charitable contributions or the personal exemption, passes such items out to beneficiaries in the final year of the trust or estate. [IRC §642(h)(2)]

To the extent the deductions in question are deductions allowed in computing investment income under IRC §1411, the beneficiary may use the deductions in the beneficiary's computation of his or her investment income for the year. [Reg. §1.1411-4(g)(4)(iii)]

Under Proposed Regulations 113295-18, an excess deduction on termination of an estate or trust is allowed in arriving at adjusted gross income. We discussed previously how expenses that fall under Section 67(e), that are unique to the administration of an estate or trust, remain deductible. The excess deductions are reported to each beneficiary on Schedule K-1, in Box 11 with code A. The IRS has issued instructions to the taxpayer that an excess deduction on termination for IRC section 67(e) expenses is reported as a write-in as an above the line adjustment to income with the description “ED67.”

Non-miscellaneous itemized deductions are reported on Schedule A of Forms 1040, 1040-SR, and 1040-NR, but miscellaneous itemized deductions are not deductible. Remember that IRC Section 67(g) suspends 2% miscellaneous itemized deductions as part of the Tax Cuts and Jobs Act of 2017 for tax years beginning after 2017 and before 2026.

Note

Final regulations were issued in October 2020 under TD 9918. Taxpayers may elect to apply the amendments to tax years beginning after December 31, 2017 and on or before October 19, 2020. Please see the K-1 instructions for additional information.


Capital Loss Carryover

If there is an unused capital loss carryover in the final year of the estate or trust, that amount is passed out to the beneficiaries. [IRC §642(h)(1)]

Except for a corporate beneficiary, the loss retains its nature (short- or long-term) when it passes out of the trust or estate. However, for a corporate beneficiary, the loss is treated as a short-term capital loss carryover. [Reg. §1.642(h)-1(b)]

The first tax year in which the beneficiary may make use of the capital loss carryover is the year in which the trust or estate terminates. [Reg. §1.642(h)-1(b)]

Example 6.6

The Windsor Trust distributes all of its assets to Alan, the sole remainderman, and terminates on December 31, 2022, when it has a capital loss carryover of $10,000 attributable to transactions during the taxable year 2022. Alan, who reports on the calendar year basis, otherwise has ordinary income of $10,000 and capital gains of $4,000 for the taxable year 2022. Alan would offset his capital gains of $4,000 against the capital loss of the trust and, in addition, deduct $3,000 on his return for the taxable year 2022. The balance of the capital loss carryover of $3,000 may be carried over into future years by Alan.

Instructions for Schedule K-1 (Form 1041) explain that a short-term capital loss carryover is passed out to the beneficiary on Schedule K-1, box 11 with a code C. A long-term carryover is passed out to the beneficiary on Schedule K-1, box 11 with code D.


Net Operating Loss Carryover


Entire Course (including glossary) (100)

Similar to excess deductions on termination and unused capital loss carryovers, a net operating loss carryover remaining unused as of the trust or estate's final tax year is available to be passed to the beneficiaries. [IRC §642(h)(1)]

Both the return for the final year of the trust and estate and the return of the beneficiary for the year in which the K-1 passing out the net operating loss is applied count as separate years for purposes of determining the maximum number of years to which a loss a may be carried. That is true even if both returns share the same year-end date (such as a trust that terminated on December 31, with the loss flowing to a K-1 for an individual beneficiary with the same year-end). [Reg. 1.642(h)-1(b)]

A net operating loss is passed out to a beneficiary on Schedule K-1, box 11 with a code E for the regular tax net operating loss and a code F for the alternative tax net operating loss.


Allocation of Final Year Deductions


Entire Course (including glossary) (101)

The expenses are allocated to beneficiaries proportionately based on the burden of each in the loss or deduction. [Reg. §1.642(h)-4]

Example 6.7

Zeke Swanson's will leaves $100,000 to Annette, and the residue of his estate equally to Bernard and Carl. His estate is sufficient to pay only $90,000 to Annette, and nothing to Bernard and Carl. There is an excess of deductions over gross income for the last taxable year of the estate of $5,000, and a capital loss carryover of $15,000, to both of which IRC §642(h) applies. Annette is a beneficiary succeeding to the property of the estate to the extent of $10,000, and since the total of the excess of deductions and the loss carryover is $20,000, Annette is entitled to the benefit of one-half of each item, and the remaining one-half is divided equally between Bernard and Carl. [Based on Reg. §1.642(h)-4 example]

To view this interactivity please view chapter 6, page 25

Interactivity information:

Alternative Minimum Tax Adjustment

A trust or estate is subject to the alternative minimum tax and, like other flow-through entities, also passes out an AMT adjustment to beneficiaries on Schedules K-1.

  • The trust computes distributable net alternative minimum taxable income which is the AMT version of DNI.

  • Fiduciary AMT is computed on Schedule I, Alternative Minimum Tax—Estates and Trusts, Form 1041. If a fiduciary is claiming an income distribution deduction, this is reported on Schedule I, Part II (Form 1041).

  • The difference between the distribution deduction for regular taxable income and the distribution deduction computed on the AMT basis is reported on Schedule K-1, line 12.

Tax Credits

Estates and trusts qualify for a number of tax credits. Except for a limited exception for foreign tax credits allocable to income in respect of a decedent, an estate does not succeed to credit carryovers from the decedent. [IRC §691(b)]

If the specific credit is one that the tax law provides an allocation for, the tax credit is allocated between the fiduciary and the beneficiary based on the income allocated to each. The Schedule K-1 instructions lists the credits that are subject to allocation between the trust and the beneficiary and are reported in Box 13. This list includes:

  • Credit for estimated taxes (code A). Payment of estimated tax to be credited to the beneficiary (section 643(g))

  • Credit for backup withholding (code B). Income tax withheld on wages cannot be distributed to the beneficiary

  • Low-income housing credit (code C)

  • Rehabilitation credit and energy credit (code D)

  • Other qualifying investment credit (code E)

  • Work opportunity credit (code F)

  • Credit for small employer health insurance premiums (code G)

  • Biofuel producer credit (code H)


Tax Credits (cont'd)

  • Credit for increasing research activities (code I)

  • Renewable energy, refined coal and Indian coal production credit (code J)

  • Empowerment zone employment credit (code K)

  • Indian employment credit (code L)

  • Orphan drug credit (code M)

  • Credit for employer-provided child care and facilities (code N)

  • Biodiesel and renewable diesel fuels credit (code O)

  • Credit to holders of tax credit bonds (code P)

  • Credit for employer differential wage payments (code Q)

  • Recapture of credits (code R)

  • Other credits, including the employer retention credit (code Z)


Other Information


Entire Course (including glossary) (102)

Box 14 of Schedule K-1 reports other information to the beneficiary.

  • Tax-exempt interest (code A)

  • Foreign taxes (code B)

  • Net investment income (code E)

  • Gross farm and fishing income (code F)

  • Foreign trading gross receipts (code G)

  • Adjustment for section 1411 net investment income or deductions (code H)

  • Section 199A information (code I)

  • Other information (code Z)

Entire Course (including glossary) (103)

A foreign tax credit may be claimed by the estate to the extent the taxes are not properly allocable to the beneficiary. The beneficiary receives a proportionate share of taxes paid or accrued to the foreign country during the tax year. [IRC §§642(a), 901(b)(5)]

Entire Course (including glossary) (104)

The election not to file Form 1116, Foreign Tax Credit (Individual, Estate, or Trust), in certain cases is only an option for individuals. A trust or estate claiming the foreign tax credit must always file Form 1116.



Chapter 7. Tax Computation

This chapter provides an overview of the tax computation for estates and trusts filing Form 1041. It introduces the basic calculation and then explores tax credits such as the foreign tax credit. It then goes on to discuss how the net investment income tax impacts estates and trusts.


Entire Course (including glossary) (105)

Trusts and estates are generally subjected to income taxes under rules very similar to those that apply to individual taxpayers. However, they do face their own rate brackets.

Fiduciary Income Tax Brackets

The tax rate schedule for fiduciary returns for 2022 is as follows. [Revenue Procedure 2020-45]

If taxable income is: The tax is:
Not over $2,750 10% of taxable income
Over $2,750 but not over $9,850 $275 plus 24% of the excess over $2,750
Over $9,850 but not over $13,450 $1,979 plus 35% of the excess over $9,850
Over $13,450 $3,239 plus 37% of the excess over $13,450

Study Question 17

In Year 1, Brianna's Investment Trust has dividend income of $3,000, interest income of $2,000, capital gain of $1,000, and $25,000 of taxable income relating to a distribution from an individual retirement account (as defined under IRC §408). The trust has no expenses. What is distributable net income?

A$5,000
B$6,000
C$30,000
D$31,000

Entire Course (including glossary) (106)

Like individuals, trusts and estates qualify for a reduced rate for adjusted net capital gains and qualified dividends. For 2022 the maximum rate imposed on long-term capital gains for the trust or estate is set at 20%. For 2022, the 20% maximum rate applies to long-term capital gains for trusts and estates with total taxable income greater than $13,700. The 15% rate applies to long-term capital gains for trusts and estates with total taxable income of at least $2,800 and less than $13,70. For trusts and estates with total taxable income of less than $2,800, the long-term capital gains rate is 0%.

Adjusted capital gain is computed by taking the net capital gain (long-term capital gains in excess of short-term capital losses) and reducing the amount (but not below zero) by capital gain arising from unrecaptured depreciation on IRC §1250 property and by 28% rate capital gains (collectibles).

Qualified dividends are added to this number to produce the amount subject to the above tax rules.

Example 7.1

The Marion Eastlake Trust has taxable income for 2022 of $10,000 after all deductions (including the income distribution deduction), all which arose from taxable interest income. The trust's income tax for 2022 is calculated as follows:

Amount of income over $9,850 but not over $13,450 = $150
Tax = $1,979 plus 1,979.00
35% of the excess over $9,850 = $150 × .35 = 52.50
Total federal income tax $2,031.50

The regular income tax of a trust or estate is reported on line 1a of Schedule G of Form 1041, page 2.


Entire Course (including glossary) (107)

Personal Exemptions

Trusts and estates are allowed a personal exemption that is used in computing the trust or estate's taxable income. The amount of the exemption is:

Entire Course (including glossary) (108)

$600 for a decedent's estate, [IRC §642(b)(1)]

Entire Course (including glossary) (109)

$300 for a trust required to distribute all of its income currently, [IRC §642(b)(2)(B)] and

Entire Course (including glossary) (110)

$100 for a trust not required to distribute all of its income currently. [IRC §642(b)(2)(A)]

These exemptions are not reduced during the trust or estate's initial year. However, they are prorated should the trust or estate change its accounting year for the short year triggered by the change in year-end. [IRC §443(c) and Reg. §1.443-1(a)(2)]


Alternative Minimum Tax (Schedule I, Form 1041)

The alternative minimum tax applies to trusts and estates using adjustments that are the same as for individual taxpayers. The much higher marginal rates under the regular tax computation make the application of the alternative minimum tax to the trust or estate less likely than for individuals.

The same is not true for the beneficiaries as the amount of alternative minimum tax adjustment flowing out of the trust/estate to the beneficiaries' Schedules K-1 will directly affect the alternative minimum taxable income of the beneficiary.

Computation of Alternative Minimum Taxable Income

The trust or estate computes the alternative minimum tax on Schedule I, Form 1041. Part I is a computation of the estate or trust's share of alternative minimum taxable income.

The tax is imposed by IRC §55. The tax is the excess of the tentative minimum tax over the regular tax computed for the trust or estate. [IRC §55(a)]

Lines 1-24 compute the overall adjusted alternative minimum taxable income to be divided between the trust/estate and its beneficiaries. A tax advisor should note that this portion of the form is very similar to what is found on Form 6251 for computing alternative minimum taxable income for an individual.


Computation of Alternative Minimum Taxable Income (cont'd)

As with an individual, the calculation begins with taxable income. The items of adjustment and preference applied to the taxable income on Schedule I of Form 1041 are, in order:

  • Interest

  • Taxes

  • Add back miscellaneous deductions subject to the 2% floor that exceeded that floor (suspended for tax years 2018-2025)

  • Subtract any refund of taxes included in taxable income

  • Adjust for the difference in depletion between the regular tax and the AMT

  • Add back regular tax net operating loss

  • Add interest received on specified private activity bonds

  • Adjustment for exclusion of qualified small business stock

  • Difference between the fair value and strike price at date of exercise of incentive stock options

  • Adjust for differences in the basis of property sold for regular tax and alternative minimum tax (subject to limits on deduction of capital losses in excess of gains)


Computation of Alternative Minimum Taxable Income (cont'd)

  • Adjust for difference in regular tax and AMT depreciation

  • Adjust for difference in passive activity income/loss between the regular tax and AMT

  • Adjust for basis and at-risk loss limitation differences between regular tax and AMT

  • Adjust for difference in circulation costs between regular tax and AMT

  • Adjust for differences in long-term contract income/loss between regular tax and AMT

  • Adjust for differences in mining cost deductions between regular tax and AMT

  • Adjust for difference in research and experimental costs between regular tax and AMT

  • Adjust for differences in pre-1987 installment obligation income between regular tax and AMT

  • Adjust for intangible drilling cost preference

The final line allows for adjustments for any other differences that need to be taken into account between the regular taxable income and alternative minimum taxable income.

The adjusted alternative minimum taxable income is then adjusted for the income distribution deduction on an alternative minimum tax basis computed in Part II and the estate tax deduction claimed on IRD.

Distributable Net Alternative Minimum Taxable Income is computed in Part II using the AMT adjustments as applied to the DNI calculation.


Alternative Minimum Tax Rates for Trusts and Estates

The tax rates for the alternative minimum tax for trusts and estates are the same as for individuals. These rates are:

  • 26% of net alternative minimum taxable income (AMTI) (reduced by the exemption described below) up to $206,100(for 2022), and

  • 28% of the net alternative minimum taxable income in excess of that amount. [IRC §55(b)(1)(A)]

The maximum tax rates on capital gains apply to this calculation, holding the maximum rate on net capital gains to no more than 20%.

The AMT exemption for trusts and estates is set at $26,500 for 2022. Revenue Procedure 2021-45. The exemption is reduced (but not below zero) by 25% of AMTI in excess of $86,650 for 2022. It is completely phased out when AMTI reaches $192,650 (for 2022).

This computed amount is the tentative minimum tax. That tax is compared with the tax computed using the standard rules. If the tentative minimum tax exceeds the regular tax, the difference is carried to Line 1c of Schedule G of Form 1041.

The tax is computed on 2021 Schedule I of Form 1041. (The 2022 Schedule I-Draft of this form is included in the Course References for this course which can be accessed from the Toolbox on the menu bar.) Both the 2021 Sch I Instructions and 2022 Sch I Instructios draft version are included in the Course References for this course, which may be accessed within the Toolbox on the menu bar.


Study Question 18

In 2022, a trust is subject to a maximum regular tax rate of _____ on ordinary income over _____.

A20%; $441,450
B15%; $2,650
C24%; $2,600
D37; $13,450

Study Question 19

Which of the following is not an item of adjustment or preference for a trust or estate in computing alternative taxable income?

ACharitable contributions
BNet operating loss
CInterest received on specified private activity bonds
DRefund of taxes included in taxable income

General

As with other taxpayers, trusts and estates can qualify for various tax credits. Some of these credits are required to be shared with beneficiaries who receive a distribution of income. That list is found in Chapter 6 on taxation of distributions to beneficiaries.


Entire Course (including glossary) (111)

Foreign Tax Credit

Since a large proportion of trusts hold investment assets, the foreign tax credit is the credit most practitioners will see the most often when dealing with the preparation of trust or estate income tax returns.

The same basic rules that apply to an individual claiming the foreign tax credit will apply to a trust or estate, with the additional caveat that the trust or estate may claim the credit only to the extent that the credit is not properly allocable to beneficiaries. [IRC §642(a)]

Unfortunately, there is not a large amount of guidance regarding what that properly allocable amount may be, as IRC §901(b)(5) merely says the beneficiary will receive his “allocable share.” Given that lack of specific guidance, under the standard distribution rules of IRC §§661 and 662, assuming that income distributions came from foreign and domestic sources on a pro rata basis would seem appropriate.

Example 7.2

The Elizabeth Brixton Trust's DNI of $100,000 for the year consists of $50,000 of dividends from domestic sources and $50,000 of dividends from foreign sources. The trust also has paid $10,000 in foreign taxes related to those dividends. The trust has claimed an income distribution deduction to its beneficiaries of $50,000. Using the principles of IRC §§661 and 662, the trust assumes that $25,000 of the foreign income is deemed distributed, thus one-half of the foreign taxes (or $5,000) is properly allocable to the beneficiaries. The trust, therefore, is eligible to use $5,000 of taxes and $25,000 of foreign income in computing its foreign tax credit.

Like individuals, the trust or estate can elect to claim a tax deduction for foreign taxes in lieu of the credit. The trust's decision to claim a credit or deduction can be changed at any time before the period to apply for a refund of taxes expires, but no deduction is available for any year the credit is elected. [IRC §901, Reg. §1.901-1(c)]


General Business Credit

The various items that make up the general business credit will usually be available to a trust or estate. [IRC §38] However, like the foreign tax credit, the general business credit must be allocated between the trust/estate and the beneficiaries.

The carryover and carryback rules applicable to general business credits apply to trusts and estates. Thus, an unused general business credit is carried back one year and forward 20 years. [IRC §39(a)(1)]

If a credit is one for which a deduction is allowed if the credit goes unused (normally due to the expiration of the 20-year carryforward period) under IRC §196, the trust or estate is allowed to claim a deduction on its final return to the extent the credit goes unused. [IRC §196(b)]

Some of the credits for which an IRC §196 last deduction is available include the work opportunity credit under IRC §51(a), the alcohol fuels credit under IRC §40(a), and the incremental research credit under IRC §41(a).

Note

Note that general business credit carryovers unused at the trust or estate level do not flow out to the beneficiaries in the final year of the trust's return. So general business credits for which §196 does not apply (such as the low-income housing credit) are simply lost if the trust or estate is unable to use the credit before the trust or estate terminates.


Prior Year Minimum Tax Credit

Like individuals, trusts qualify for a minimum tax credit. [IRC §53]

Deferral Preferences and Adjustments

As with individuals, the credit only applies to deferral preferences and adjustments. A deferral item is one where the amount of income or deduction is ultimately the same for the regular tax and the alternative minimum tax, but the timing of the recognition is different.

Example 7.3

The Ira Roderick Trust places in service a piece of equipment during the year. The regular tax depreciation on the equipment is $5,000, while the alternative minimum tax depreciation is $3,000. The $2,000 difference in depreciation is a timing difference only. Should the trust hold the equipment until it is fully depreciated, the total depreciation claimed over the life of the equipment will be the same. Similarly, if the property is sold before it is fully depreciated, the trust will be able to claim the difference in depreciation as a difference in basis when computing the regular tax and the alternative minimum tax. Thus, the preference is a deferral item and can give rise to a minimum tax credit.


Exclusion Preferences and Adjustments

Preferences that are not deferral preferences are referred to as exclusion preferences and adjustments. Exclusion preferences and adjustments shown on the trust return include:

  • Interest,

  • Taxes,

  • Miscellaneous itemized deductions subject to the 2% exclusion, (suspended 2018 – 2025)

  • Income tax refunds,

  • Depletion,

  • Private activity bond interest,

  • IRC §1202 exclusion for gain on qualified small business stock, and

  • The personal exemption of the trust or estate.


Calculation

The minimum tax credit is calculated by comparing the minimum tax paid (that is, tentative minimum tax in excess of the regular tax for the year) with the minimum tax that would be paid if only the trust's share of the exclusion preferences and adjustments had been included in the calculation of minimum tax.

If a minimum tax credit is generated for a year, it is carried forward to future years. In future years, it may reduce the regular tax, but only to the extent the regular tax exceeds the tentative minimum tax calculated for the year. [IRC §53]

Example 7.4

The Christina Pink Trust had exclusion preferences of $5,000 and deferral preferences of $4,000 for the year. During the year, the trust computed a minimum tax liability of $1,000. In determining whether there is a minimum tax credit available for carryover, the trust recomputes Schedule I using only the $5,000 of deferral preferences. On that “as if” Schedule I, the trust shows no net minimum tax due (tentative minimum tax is now less than the regular tax). The minimum tax credit carried forward to the next year is $1,000 ($1,000 shown on return – $0 minimum tax due when only exclusion preferences are considered).

The following year the trust does not compute any minimum tax being due. However, the trust has a regular tax liability of $12,000. The trust's tentative minimum tax for the year is $11,250. The trust is allowed to claim as a minimum tax credit the lesser of the credit carried forward into the year ($1,000) or the difference between the regular tax and the tentative minimum tax ($12,000 – $11,250, or $750). Thus the trust will claim a credit of $750 on the return and carry the remaining $250 unused credit to the following year.

The minimum tax credit is claimed on Form 8801, Credit for Prior Year Minimum Tax—Individuals, Estates, and Trusts. If a credit is allowed, it is carried forward to Schedule G, Form 1041, line 2c.


General


Entire Course (including glossary) (112)

Like individuals, trusts and estates are subject to the 3.8% tax on net investment income, but the tax applies in a modified form.

As is true with the individual income tax, the net investment income tax is applied against the lesser of two amounts computed on Form 8960, Net Investment Income Tax—Individuals, Estates, and Trusts. Those amounts are:

  • Undistributed net investment income.

  • Adjusted gross income in excess of the amount at which the maximum marginal tax rate applies to taxable income of a trust or estate ($13,450 for 2022).

To view this interactivity please view chapter 7, page 20

Interactivity information:

Net Investment Income

The definition of what is included in net investment income is the same for trusts and estates as it is for individuals.

Generally, investment income consists of the following:

  • Gross income from interest, dividends, annuities, royalties, and rents, other than such income which is derived in the ordinary course of a trade or business (although see the next bullet point),

  • Income from a trade or business if that trade or business is a passive activity under IRC §469 or a trade or business of trading in financial instruments or commodities (as defined in IRC §475(e)(2)) [IRC §1411(c)(2)],

  • Net gain from the disposition of property other than from trade or businesses not described above (with a special look-through rule for interests in partnerships and S corporations found at IRC §1411(c)(4)), and

  • Income on investment of working capital, determined under rules similar to that found at IRC §469(e)(1)(B). [IRC §1411(c)(3)]

To arrive at net investment income, that amount is reduced by expenses properly allocable to investment income. [IRC §1411(c)]

Exempt Income

Some items are exempt from inclusion in net investment income. For instance, some trusts receive distributions from retirement plans that are listed in IRC §1411(c)(5). Such distributions are exempt from treatment as investment income, even if the entire distribution is derived from what would otherwise be investment income (interest, dividends, gains on sale, etc.). Such plans include:

  • IRC §401(a) qualified plans, such as profit-sharing plans, defined benefit pension plans, §401(k) plans, ESOPs, etc.,

  • IRC §403(b) plans,

  • Individual retirement accounts (IRC §408), including distributions from IRAs that received simplified employee pension plans contributions (sometimes called SEP-IRA accounts) or SIMPLE-IRA accounts,

  • Roth IRAs (IRC §408A), and

  • IRC §457(b) governmental plans.

Note

Thus, a trust designated as the beneficiary of such an account that has no other source of taxable income will not be subject to the IRC §1411 net investment income tax. Note, though, that nonqualified deferred compensation is not listed an item exempt from inclusion in the calculation of the net investment income tax.


Undistributed Net Investment Income

One key difference from the individual tax is that an additional deduction is allowed in computing the investment income component of the tax under §1411. For a trust or estate the tax is imposed on the undistributed net investment income.

Entire Course (including glossary) (113)

Allowing investment income to be reduced at the trust or estate level for amounts distributed to the beneficiaries eliminates double-counting the items of income that will be reported on the Schedules K-1 for purposes of computing net investment income at both the trust and beneficiary level. The beneficiary will include items flowing out to him/her on the K-1 that are included in the definition of investment income when computing his/her net investment income for the tax year.


The investment income is reduced by only a portion of the income distribution deduction—the amounts of the distribution treated as coming from non-investment income (under IRC §1411's definition of that term). [Reg. §1.1411-3(e)]

The computation involves computing the ratio of the trust's investment income under §1411 to the total of such investment income and excluded income which consists of taxable items not treated as investment income plus nontaxable items. The calculation is similar to the computation of the portion of an income distribution allocable to tax-exempt income under Reg. §1.661(b)(1).

Example 7.5

Calculation of undistributed net investment income (with no charitable contribution deduction).

In Year 1, Big Investment Trust has dividend income of $15,000, interest income of $10,000, capital gain of $5,000, and $75,000 of taxable income relating to a distribution from an individual retirement account (as defined under IRC §408). Trust has no expenses. Trust distributes $10,000 of its current year trust accounting income to Edgar, a beneficiary of Trust.

Big Investment Trust's distributable net income is $100,000 ($15,000 in dividends plus $10,000 in interest plus $75,000 of taxable income from an individual retirement account), from which the $10,000 distribution to Edgar is paid.

Big Investment Trust's deduction under IRC §661 is $10,000. Under Reg. §1.662(b)-1, the deduction reduces each class of income comprising distributable net income on a proportional basis. The $10,000 distribution equals 10% of distributable net income ($10,000 ÷ $100,000). Therefore, the distribution consists of dividend income of $1,500, interest income of $1,000, and ordinary income attributable to the individual retirement account of $7,500. Because the $5,000 of capital gain allocated to principal for trust accounting purposes did not enter into distributable net income, no portion of that amount is included in the $10,000 distribution, and it does not qualify for the deduction under IRC §661.

Big Investment Trust's net investment income is $30,000 ($15,000 in dividends plus $10,000 in interest plus $5,000 in capital gain). Trust's $75,000 of taxable income attributable to the individual retirement account is excluded income under Reg. §1.1411-1(d)(4). Big Investment Trust's undistributed net investment income under paragraph (e)(2) of this section is $27,500, which is Big Investment Trust's net investment income ($30,000) less the amount of dividend income ($1,500) and interest income ($1,000) distributed to Edgar. The $27,500 of undistributed net investment income is comprised of the capital gain allocated to principal ($5,000), the remaining undistributed dividend income ($13,500), and the remaining undistributed interest income ($9,000).

Edgar's net investment income includes dividend income of $1,500 and interest income of $1,000, but does not include the $7,500 of ordinary income attributable to the individual retirement account because it is excluded from net investment income under Reg. §1.1411-8. [Based on IRS examples in the §1411 Regulations]

Example 7.6

Calculation of undistributed net investment income with a charitable contribution deduction.

The facts are the same as Example 7.5, except Big Investment Trust is required to distribute $30,000 to Edgar. In addition, Big Investment Trust has a $10,000 deduction under IRC §642(c) (deduction for amounts paid for a charitable purpose). Big Investment Trust also makes an additional discretionary distribution of $20,000 to Shannon, a beneficiary of Trust. As in Example 7.5, Big Investment Trust's net investment income is $30,000 ($15,000 in dividends plus $10,000 in interest plus $5,000 in capital gain). In accordance with Reg. §§1.661(b)-2 and 1.662(b)-2, the items of income must be allocated between the mandatory distribution to Edgar, the discretionary distribution to Shannon, and the $10,000 distribution to a charity.

For purposes of the mandatory distribution to Edgar, Big Investment Trust's distributable net income is $100,000. See §1.662(b)-2, Example 1(b). Trust's deduction under IRC §661 for the distribution to Edgar is $30,000. Under Reg. §1.662(b)-1, the deduction reduces each class of income comprising distributable net income on a proportional basis. The $30,000 distribution equals 30% of distributable net income ($30,000 ÷ $100,000). Therefore, the distribution consists of dividend income of $4,500, interest income of $3,000, and ordinary income attributable to the individual retirement account of $22,500. Edgar's mandatory distribution thus consists of $7,500 of net investment income and $22,500 of excluded income.

Big Investment Trust's remaining distributable net income is $70,000. Big Investment Trust's remaining undistributed net investment income is $22,500. The $10,000 charitable contribution deduction is allocated in the same manner as the distribution to Edgar, where the $10,000 distribution equals 10% of distributable net income ($10,000 ÷ $100,000). For purposes of determining undistributed net investment income, Trust's net investment income is reduced by $2,500 (dividend income of $1,500, interest income of $1,000, but with no reduction for amounts attributable to the individual retirement account of $7,500).

With respect to the discretionary distribution to Shannon, Big Investment Trust's remaining distributable net income is $60,000. Trust's remaining undistributed net investment income is $20,000. Trust's deduction under section 661 for the distribution to B is $20,000. The $20,000 distribution equals 20% of distributable net income ($20,000 divided by $100,000). Therefore, the distribution consists of dividend income of $3,000, interest income of $2,000, and ordinary income attributable to the individual retirement account of $15,000. Shannon's distribution consists of $5,000 of net investment income and $15,000 of excluded income.

Big Investment Trust's undistributed net investment income is $15,000 after taking into account distribution deductions and the charitable contribution. To arrive at Trust's undistributed net investment income of $15,000, Trust's net investment income of $30,000 is reduced by $7,500 of the mandatory distribution to Edgar, $2,500 of the charitable deduction, and $5,000 of the discretionary distribution to Shannon. The undistributed net investment income consists of the remaining dividend income of $6,000 ($15,000 less $4,500 less $1,500 less $3,000), interest income of $4,000 ($10,000 less $1,000 less $3,000 less $2,000), and the $5,000 of undistributed capital gain.

Entire Course (including glossary) (114)

Generally, capital gains are by default treated as allocable to corpus under state principal and income acts and do not enter into the calculation of distributable net income under the DNI rules. As such, they won't be considered to be reduced by the trust's distributions, regardless of the size of those distributions.

However, as was discussed in the chapter on the computation of distributable net income, capital gains can be included in the computation of DNI in certain circ*mstances, including those where the trust document provides that such gains are to be treated as allocable to income. If capital gains are allocable to income, then they do enter into the computation of distributed net investment income.

As was also discussed in the chapter on the calculation of distributable net income, whether a receipt is allocated to principal or income for trust accounting purposes has no impact on DNI, aside from the treatment of capital gains noted above.

Example 7.7

The Yancey Trust received $20,000 in IRA distributions, $10,000 of dividends, and $10,000 of interest income. Per the terms of the trust, all income is to be distributed currently to Mary. Under the applicable principal and income act, 10% of the IRA distribution is allocated to income and 90% to principal.

The trust has accounting income for the year of $22,000, consisting of $10,000 of dividend income, $10,000 of interest income, and $2,000 of the IRA distribution. The trustee, as required, distributed $22,000 to Mary.

The trust's distributable net income is $40,000 ($20,000 from the IRA, $10,000 of dividends, and $10,000 of interest income). Thus, 50% of DNI is deemed to represent IRA distribution income, 25% interest income, and 25% dividend income.

The IRC §1411 income distribution deduction is thus determined to be equal to $11,000, or one-half of the total distribution to Mary ($22,000). That is true despite the fact that Mary's distribution was computed based on all of the interest income and dividend income.

The trust will have $9,000 of undistributed net investment income. The investment income of $20,000 ($10,000 of dividends and $10,000 of interest) is reduced by the $11,000 IRC §1411 investment income distribution deduction, leaving $9,000 of undistributed net investment income.

On the other hand, Mary is treated as only receiving $11,000 of investment income from the trust, based on the makeup of the income flowing onto her Schedule K-1 from the trust.


Study Question 20

Distributions from which of the following is not exempt from inclusion in net investment income for the net investment income tax?

AIRC §401(a) qualified plans
BIRC §457(b) governmental plans
CNonqualified deferred compensation plans
DRoth IRAs

Study Question 21

In Year 1, Brianna's Investment Trust has dividend income of $3,000, interest income of $2,000, capital gain of $1,000, and $25,000 of taxable income relating to a distribution from an individual retirement account (as defined under IRC §408). The trust has no expenses. In Year 1, Brianna's Investment Trust distributes $3,000 to her. What is the character of Brianna's distribution?

A$3,000 in dividends
B$1,000 in dividends, $1,000 in interest, plus $1,000 capital gain
C$300 in dividends, $200 in interest, plus $2,500 taxable income from an IRA
D$3,000 in taxable income from an IRA

Adjusted Gross Income Limitation

One of the key issues that has traditionally faced trusts and estates is the fact that their undistributed income is subject to tax under a compressed progressive rate schedule, placing them at a rate disadvantage when compared to other tax entities.

The new §1411 tax adds another disadvantage to income taxed inside a trust, making the “protected” income level where investment income will not be subject to tax the level at which the maximum tax rates under IRC §1(e) apply to the trust or estate.

For 2022, the 3.8% surtax (net investment tax) begins to apply at $13,450 for trusts and estates, which is well below the protected level for married individuals filing separate returns.

Since, in most cases, a trust or estate's income will consist entirely of income that will be deemed investment income for purposes of this tax, this low trigger means that trusts designed to accumulate income or which have significant amounts of taxable items that are allocable to corpus will face a significantly higher tax burden.

As was discussed in Chapter 3, a trust's or estate's adjusted gross income is computed under the provisions found in IRC §67(e).


Adjusted Gross Income Limitation (cont'd)

For purposes of this section, the adjusted gross income of an estate or trust shall be computed in the same manner as in the case of an individual, except that—

  1. the deductions for costs which are paid or incurred in connection with the administration of the estate or trust and which would not have been incurred if the property were not held in such trust or estate, and

  2. the deductions allowable under sections 642(b), 651, and 661, shall be treated as allowable in arriving at adjusted gross income. Under regulations, appropriate adjustments shall be made in the application of part I of subchapter J of this chapter to take into account the provisions of this section.

The finalization of the Knight regulations at Reg. §1.67-4 will cause adjusted gross income to rise on many trusts with institutional trustees, as a portion of a bundled trustee/investment advisory fee will no longer be deductible in computing adjusted gross income.


Adjusted Gross Income Limitation (cont'd)

Notice 2018-61 clarified the deductibility of expenses under IRC Section 67(e). Proposed Regulations 113295-18 clarifies that the following deductions allowed to estates and non-grantor trusts are not miscellaneous itemized deductions:

  • Costs paid or incurred in connection with the administration of an estate or non-grantor trust that would not have been incurred if the property were not held in the estate or trust,

  • the personal exemption of an estate or non-grantor trust,

  • the distribution deduction for trusts distributing current income, and

  • the distribution deduction for estates and trusts accumulating income.

Estates, non-grantor trusts, and their beneficiaries may rely on these proposed regulations under section 67 and 642 (h) for taxable years beginning after December 31, 2017, and on or before the date these regulations are published as final regulations in the Federal Register.

Final regulations were issued on October 19, 2020 (TD 9918).

To view this interactivity please view chapter 7, page 32

Interactivity information:

Special Rule for Bankruptcy Estates

A bankruptcy estate is treated as a married taxpayer filing a separate return for purposes of applying the net investment income tax of IRC §1411. [Reg. §1.1411-2(a)(2)(iii)] The good news is that this means a bankruptcy estate will have a far higher adjusted gross income threshold ($125,000) before triggering the §1411 net investment income tax than will a standard trust or estate. But the threshold level will be materially lower than that for a single individual ($200,000).

Exempted Trusts and Estates

The statute contains one statutory exemption for a category of trusts, and the regulations provide a list of other trusts that are not to be considered subject to the tax.

Purely Charitable Trusts

IRC §1411(e)(2) exempts from tax trusts for which all unexpired interests are devoted to charitable purposes as described in IRC §170(c)(2)(B) or, as §170 phrases it, “organized and operated exclusively for religious, charitable, scientific, literary, or educational purposes, or to foster national or international amateur sports competition (but only if no part of its activities involve the provision of athletic facilities or equipment), or for the prevention of cruelty to children or animals.”

Although this is the only exemption that Congress explicitly mentioned, the IRS in the proposed regulations felt the need to clarify that the tax provision also would not apply to certain other trusts.

Charitable Purpose Estates

An estate in which all unexpired interests are devoted to one or more charitable purposes described in IRC §170(c)(2)(B) is exempted from the tax. They were added to the final regulations to be consistent with the exclusion of similar Charitable Purpose Trusts. [Reg. §1.1411 3(b)(1)]


Trusts Exempt from Tax

Trusts exempted from tax under IRC §501(a) are also not subject to the §1411 tax. [Reg. §1411-3(b)(2)]

Such trusts include:

  • Qualified retirement plan trusts, and

  • Charitable organizations that are themselves trusts.

The exclusion applies to all income of such a trust and, per the preamble, would include amounts subject to the IRC §511 unrelated business income tax (including items subject to that tax that would otherwise be investment income).

Charitable Remainder Trusts

A charitable remainder trust is not subject to the tax under §1411. [Reg. §1411-3(b)(3)]. That is true even if all of the income of the charitable remainder trust consists of interest, dividends, and capital gains. [Proposed Reg. §1411-3(b)]

However, that does not mean that income flowing out of a charitable remainder trust to income beneficiaries is not part of the beneficiary's net investment income.


Trusts Exempt from Tax by Statute

Any trust exempted from tax by statute is exempt from the net investment income tax of §1411. [Reg. §1411-3(b)(4)] The regulation gives specific examples of such trusts, listing:

  • Archer Medical Savings Account (MSA) trusts exempt under IRC §220(e)

  • Health Savings Account (HSA) trusts exempt under IRC §223(e)(1)

  • Qualified tuition program trusts (“529 Plans”) exempt under IRC §529(a)

  • Coverdell education savings account trusts exempt under IRC §530(a) (ESAs or, if you remember the original term, Education IRAs)

The regulation does not limit its application to the listed trusts. Thus, if Congress creates additional trust entities that are exempted from income tax, they would also be exempted from the IRC §1411 investment tax.

Grantor Trusts

Any trust or portion of a trust treated as a grantor trust under the rules found in IRC §§671-679 is also not subject itself to the investment income tax. However, the income in such a trust or portion of a trust is treated as earned directly by the grantor and to the extent such income is investment income will be included in computing the grantor's net investment income. [Reg. §1411-3(b)(5)]


Foreign Estates

Reg. §1.1411-3(b)(1)(ix) provides that the net investment income tax does not apply to foreign estates. However, the preamble to the regulations notes that U.S. beneficiaries are potentially subject to the §1411 tax related to distributions from such estates.

Foreign Trusts

The IRS continues to study how to deal with distributions of accumulations of income from foreign trusts to U.S. beneficiaries. The IRS clearly states in the final regulations that such trusts are not subject to the §1411 tax, but restates the agency's belief that a U.S. beneficiary should be subject in some form to the §1411 tax if he or she receives an accumulation distribution from such a trust.

However, for now the IRS hasn't figured out how to handle that matter, so the IRS has “reserved” Reg. §1.141-4 (e)(1)(ii) to deal with the issue in future. The preamble provides that, until such guidance is issued, the §1411 tax will not apply to such accumulation distributions from foreign trusts to U.S. beneficiaries.


Common Trust Funds

While not identified in the proposed regulations, the preamble points out that common trust funds described in IRC §584, being expressly exempt from taxation under Chapter 1 of the IRC, are not subject to this tax.

Designated Settlement Funds

Designated settlement funds that are taxable under IRC §468B in lieu of taxation under any other provision of subtitle A (which includes §1411) pursuant to §468B(b)(4) are also noted in the preamble as being exempt from the investment income tax of §1411.

Cemetery Perpetual Care Funds

Cemetery perpetual care funds as defined in IRC §642(i) are excluded from the application of the net investment income tax. [Reg. §1.1411 1(b)(1)] The IRS determined that such trusts, benefitting an operating company, are similar to business trusts.

Electing Alaska Native Settlement Trusts

Alaska Native Settlement Trusts that have the election under IRC §646 are not subject to the tax on net investment income under IRC §1411. [Reg. §1.1411 3(b)(1)]


Trusts Not Exempted


Entire Course (including glossary) (115)

In issuing the final regulations, the IRS made the determination that the following trusts would be subject to the net investment income tax. The IRS had asked for comments regarding whether these trusts should be exempted, but decided they should be subject to the tax.

Entire Course (including glossary) (116)

Pooled Income Funds [IRC §642(c)(5)]

Entire Course (including glossary) (117)

Qualified Funeral Trusts – for purposes of the calculation of any tax imposed by section 1411, section 1411 and the regulations thereunder are applied to each qualified funeral trust (within the meaning of section 685) by treating each beneficiary's interest in each such trust as a separate trust.


Computing the Tax

The trust computes the tax under IRC §1411 on Form 8960, the same form used by individuals to compute the tax.

  1. Part I of Form 8960 is used to compute the investment income of the trust or estate.

  2. Part II reports the investment expenses of the estate or trust.

  3. These deductions do not include the investment income distribution deduction under §1411. Rather, undistributed net investment income is computed on line 18 of Part III of Form 8960.

    • On line 18a the trust or estate reports net investment income (that is, investment income computed in Part I, reduced by investment expenses computed in Part II).

    • On line 18b the trust reports the total of the §1411 investment income distribution deduction computed above and any charitable deduction allowed to the trust or estate pursuant to IRC §642(c). The difference between line 18a and 18b, which represents undistributed net investment income, is reported on line 18c.

  4. Line 19 computes the excess adjusted gross income of the trust or estate. The trust's adjusted gross income is reported on line 19a, while the level at which the maximum trust rates on taxable income begin to apply is reported on line 19c.

  5. The lesser of the amounts on line 18c or 19c, which will be the base on which the §1411 tax is computed, is reported on line 20, with that number multiplied by 3.8% and that product reported on line 21. Line 21 is the tax on net investment income owed by the trust.

The tax is carried to line 5 of Schedule G of Form 1041.


Net Investment Income Tax and Electing Small Business Trusts


Entire Course (including glossary) (118)

Another special type of trust that has to be dealt with is the electing small business trusts (ESBTs). An ESBT is a trust that holds S corporation stock and which has elected under IRC §1361(e)(3) to be treated as such.

In such a case, the trust pretends to split off the portion of the trust holding the S shares into a separate trust and pay tax on the income of that trust separate and apart from any other trust income or expense.

The IRS does not, in the Regulations, allow this two trust fiction to create two separate AGIs and two separate base levels of AGI not subject to tax. Rather, the trust is reassembled to compute the tax. [Reg. §1.1411-3(c)]


Net Investment Income Tax and Electing Small Business Trusts (cont'd)


Entire Course (including glossary) (119)

Reg. §1.1411-3(c) provides the following three-step calculation:

  • Step 1 – Both fictional trusts (the S portion and non-portion of the trust) separately compute their undistributed net investment income using the standard trust rules for the computation of such items. These two numbers are combined to yield the single trust undistributed net investment income.

  • Step 2 – The adjusted gross income for the single trust is computed by taking into account the adjusted gross income of the non-S portion (computed using the standard mechanisms for a trust) and then combining that with the pass-through income/loss of the S portion of the trust.

  • Step 3 – The overall trust computes its liability for the §1411 tax by paying the lesser of:

    • The net undistributed investment income computed in Step 1, or

    • The amount by which the combined adjusted gross income in Step 2 exceeds the base level of trust AGI to which the tax does not apply (the dollar amount at which a trust begins to pay tax at the highest bracket for the year in question).

The IRS gives a comprehensive example of the computation of the tax for an electing small business trust in Example 3 at Reg. §1.1411-3(f).

Example 7.8

Calculation of an ESBT's tax for purposes of IRC §1411.

In Year 1, the non-S portion of Trust, an ESBT, has dividend income of $15,000, interest income of $10,000, and capital gain of $5,000. Trust's S portion has net rental income of $21,000 and a capital loss of $7,000. The Trustee's annual fee of $1,000 is allocated 60 percent to the non-S portion and 40 percent to the S portion. Trust makes a distribution from income to a single beneficiary of $9,000.


Entire Course (including glossary) (120)

Example 7.8 (cont'd)

Step One

Trust must compute the undistributed net investment income for the S portion and non-S portion in the manner described in paragraph (c)(1) of this section.

The undistributed net investment income for the S portion is $20,600 and is determined as follows:

Net Rental Income $ 21,000
Trustee Annual Fee (400)

Entire Course (including glossary) (121)

Total S Portion Undistributed Net Investment Income $ 20,600

Entire Course (including glossary) (122)

No portion of the capital loss is allowed because, pursuant to Reg. §1.1411-4(d)(2), net gain cannot be less than zero and excess capital losses are not properly allocable deductions under §1.1411-4(f). See Example 1 of §1.1411-4(h). In addition, pursuant to Reg. §1.641(c)-1(i), no portion of the $9,000 distribution is allocable to the S portion.

Step One (cont'd)

The undistributed net investment income for the non-S portion is $20,400 and is determined as follows:

Dividend Income $ 15,000
Interest Income 10,000
Capital Gain 5,000
Trustee Annual Fee (600)
Distributable Net Investment Income Distribution (9,000)

Entire Course (including glossary) (123)

Total S Portion Undistributed Net Investment Income $20,400

Entire Course (including glossary) (124)

Trust will combine the undistributed net investment income of the S portion and non-S portion from (ii)(A) and (B) to arrive at Trust's combined undistributed net investment income.

S Portion's Undistributed Net Investment Income $20,600
Non-S Portion's Undistributed Net Investment Income 20,400

Entire Course (including glossary) (125)

Combined Undistributed Net Investment Income $41,000

Entire Course (including glossary) (126)

Step Two

The ESBT will calculate its adjusted gross income. Pursuant to paragraph (c)(1)(ii)(B) of this section, the ESBT's adjusted gross income is the non-S portion's adjusted gross income increased or decreased by the net income or net loss of the S portion. The adjusted gross income for the ESBT is $38,000 and is determined as follows:

Dividend Income $ 15,000
Interest Income 10,000
Capital Gain 5,000
Trustee Annual Fee (600)
Distributable Net Investment Income Distribution (9,000)
S Portion Income (see (iii) (C)) 17,600

Entire Course (including glossary) (127)

Adjusted Gross Income $38,000

Entire Course (including glossary) (128)

The S portion's single item of ordinary income used in the ESBT's adjusted gross income calculation is $17,600. This item of income is determined by starting with net rental income of $21,000 and reducing it (1) by the S portion's $400 share of the annual trustee fee, and (2) as allowed by section 1211(b)(1), $3,000 of the $7,000 capital loss.

Step Three

Trust will pay tax on the lesser of:

  1. The combined undistributed net investment income ($41,000 calculated in (ii)(C)), or

  2. The excess of adjusted gross income ($38,000 calculated in (iii)(B)) over the dollar amount at which the highest tax bracket in section 1(e) applicable to a trust begins for the taxable year.


Entire Course (including glossary) (129)

General

A trust or estate that pays household employees is subject to the household employment taxes under the same rules as an individual. The tax due is computed on Schedule H, Form 1040, and reported on line 7 of Schedule G, Form 1041.

Liability for Tax

The trust or estate is liable for the household employment taxes if it:

  • Paid any single household employee cash wages of $2,400 or more in 2022.

  • Withheld any federal income taxes during the year from the wages of any household employee.

  • Paid total cash wages of $1,000 or more to all household employees in any quarter of the current or prior tax year. [IRC §3121(a)(7)(B)]

Wages Paid by Estate/Trust and Decedent/Beneficiary

If wages are paid during the year by both the estate/trust and either the decedent or beneficiary also paid cash wages during the same calendar, the payments by both are combined in determining if the above thresholds are met. [IRS Notice 95-18, 1995-1 CB 300]


Chapter 8. Penalties

This chapter covers penalties that may be imposed from underpayments of estimated taxes, failure to appropriately apply the consistent basis rules, and accuracy-related penalties. It discusses when payments are required to be made, the burden of proof for both the IRS and taxpayer, and relief from penalties.


Entire Course (including glossary) (130)

As with other taxpayers, trusts and estates face penalties if they fail to comply with the requirements of the tax law. In this chapter, we will look at some of the more significant penalties a tax advisor is likely to encounter when preparing tax returns for a trust or an estate.

Underpayment of Estimated Income Tax

A trust is subject to the same estimated tax rules as an individual. [IRC §6654(l)] However, as will be discussed below, estates have an exemption from these rules until the first tax year ending two or more years after the date of death.

General Rule for Required Payments

Generally, a trust (or estate once it is subject to the rules) is required to pay estimated taxes if it has a tax liability of at least $1,000 or more after taking into account any tax credits or taxes that have been withheld. [IRC §6654(e)(1)]


General Rule for Required Payments (cont'd)

Under the general rule, the estimated taxes due are the lesser of:

Entire Course (including glossary) (131)

90% of the net tax due after taking into account the credits and taxes withheld, or

Entire Course (including glossary) (132)

100% of the tax shown on the return for the preceding year (but only if the prior year was at least 12 months and a return was filed for that year). [IRC §6654(d)(1)]

The 100% exception is modified if the trust's or estate's adjusted gross income exceeds $150,000 for the preceding year. In that case, the percentage of the prior tax that must be paid to qualify for the second test rises to 110% of the tax shown on the prior return (subject to the same 12-month requirement and the requirement that a return must have been filed for that year). [IRC §6654(d)(1)(C)]

Note

Due to the 12-month rule, the “100/110%” safe harbor generally will not be available to a trust for its first two years in existence. In neither case will there be a preceding year of 12 months.


Due Dates for Estimated Taxes


Entire Course (including glossary) (133)

The estimated taxes must be paid in equal installments at each estimated tax due date. [IRC §6654(c)(1)] Generally, the estimated payments are due on the fifteenth day of the fourth, sixth, and ninth month of its tax year, with the final payment due by the fifteenth day of the month immediately following the end of the tax year. [IRC §6654(c)(2)]

Entire Course (including glossary) (134)

The due dates for short tax years are the same as the above. However, the payment due on the fifteenth day of the fourth, sixth, or ninth month need not be made if the short year ends during or before that month. Note that the final payment would still be due by the fifteenth day of the month following the year-end. [Notice 87-32, 1987-1 CB 477]



Exception for Initial Years of Estate or Qualified Revocable Trust

No estimated tax payment is required of an estate until the first tax year that ends more than two years after the date of death of the decedent. [IRC §6654(l)(2)(A)]

A similar exception applies for a trust that, prior to the death of the decedent, was a qualified revocable trust (QRT) treated as a grantor trust (with the assets treated as being wholly owned by the decedent) and to which the remainder of the decedent's estate passes under the decedent's will. [IRC §6654(l)(2)(B)] Thus, the traditional revocable living trust paired with a pour over will (that is, a will where all probate assets are left to the living trust) will qualify for this exception.

Example 8.1

Phillip Mann died on October 23, 2019. His estate selects a September year-end. In this case, no estimated taxes will be due for estate income tax returns filed for the years ended September 30, 2020, and September 30, 2021, since both year dates are less than two years after Phillip's death. If the estate is still open and files a return for the tax year ended September 30, 2022, the estate will be required to make estimated tax payments that year.


Entire Course (including glossary) (135)

No Tax Liability in Prior 12 Months


Entire Course (including glossary) (136)

A trust or estate also qualifies for an exception from the estimated tax requirements if the preceding tax year was one of 12 months and the trust or estate had no tax liability for the preceding year. [IRC §6654(e)(2)]

Entire Course (including glossary) (137)

Initially, it may appear that this exception duplicates the 100%/110% of the prior year's tax rule which also only applies for a prior 12-month year. If the prior year's tax is zero, then 100%, or even 110%, of zero is still zero. However, unlike the 100%/110% test, the no tax liability test does not require that a return be filed for the prior year. Thus, if the trust or estate was not required to file a return in the prior year, it is still protected in the following year.



Penalty Calculation

The underpayment of estimated tax penalty for a trust or estate is computed in the same manner and at the same rates as the underpayment penalty for an individual. [§6654(a)(1)]

As with an individual, an estate or trust may use an annualization calculation to compute a reduced payment for one or more of the first three estimated tax payments. [IRC §6654(d)(2)] A fiduciary is allowed to take into account the distribution deduction for a simple trust in computing the annualized income, regardless of whether the distribution was made during the period in question. [Notice 87-32, 1987-1 CB 477] The deduction is limited to no more than the DNI for the period of months ending the month before the due date of the payment in question.

For a complex trust, the distribution allowed for the period ending the one month before the date is the greater of:

  • The amount of income actually distributed during that period, or

  • The prorated amount of income required to be distributed during the tax year. [Notice 87-32, 1987-1 CB 477]

The penalty for failure to timely pay estimated taxes is reported on Form 2210, Underpayment of Estimated Tax by Individuals, Estates, and Trusts. If the benefits of annualization are claimed by the trust or estate, Form 2210 and Schedule AI, Annualized Income Installment Method, are both filed.


Failure to Pay Tax When Due

A failure to pay penalty of 0.5% is due for each month or portion of a month that tax remains unpaid after the unextended due date of the trust or estate's tax return. [IRC §6651(a)(2)] However, the penalty can be waived if the trustee/executor can show the failure to timely pay the tax was due to a reasonable cause and not willful neglect. Generally, reasonable cause can be shown where:

  • The trust/estate was unable to pay the tax when due even though the trustee/executor exercised ordinary business care and prudence; or

  • Undue hardship would have resulted from paying the tax on the due date. [Reg. §301.6651-1(c)]

    1. If the taxpayer requests a reasonable cause waiver of the penalty, the taxpayer must attach to the return a statement setting out such reasonable cause as required by Reg. §301.6651-1(c)(1).

    2. The failure to pay penalty is capped at 25%. [IRC §6651(a)(2)]

    3. The total amount of this penalty and the failure to timely file penalty cannot exceed 5% for any individual month if both penalties apply (except for the minimum failure to timely file penalty). [IRC §6651(c)(1)]


Failure to File Tax Return (Late Filing) Penalty

A penalty is imposed on a trust or estate that is required to file a tax return if such return is not timely filed. The law provides a penalty of 5% of the tax due for each month, or part of a month, for which a return is not filed up to a maximum of 25% of the tax due. If the return is more than 60 days late, the minimum penalty is the smaller of $435 or the tax due. [IRC §6651(a)(1)-(3)]

Note

Note that the penalty does not apply if a return is filed timely, even if the tax is not paid. Thus, a trust or estate that is unable to pay the tax shown on the return should file the return without the payment. While the failure to pay penalty will continue to run, there will be no late filing penalty, so long as the return itself is timely filed.

As with the failure to pay penalty, this penalty can be waived if reasonable cause for the late filing is shown.

If there is a fraudulent failure to file the return, the penalty increases to 15% per month up to a maximum penalty of 75%. [IRC §6651(f)] A fraudulent failure to file will be generally found if the failure to file the return was part of an attempt to evade tax.


Failure to Provide Information Timely


Entire Course (including glossary) (138)

Per the instructions for Form 1041, Schedule K-1 (Form 1041) must be provided on or before the day you are required to file Form 1041 to each beneficiary who receives a distribution of property or an allocation of an item of the estate. For each failure to provide Schedule K-1 to a beneficiary when due—and each failure to include on Schedule K-1 all the information required to be shown (or the inclusion of incorrect information)—a $280 penalty (2022) may be imposed with regard to each Schedule K-1 for which a failure occurs. The maximum penalty is $3,426,000 (2022) for all such failures during a calendar year. If the requirement to report information is intentionally disregarded, each $280 penalty is increased to $560 or, if greater, 10% of the aggregate amount of items required to be reported, and the $3,426,000 maximum doesn't apply.

The penalty will not be imposed if the fiduciary can show that not providing information timely was due to reasonable cause and not due to willful neglect.


Study Question 22

Which of the following does not accurately describe the general required payment rule for estimated taxes for a trust?

AA trust is not required to make estimated tax payments if it has net tax due of less than $1,000 after taking account any taxes withheld.
BIf a prior tax year was less than 12 months, the prior year tax must be annualized when computing the minimum payment under the 100%/110% of prior year's tax rule.
CThe estimated taxes due are generally required to be paid in four equal installments.
DNo estimated tax penalty will be due, so long as the payments were timely, if the estimates paid are at least equal to 90% of the net tax due after taking into account credits and withholdings.

General

The trust/estate will be subjected to a 20% accuracy-related penalty on accuracy-related misstatements under IRC §6662. This penalty will generally be assessed when the IRS adjusts a tax return to increase the tax due. Of particular interest are the following specific accuracy-related issues named in IRC §6662.

Substantial Understatement of Income Tax

Defined

A substantial understatement of tax for a trust or estate exists when the understatement of tax exceeds the greater of:

  • 10% of the tax required to be shown on the return, or

  • $5,000. [IRC §6662(d)]

Burden of Proof

While the initial burden is on the IRS to show the penalty applies, once the IRS demonstrates that an understatement in excess of these amounts exists, the burden shifts to the taxpayer to show the reason the penalty should not apply.

To view this interactivity please view chapter 8, page 13

Interactivity information:

Exceptions

The penalty does not apply under the following conditions:

  • To the extent the understatement attributable to a non-tax shelter item was from a position for which the taxpayer had substantial authority, as that is defined by Reg. §1.6662-4(d). Authority for these purposes is limited to the source documents outlined in that regulation and does not include any editorial commentary (such as this manual), legal opinions, or articles in various tax-related publications. [IRC §6662(d)(2)(B)(i)]

  • For a non-tax shelter position which has been disclosed (generally on Form 8275, Disclosure Statement, or 8275-R, Regulation Disclosure Statement, or specifically in accordance the applicable forms and instructions) and for which the taxpayer has a reasonable basis. [IRC §6662(d)(2)(B)(ii)]

  • The taxpayer had reasonable cause for taking the position on the return and acted in good faith. [IRC §6664(c)]

Note

One of the standard ways a taxpayer can gain the protection of the reasonable cause defense is to show that he or she sought out the advice of a competent tax professional (other than one associated with marketing any structure) who was provided with all relevant information, and the professional advised the taxpayer the position was acceptable under the tax law. Of course, in such a case it may be found that the advisor had given improper advice and potentially could be subject to preparer penalties under IRC §6694.

Negligence or Disregard of the Rules or Regulations

Even if the understatement does not meet the standard of being substantial, the 20% penalty can still apply if the position was taken due to negligence or disregard of the rules and regulations by the taxpayer. Disregard of the rules includes careless, reckless, or intentional disregard of the rules—the fact the taxpayer was ignorant of the issue isn't necessarily a defense if the taxpayer failed to take reasonable steps to become informed on the matter. [IRC §6662(c)]

A taxpayer who knowingly takes a good faith position contrary to a regulation will still be subject to this penalty if an understatement is found to exist. To protect against that assessment, the taxpayer would generally need to fully disclose the fact that the trust or estate is taking such a position (normally via a Form 8275-R). [Reg. §1.6662-3(b)(2)]

Note

This penalty is generally a more difficult burden for the IRS to sustain since the IRS must do more than merely show that there is an understatement, unlike the case of the substantial understatement penalty. The taxpayer's burden here is merely to show he or she was not negligent and was reasonably diligent in attempting to properly determine his or her tax, unless the taxpayer knowingly took a position contrary to a regulation and failed to properly disclose it.


Consistent Basis Penalty

As was discussed in an earlier chapter, if a trust or estate receives property from a decedent's estate that is subject to the consistent basis reporting rules and fails to report a consistent basis, a penalty under IRC §6662(a)(8)'s “inconsistent basis” rule will be imposed. A 20% penalty under IRC §6662 applies to the extent of any tax understatement arising from the use of a basis on a Form 1041 in excess of the basis determined under IRC §1014(f) and reported to the trust on Form 8971 Schedule A by the estate.

Entire Course (including glossary) (139)

This rule only applies to assets received from estates subject to the consistent basis reporting rules, which generally includes only estates that are required to file a Form 706 and which actually filed the Form 706 after July 31, 2015 or which did not file the Form 706 and its due date was after July 31, 2015. If no Form 706 was required, even if one was filed to elect portability under IRC §2010(c) or as a protective filing, then the consistent basis rules do not apply.



Study Question 23

Which of the following most accurately describes the substantial understatement of income tax penalty?

AThe IRS must demonstrate the taxpayer acted negligently in order to sustain a substantial understatement penalty.
BNo penalty is due if the taxpayer's position had a reasonable possibility of success.
CThe penalty will not apply if the taxpayer has reasonable cause for taking the position and acted in good faith.
DThe penalty is due any time the understatement of tax exceeds $5,000.

Chapter 9. Extended Returns, Amended Returns and Claims for Refund

This chapter discusses considerations for extending returns and amending returns for trusts and estates.


Entire Course (including glossary) (140)

Trusts and estates, like other tax entities, have the ability to request extensions of time to file a return. Similarly, the trust may file a claim for refund and/or amend a previously filed tax return to modify amounts reported or to report additional tax due. As is true with other entities, all of these items are subject to special limitations and rules.


Entire Course (including glossary) (141)

General


Entire Course (including glossary) (142)

Trust and estates generally are required to file their income tax returns by the fifteenth day of the fourth month following the end of their tax year. [Reg. §1.6072-(a)(1)]

A trust or estate is allowed a five and one half month extension of time to file the return if the trust files a request for an extension of time to file on or before the original due date of the return. [Reg. §1.6081-6(a), (b)]

The five and one-half month extension period was contained in the Surface Transportation and Veterans Health Care Choice Act of 2015, as part of an overall restructuring of due dates and extension periods. The Act directed the IRS to revise the extension period for trusts and estates from its prior time period of five months.

Example 9.1


Entire Course (including glossary) (143)

The James Roberts Irrevocable Trust has a calendar tax year. For tax year 2015, the trustee had to file a request for an extension of time to file on Form 7004 by April 15, 2016. The trust then had until September 15, 2016, to file its 2015 trust income tax return.

For the following year, the Form 7004 will again be due by April 15, but the return can then be filed as late as September 30, 2017, and still be treated as timely filed.

The revised due dates and extension dates are meant to provide for earlier filing for returns that have information needed on other returns. The dates selected had a limited (and most likely in the view of trustees) favorable effect on trusts. The end of September extended due date provides time for individual beneficiaries to receive Schedules K-1 for extended trust returns prior to the date the individual return must be timely filed on extension (October 15). However, since the IRS previously had moved trusts to a five-month extension, this change will potentially give individuals less time to deal with trust Schedules K-1.


Form 7004

The request must be made on a timely filed Form 7004, Application for Automatic Extension of Time to File Certain Business Income Tax, Information, and Other Returns, (or any other manner the IRS might later prescribe for doing so) and the trust must show the amount properly estimated as tax for the estate or trust for the year. [Reg. §1.6081-6(b)]

The filing of an extension does not extend the time for payment of tax due for the trust. [Reg. §1.6081-6(c)]

The filing of an extension by the trust or estate does not serve to extend the time for filing the return of any beneficiaries of the estate. [Reg. §1.6081-6(d)]

Rather, any beneficiaries will need to request their own extension of time to file their return.

Entire Course (including glossary) (144)

As a practical matter, a trust or estate filing for an extension of time to file a trust tax return will necessitate the filing of an extension by a beneficiary who will receive a K-1 from the estate or trust. The trust generally should communicate to that beneficiary the expected amounts of income and other items that will be shown on the resulting K-1 to enable the beneficiary to properly estimate the tax due on his/her return.


The IRS has the right to terminate an automatic extension of time by giving 10 days' notice of the termination date. The notice is to be sent to either the address shown on the Form 7004 or the last known address of the trust or estate. [Reg. §1.6081-6(e)]

Note

While the IRS termination of an automatic extension is an extremely unusual event, advisors must be aware that the regulations allow the IRS to do so. Most likely the IRS would take such an action only when the agency believes the trustee is acting in bad faith and using the extension to attempt to make it more difficult or impossible for the IRS to eventually collect the tax that will ultimately be due.


Entire Course (including glossary) (145)

10-Day Grace Period for Termination of Extension

The IRS does not need to grant the 10-day grace period for an extension that was not validly obtained. In GCM 39014 (7/25/83), the IRS Chief Counsel's office noted that an extension is invalid if the estimate of income tax liability reported on the automatic extension request was not based on the information the taxpayer had when he filed for the extension. The IRS should treat the taxpayer as failing to have any extension of time to file and to apply penalties for failure to file.

However, the GCM noted that the mere fact that the tax on the return might differ from that shown on the extension request is not enough by itself to allow the IRS to treat the extension as not valid. The key question is not whether the taxpayer knew that the estimate in question was not proper, but whether the taxpayer knowingly underestimated the tax expected to be due on the return.


Entire Course (including glossary) (146)

Example 9.2


Entire Course (including glossary) (147)

Jack is the trustee of the ABC Trust. Jack is aware that the trust will have a substantial capital gain to report on its current year return. Per the terms of the trust, that gain will be treated as part of trust principal and is not to be distributed to any beneficiary. Nevertheless, in preparing the Form 7004 to request an extension of time to file the tax return for the trust, Jack shows that he expects no tax to be due. In this case, the extension could be held to be invalid by the IRS and a late payment penalty could be assessed against the trust. The problem is that Jack has knowingly understated the tax expected to be due when he completed the Form 7004 and, therefore, failed to comply with the requirements of Reg. §1.6081-6(b)(3).

Advisors should be aware that a failure to inquire about or make a reasonable estimate of the tax due can create liability issues for the advisor. In a case dealing with an advisor's liability to a client for a claimed failure to advise the client of tax that would be due on a return, the failure to show tax on the Form 7004 the advisor knew would be due was found to be partial evidence of fraud, triggering treble damages on the part of the advisor in the case of Haddad Motor Group, 603 F3d 1 (CA1). While that case involved an S corporation, a similar exposure would exist for any advisor who is preparing a Form 7004.

The trustee will complete the top portion Form 7004, giving the name, address, and identifying number. The following code should be listed in Part I, line 1 as follows:

  • 03 – for a bankruptcy estate only

  • 04 – for an estate other than a bankruptcy estate

  • 05 – for a trust

The tax year of the trust or estate will be indicated on line 5a and, if a short tax year is involved, the proper box will be checked on line 5b.

On line 6, the trustee must provide a reasonable estimate of the tax expected ultimately to be due on the return. As was noted above, this number should be based on the information available to the trustee at the time the extension is being requested. As the IRS noted in the GCM cited above, the number will likely be different from the actual tax due since if the trustee actually knew the tax that was going to be due, it's far less likely the trustee would need to request an extension of time to file the return.

On line 7 the trust reports the total payments and credits for the year, and the expected balance due is reported on line 8 by subtracting line 6 from line 7.

Note

While corporations are required to pay the balance to have a valid extension, a trust or estate is not required to pay this amount in order to obtain a valid extension, but the IRS instructions suggest paying as much as the trust or estate can to limit the penalties and interest that will be due. [Instructions for line 8, Form 7004, Revised December 2018, p. 3]


Study Question 24

The Mary Sandy Trust, a simple trust, has a calendar year-end and files for an extension of time to file its return for the year ended December 31, 2022. The extended due date for the Form 1041 is which of the following?

ASeptember 15, 2023
BSeptember 30, 2023
COctober 15, 2023
DNovember 15, 2023

General

While special amended income tax return forms exist for individuals (Form 1040X) and C corporations (Form 1120X), no such special amended return form exists for Form 1041. This does not mean that such returns cannot be revised once they are filed, just that the revision will be made on Form 1041 itself, rather than on a dedicated form.

To prepare an amended Form 1041, a complete, revised Form 1041 is prepared. The return is to be completed just as if the advisor was preparing an original return, only with the revised information contained in the return. [2020 Form 1041 instructions, page 19]

If you are filing an amended Form 1041, check the “Amended return” box in Item F. If you are amending the return for a net operating loss carryback, also check the “Net operating loss carryback” box in item F. [2020 Form 1041 instructions, page 19]

Note

A statement should be attached to the Form 1041, explaining the reason for the amendments, as well as the lines and amounts that were changed on the amended return. [2020 Form 1041 instructions, page 19]

If the amended return results in a change to the K-1 of any or all beneficiaries, a revised K-1 should be prepared with the “Amended K-1” box checked at the top of the Form 1041. A copy of the revised K-1s should be provided to each affected beneficiary. [2020 Form 1041 instructions, page 19]


Statute of Limitations Issues


Entire Course (including glossary) (148)

Federal law generally imposes time limits both on the time period during which the IRS can assess additional tax against the taxpayer and the time period during which the taxpayer can file a claim for refund of overpaid taxes. Such rules apply to trusts and estates in a fashion similar to how they apply to other taxpayers.

Note

Note that the statutes referred to in this topic never refer to limiting the amount of time during which the amounts of income and/or deduction may be questioned by the IRS or revised by the taxpayer. Any item of income or deduction that has an impact on the taxes for a year open for assessment or for a claim for refund may be questioned and/or revised. For example, the IRS may properly question deductions from five years in the past if those deductions would impact a capital loss carryforward being deducted on a return for which the statute of limitations on assessments remains open.


Claims for Refund

A trust or estate looking to claim a refund or credit on an overpayment must file the claim within certain time periods.

If the taxpayer was required to file a return and did file such return, the claim is timely if filed within 3 years of the date the return was filed or 2 years from the time the tax was paid, whichever period expires later. If a return is filed before the initial due date, the return is treated as filed on the initial due date for purposes of the statute rule.

If no return was filed by the trust or estate, the claim must be filed within two years from the time the tax was paid. [IRC §6511(a)]

Example 9.3

The John Newton Trust (located in Texas) files its 2021 return on March 20, 2022, paying $2,500 in taxes. The original due date for the 2021 return was April 15, 2022. The trust has until April 15, 2025, to file a claim for refund as the trust was required to and did file a return, triggering the three-year statute for refund claims.


Net Operating Loss Carryback

If the claim relates to a net operating loss carryback, the period for claiming the refund related to the loss carryback is the later of the period provided above for the year the loss is carried to or the time period ending three years after the time prescribed for filing the return that generated the loss (including any extensions of time granted for filing that return). [IRC §6511(d)(2)] Similar rules apply to carrybacks of credits. [IRC §6511(d)(4)]

Note

The Tax Cuts and Jobs Act of 2017, Section 13302, generally repealed the carryback periods for net operating losses incurred in tax years ending after December 31, 2017 but allows for the carryback of certain net operating losses incurred in the trade or business of farming and for non-life insurance companies only. All other entity types can carry net operating losses forward only.

The Coronavirus Aid, Relief, and Economic Security (CARES) Act of 2020 temporarily reversed the restrictions on what entities were eligible to carry back net operating losses for in tax years 2018, 2019, and 2020 and allowed for a five year carryback period. Under current law NOLs cannot generally be carried back and can only be carried forward indefinitely. An exception applies to certain farming losses, which may be carried back 2 years.

Bad Debt or Worthless Security

If a claim arises from a bad debt or worthless security, the period is extended to seven years. The same expansion to seven years will be applied to a net operating loss carryback generated by the allowance of such a deduction on the original return. [IRC §6511(d)(1)]


Tentative Refund

To claim a refund of taxes for an NOL carryback, taxpayers can file a single tentative refund claim that covers all carryback years instead of an amended return for each carryback year.A tentative refund can be claimed on Form 1045, Application for Tentative Refund. A Form 1045 can be filed by estates or trusts to apply for a quick tax refund resulting from:

  • the carryback of a Net Operating Loss (NOL),

  • the carryback of unused general business credits,

  • the carryback of a net section 1256 contract loss or

  • an overpayment of tax due to a claim of right adjustment under section 1341(b)(1)

A tentative refund must be filed within one year from the end of the loss year and include all necessary documentation to support the requested carryback loss. The refund claims are subject to a 90-day statutory processing time-frame which begins when a complete application was filed or the last day of the month that includes the due date (including extensions) for filing the income tax return for the year in which the loss or credit arose.


Study Question 25

Which of the following statements most accurately describes the statute of limitations rules?

AIf the return is filed late, the IRS generally has three years from the date the return was filed to assess tax on that return.
BThe statute for assessment for a December 31, 2022 calendar year trust tax return filed on March 1, 2023 will expire on March 1, 2026.
CA taxpayer can file a claim for refund arising from carrying back a net operating loss at any time while the statute of limitations for assessment is open on the return generating the loss carryback.
DIn determining whether the six-year statute applies to a return filed for the 2022 tax year, an overstatement of basis is not considered an understatement of gross income.

General


Entire Course (including glossary) (149)

Similar rules operate on the time period the IRS has to assess taxes against a trust or estate. Under the general rule, the IRS has three years from the date the return was filed. [IRC §6501(a)] As was described for refund claims above, if the return is filed before the original due date, it is deemed filed on the original due date for these purposes. [IRC §6501(b)(1)]

Note

If no return is filed by the trust or estate, the IRS may assess tax against the trust or estate at any time. Thus, if the trustee believed that the trust had insufficient income to require filing a return and does not file one, but it is discovered seven years later that, in fact, there was sufficient income to require filing a return, the IRS can assess and collect the tax due. [IRC §6501(c)(3)]


Substantial Understatement

The IRS gains an additional three years during which it may assess tax against the trust or estate if there is a substantial omission of gross income from the return. For these purposes, a substantial omission of income is an amount omitted from gross income on the return that is in excess of 25% of the amount of gross income stated on the return. [IRC §6501(e)(1)(A)]

Note

Effective for returns whose statute of limitations had not closed by July 31, 2015, a substantial understatement of gross income now includes an overstatement of the basis of an asset leading to an understatement of gain and/or an overstatement of loss on the return. Congress enacted this provision to reverse the results of the Supreme Court's ruling in the Home Concrete & Supply case, 109 AFTR 2d 2012-1692 (2012, S.Ct.).

The statute of limitations for assessment of tax by the IRS never closes if the trust or estate either filed a fraudulent return [IRC §6501(c)(1)] or there exists a willful attempt in any manner to defeat or evade tax. [IRC §6501(c)(2)]


Substantial Understatement (cont'd)

The taxpayer and IRS may consent to extend the statute of limitations beyond its expiration date, so long as an agreement is entered into by the parties prior to the expiration of the statute. [IRC §6501(c)(4)(A)] If the IRS requests such an extension, the agency must notify the trustee or executor of his or her right to refuse to extend the statute. [IRC §6501(c)(4)(B)] The agreement is generally executed on a Form 872.

Note

The IRS will request such an extension if the agency has not completed work on an examination and the statute date begins to come near. While initially it may seem there is no reason for a taxpayer to consent to an extension of the statute, a failure to do so will generally lead to an IRS assessment disallowing all items remaining at issue in the exam. Thus, the decision on whether or not to extend the statute is one that will require careful consideration by the trustee, in consultation with the tax advisor.

Note that for the amounts passing through to a beneficiary on a Schedule K-1, the statute of limitation for both assessments and claims for refunds will be tied to the individual return statute. [IRC §6501(a)]


Statute Mitigation Provisions

The IRC provides a limited, and somewhat complex, set of provisions that are meant to allow for mitigation of some (but not all) inequitable results caused by a strict application of the statute of limitations. The rules apply to items that are treated inconsistently, either in the same or different tax years, by related parties.

Such parties that would be relevant in the case of a trust or estate include:

  • A grantor and a trust

  • A grantor and a beneficiary

  • A trust/estate and a beneficiary, legatee, or heir

  • A decedent and the decedent's estate [IRC §1313(c)]

The provisions are triggered to deal with inequities that result in the following:

  • Double inclusion of an item of gross income

  • Double allowance of a deduction or credit

  • Double exclusion of an item of gross income

  • Double disallowance of a deduction or credit [IRC §1312]


Statute Mitigation Provisions (cont'd)


Entire Course (including glossary) (150)

Generally, the rules are triggered where the ability either of the IRS to assess tax or the taxpayer to claim a refund is blocked by the statute of limitations where, say, an amount was included in income and now a related taxpayer finds that the amount is properly reportable on a return of his or hers for which the time period for assessment is still open.

Example 9.4

The Winston Powell Estate erroneously failed to claim $20,000 as an income distribution deduction on its return for the year ending March 31, 2018, reporting it on a K-1 to beneficiary Mary Powell that showed no taxable distribution. After the time period had passed for the estate to file a claim for refund of overpaid tax based on the distribution, but before the statute closed on assessment for Mary's 2018 income tax return, the IRS noticed the mistake and assessed Mary tax on the income that should have been passed through to her on the K-1. The estate, under the mitigation provisions, is able to claim a refund even though the general statute for filing a claim for refund had closed.

To view this interactivity please view chapter 9, page 22

Interactivity information:

Request for Prompt Assessment of Taxes

A complicating factor in dealing with a decedent's estate is the fact that the executor can be liable for the unpaid income taxes of the decedent, as well as any unpaid income taxes of the decedent's estate. Under IRC §6501(d), an executor may file a request for prompt assessment of taxes due, both for any tax of the decedent and any additional income taxes that might be assessed against the decedent's estate.

The request compels the IRS to assess any tax due on those returns within 18-months after the request is filed. The request is valid only for returns which have already been filed by the time the request is made.

Since the 18 months begins with the date the request is filed, the request will not have any impact on taxes for which the statute for assessment is already due to expire before 18 months passes. But making the request in that case does not extend the statute until the end of the 18-month period. That is, the request can only shorten, but not extend, the statute for assessing tax.

The request can be made on IRS Form 4810, Request for Prompt Assessment Under Internal Revenue Code Section 6501(d), or by making a separate letter request containing the required information.

Chapter 10. Grantor Trusts

This chapter provides an overview of grantor trusts, including the triggers of a grantor trust and tax treatment of a grantor trust.


Entire Course (including glossary) (151)

The prior chapters have dealt with trusts that file their own tax returns. However, Congress has created rules to attempt to limit the ability to use trusts when, in reality, the trustor or some other party is really the effective owner of the assets. The grantor trust rules provide that if a trust falls under these rules, the assets will be treated as owned (or partially owned) by the party possessing the prohibited rights.

Grantor trusts are a special category of trusts described in IRC §§671-679. While originally created as a set of rules to prevent the use of trusts to take advantage of lower income tax brackets by spawning a bunch of trusts over which the taxpayer retained full control, the trusts are now often intentionally formed.


Entire Course (including glossary) (152)

General

Subpart E of Subchapter J governs the treatment of grantor trusts. IRC §671 has the general rule for taxation of grantor trusts; §672 has definitions and rules that apply for grantor trusts; §§672-677 provide for specific situations that will result in a trust being treated as a grantor trust; §678 deals with situations where someone other than the grantor will be treated as a substantial owner; and §679 handles foreign trusts.

We will consider the various triggers found in Subpart E that cause the trust to be subjected to the grantor trust rules. Being aware of these rules is important both to assure that a trust does not accidentally become a grantor trust, but also to recognize the use of such provisions (even if they are never actually expected to be used) to create grantor trust income taxation (the intentionally defective grantor trust).

When testing a trust for grantor status, the advisor must be aware that if the trust meets any of the conditions below for being treated as owned by the grantor, the rules are triggered. Any attempts to design a trust that is meant not to be ensnared by the grantor trust rules must take care when devising provisions to avoid one classification, that it does not end up causing problems under a second classification.


Grantor Defined

A grantor, under Reg §1.671-2(e), is a person who either creates a trust or, directly or indirectly, makes a gratuitous transfer to a trust. A gratuitous transfer is a transfer for other than fair market value. This is true even if the transfer is not treated as a gift for gift tax purposes.

If an intentionally defective grantor trust is desired, care must be taken to ensure that the power retained is not one that will trigger inclusion in the grantor's estate. That is, while it is possible to grant powers that will trigger the grantor trust rules for income purposes without triggering inclusion in the grantor's estate, not all (or, in fact, even most) of the powers cited below will achieve this effect.

Note that, despite the use of the term “grantor trust,” the assets of such a trust will be treated as owned (or partially owned) by the party possessing certain prohibited rights, whether that party is the grantor or someone else.


Entire Course (including glossary) (153)

Power to Revoke the Trust or Return Corpus to the Grantor [IRC §676]

This variant of the grantor trust is the one that most advisors are immediately aware of, as revocable living trusts will fall into this category. Specifically, IRC §676(a) states that “[t]he grantor shall be treated as the owner of any portion of a trust, whether or not he is treated as such owner under any other provision of this part, where at any time the power to revest in the grantor title to such portion is exercisable by the grantor or a nonadverse party, or both.”

The power in question is not simply taking ownership back, but also, per Reg. §1.676(a)-1, “a power to revoke, to terminate, to alter or amend, or to appoint.” So a grantor (or nonadverse party) who has the right to appoint beneficiaries is holding a proscribed power that will trigger treatment of the trust as a grantor trust.

Entire Course (including glossary) (154)

The type of grantor trust that most advisors are used to seeing in practice is a revocable living trust, being used to hold title to assets. The reason for setting up the trust will often not be tax related, but rather a program to allow the assets to pass outside of a formal probate. The ability to revoke the trust gives the ability to have the title of the asset revert to the grantor, thus triggering the application of IRC §676(a).



Managerial Powers


Entire Course (including glossary) (155)

Normal managerial powers granted to the trustee will not be deemed to trigger this rule. [Lowenstein v. Commissioner, 3 TC 1133 (1944), acq.] In the Lowenstein case, the grantor served as trustee and had the right to invest trust property, determine distributions, etc. Since his ability to do so was given to him as trustee and had to be exercised in favor of the beneficiaries under state law, the fact that he was given broad discretion did not amount to an indirect power to reinvest principal in himself.

Power to Lend Money to Grantor

In the Estate of Preston, 14 TC 1931 (1950), aff'd, 40 AFTR 304 (1951, CA2), that was true even where the trustee had power to lend money to the grantor, since the trustee had to exercise that power only in favor of the beneficiaries under state law. However, a power of the grantor, not acting as trustee, to borrow without security or to require the trust to purchase assets was deemed to be, effectively, a power to withdraw. [Mather v. Commissioner , 5 TC 1001 (1945)]


Adverse Interest

IRC §672(b) defines any party who is not an adverse party as a nonadverse party. While that definition may seem a bit circular, the point is that any party who has any of restricted powers must be shown to possess an adverse interest. The fact that an individual has a fiduciary responsibility to other beneficiaries (due to being a trustee or co-trustee) will not generally be found sufficient to show an adverse interest to the termination of the trust. [See Witherbee, Mary v. Com ., 13 AFTR 1065 (1934, CA2) aff'g ; Stewart, William , 28 BTA 256 (1933), cert den . 293 US 582 (1934, S.Ct.).] Other factors found not to lead to a finding of an adverse interest include:

  • Moral obligation or duty [Flood v. United States, 24 AFTR 188 (1943, CA1)]

  • Interest to continue earning commissions [Reinecke v. Smith, 12 AFTR 47 (1933, S.Ct.)]

  • Interest as a contingent remainderman [Savage v. Commissioner, 4 TC 286 (1944)]

Generally, there must be a material interest that is adversely affected by the exercise of the power to place the corpus back in the grantor's hands in order for the existence of that power not to trigger the grantor trust rules. As the above summary suggests, showing that adverse interest is not a simple task, so any trust which allows property to be transferred back to the trustor is greatly at risk to be treated as a grantor trust.

Example 10.1

Harry is a non-contingent corpus beneficiary of the trust that was established by Jane. Harry would have an adverse interest if trust principal were to be returned to Jane, thus he would have an adverse interest.

Similarly, a grantor who had the power to appoint an individual who could terminate the trust and receive the corpus was found to have a power to invest corpus in himself. In the case of Pulitzer v. Commissioner, 36 BTA 964, the Court noted that the taxpayer could enter into a contractual arrangement with that person as a condition of appointment that would require the person to turn the assets over to the grantor. Thus, the grantor retained a vehicle via which he could, at any time, get the corpus back by simply appointing a willing person to step into the role of terminating the trust.

A grantor is treated as holding any power held by his/her spouse. That includes the person who was the spouse at the time of the creation of power or interest or who later becomes a spouse of the grantor after creation of the power (for periods after that person becomes the spouse). [IRC §672(e)(1)]

Note

Before the Supreme Court's decision in the case of United States v. Windsor, 133 S.Ct. 2675 (2013), an individual in a same-sex marriage could have appointed their state law (but not federal law) recognized spouse and avoided this provision so long as that spouse was materially affected by any return of trust corpus. Today this is no longer the case, as federal law will recognize the marriage, thus triggering the grantor trust treatment.

As should be clear, any trust that is structured in a manner in which the grantor has direct or indirect access to the trust corpus is going to run afoul of this provision.


Power to Distribute Income to or for the Benefit of the Grantor [IRC §677]

IRC §677(a) provides that a trust shall be treated as owned by the grantor if the income of the trust, without the approval or consent of an adverse party, or in the discretion of the grantor or a nonadverse party may be:

Entire Course (including glossary) (156)

Distributed to the grantor or the grantor's spouse;

Entire Course (including glossary) (157)

Accumulated for future distribution to the grantor or the grantor's spouse; or

Entire Course (including glossary) (158)

Applied to payment of premiums on policies of insurance on the life of the grantor or the grantor's spouse.

Such a distribution need not be directly to the grantor or the grantor's spouse—if the grantor may direct that payment be made to another party, the constructive receipt is treated as enough to trigger this rule. [Reg. §1.677(a)-1(c)]

Example 10.2


Entire Course (including glossary) (159)

Jerry establishes a trust that provides property in the trust will pass at Jerry's death to his son, James. The income of the trust may be distributed to James if the trustee concludes that conditions established in the trust are met.

Jerry is a person who worries about all the things that could go wrong and feels a need for some protection against the unlikely circ*mstance that he would need additional funds somewhere down the line. Consequently, he retains the right to take any income from the trust each year should he find, in his sole judgment, that he needs the funds to maintain his lifestyle. In reality, Jerry never actually exercises this power.

Due to that power, the entire balance of the trust is treated as owned by Jerry and all income of the trust will be taxable to Jerry. The fact he never makes use of the power is not relevant. What is relevant is that he has the power.


Conditional Distributions


Entire Course (including glossary) (160)

There is somewhat conflicting guidance with regard to conditional distributions. In the case of the Estate of Wadewitz, 32 TC 538 (1959), the fact that the income would be distributed to the grantor only if she survived to a date in future was a proscribed accumulation. However, the Eighth Circuit reversed the Tax Court, in the later case of Johnson v. Commissioner, 84 AFTR 2d 99-5306 (1999, CA8), where the earnings of funds held in trust would either be used to pay a liability of the grantor or, if not used for that, then not returned to the grantors. The Eighth Circuit found that the grantor never had and would never receive this income. The opinion did not comment on the fact that a payment of income that may be directed to discharge the grantor's debt is considered a triggering right for use of income under Reg. §1.677(a)-1(d).

Possible Inheritance

However, the possibility that a grantor might receive an interest back by inheritance or as a surviving spouse under a statutory right of election is not considered a prohibited interest for these purposes. [Reg. §1.677(a)-1(c)]


Insurance

The insurance provision deserves special attention. If the grantor creates a funded insurance trust, then the income benefit provision is triggered. The only fact that is relevant is the trust is authorized to use income of the trust to pay for a policy on the life of the grantor or spouse. The Code does not mention any other factor—so the fact that the trust may be irrevocable, the trustee isn't required to make the payments out of income, or that the grantor has not retained any rights to name a beneficiary does not change the fact that the grantor is treated as the owner of the trust for income tax purposes.

However, the income so taxed is limited to the amount that could have been used to pay current year premiums on policies actually in force—not on policies that could have been acquired by the trust. [Iversen v. Commissioner , 3 TC 756] So the mere fact that a trustee has the right to acquire insurance policies or accept them as additions to trust corpus and could use income to pay such premiums does not make the income taxable to the grantor.

Note

Unlike revocable living trusts, treatment under this provision could be unintentionally triggered when the trust is drafted if the parties aren't careful in looking at the trust provisions. Clearly, any trust for which the grantor's spouse is provided a potential interest (such as in the Estate of Wadewitz mentioned earlier) or which is used to provide insurance can trigger this provision. At times that treatment is not an issue (for instance, a trust holding only life insurance likely has no or minimal amounts of other income), but it can be a serious problem if the treatment is not recognized at the time the trust is adopted.


Grantor Retains a More than 5% Reversionary Interest in Trust Property or Income [IRC §673]


Entire Course (including glossary) (161)

A grantor who retains an excess reversionary interest is treated as the owner of the trust under the first of two reversionary trust rules and is the one applicable to trusts currently being established.

Under this rule, any portion of a trust is treated as owned by the grantor if the grantor has a reversionary interest in either the principal or income of that portion of the trust if, as of the inception of that portion of the trust, the value of the interest exceeded 5% of the value of the portion in question. [IRC §673(a)]

In computing that value, it will be assumed that any exercise of discretion available will be used in a fashion to maximize the amount of that interest by being exercised in favor of the grantor. [IRC §673(c)]


Exception for Reversion upon Death of Minor Descendant

An exception to this rule exists for reversionary interests that take effect upon the death of a minor lineal descendant of the grantor. If a lineal descendant is a beneficiary of the trust who holds all present interests in any portion of the trust, the grantor will not be treated as the owner of such an excess reversionary interest to the extent of a reversionary interest in such portion that takes effect upon the death of the beneficiary before that beneficiary attains age 21. [IRC §673(b)]

Intestacy

The Senate Committee Reports also noted that grantor should not be treated as holding an improper reversionary interest merely due to the possibility an interest could revert to either the grantor or the spouse due to intestacy. [Senate Report No. 99-313 (PL 99-514), p. 871]

Postponed Date for Reacquisition

If the grantor postpones the date provided for reacquisition of possession or enjoyment of the property, the tests for a 5% interest should be undertaken as if this were a new transfer to the trust. However, income will not be taken into account by the grantor if the income would not have been includable to the grantor had there been no postponement. [IRC §673(d)] The House Committee Report noted that this was added to deal with the case where the date of the reversionary interest is after the life of an individual and that date is later postponed. [House Report No. 100-795 (PL 100-647) p. 342]


Determining Value of Reversionary Interest

Note that a similar 5% rule is found in the estate tax provisions at IRC §2037(a)(2) for inclusion of transferred property where an “excess” reversionary interest is retained by the donor. It seems prudent to assume the same valuation methods the IRS uses for these purposes (the IRS valuation tables found in the estate and gift tax regulations) will be used to value a reversionary interest under the grantor trust provisions.

The rate used is 120% of the applicable federal midterm rate for the month, compounded annually, rounded to the nearest two-tenths of one percent. [IRC §7520]

For a fixed period of years, this interest rate, along with the number of years to the date of reversion, is used to find the proper factor to be used to compute the interest. Assuming all principal of the trust reverts, if that factor from Table B of the IRS Actuarial Tables is greater than 0.05 (that is, 5%), then the grantor is treated as the owner of the entire trust.

In the case of an interest where a reversion takes place on the death of the beneficiary, Table S of the IRS Actuarial Tables is used. Again, with a reversion of the entire interest, the test looks for a reversion amount below 5%. Remember, however, that if the special rule for a minor who is a lineal descendant is met (which restricts the reversion to occurring before the minor obtains age 21), then this rule does not apply.

Example 10.3


Entire Course (including glossary) (162)

Mary establishes a trust for Harold that provides income to be paid to him for his life. The trust provides that if Harold predeceases Mary, the corpus of the trust reverts to Mary. Harold is age 2 when the trust is funded and the applicable discount rate at the date of the contribution is 2.2%. Per Table S, the factor for a reversionary interest for a 2-year-old would be .20656 at that interest rate. Being greater than 5%, Mary is treated as the owner of the entire trust.

However, if Harold is Mary's grandson and the trust provides the reversion takes place only should Harold die before attaining age 21, then the rule does not apply and Mary will not be taxed on the income of the trust under this rule.

Note that with low interest rates, almost any situation where the entire interest reverts to the grantor will fail the 5% test. With a 2.2% discount rate, even after 60 years the reversionary interest would remain at over 27% (0.270986). Similarly, assuming the requirements for the special minor descendant rule of IRC §673(b) are not met, a reversion of the balance on the death of the beneficiary would still result in a remainder interest of over 20% at that same 2.2% rate.


Reversionary Interest of 10 Years or Less Retained by Grantor (Pre-March 2, 1986 Trusts) [IRC §673]

A different reversionary interest applied to trusts established prior to March 2, 1986. While the number of such trusts is obviously on the decline, advisors may still run into one of these and needs to recognize such a trust.

Under the old short-term reversionary trust rules, a grantor was treated as the owner of a trust if:

  • The grantor retained a reversionary interest in either corpus or income, and

  • As of the inception of the trust, the reversionary interest either did, or was reasonably expected to, take effect within 10 years from the transfer to the trust. [IRC §673(a) prior to 1986 amendments]

The 5% rule replaced this rule and any additions to the trust made after March 1, 1986 are tested under the 5% rule—so the fact a trust was established before 1986 does not grandfather it into the short-term reversionary interest rule for post-1986 transfers.


Reversionary Interest of 10 Years or Less Retained by Grantor (Pre-March 2, 1986 Trusts) [IRC §673] (cont'd)

Such trusts were used prior to 1986 to shift income, most often, to children who were in lower tax brackets. Obviously, the usefulness of such techniques was dealt a serious blow by the enactment of the kiddie tax rules that had income taxed at the parents' marginal tax rates.

Example 10.4

In Example 10.3, the trust that Mary established for Harold involved a reversion that would not be reasonably expected to take place within a 10-year period. Thus, for a transfer that took place prior to March 2, 1986, the trust discussed above would not been treated as owned by Mary without regard to any special rule on minor descendants.

Entire Course (including glossary) (163)

As should be clear from the discussion of the 5% rule above, the 10-year rule made it much easier to retain a reversionary interest in a trust and not have the trust treated as owned by the grantor. Under the 5% rule, generally only a very limited interest may potentially revert to the grantor and not trigger treatment of the grantor as owner for income tax purposes.



Power over Beneficial Interests in the Trust [IRC §674]


Entire Course (including glossary) (164)

IRC §674(a) provides that “[t]he grantor shall be treated as the owner of any portion of a trust in respect of which the beneficial enjoyment of the corpus or the income therefrom is subject to a power of disposition, exercisable by the grantor or a nonadverse party, or both, without the approval or consent of any adverse party.”

Example 10.5

Jeanne establishes a trust that provides for income to be paid for life to her daughter, with the balance of the trust passing to her granddaughter at her daughter's death. However, Jeanne retains a unilateral power to change the beneficiaries of the trust in the future. She retains this right “just in case” her daughter or granddaughter proves to be “unworthy” in her view. The retention of that right is sufficient to cause Jeanne to be treated as owner of the trust.


Powers of Appointment

The powers to be exercised include powers of appointment. The fact that a power is subject to a fiduciary duty and can only be exercised in the interests of the beneficiaries also does not keep the power from causing the trust to be treated as owned by the grantor under these rules. [Reg. §1.674(a)-1]

Note

Note that this is a different rule from being able to invest property in the grantor him/herself. Rather, the issue is the grantor's ability to make changes in income interests for beneficiaries. Thus, an advisor dealing with a trust must be on the lookout for the grantor's power, whether the grantor actually exercises it—and whether that power would be limited by the trustee's fiduciary duties to the beneficiaries.

Power to Amend the Trust

In the case of Gurich v. Commissioner, 8 AFTR 2d 5663 (1961, CA1), the Appeals court found that the grantor's right to amend the trust gave him the right to vary income interests and, thus, run afoul of this provision.


De Facto Control

However, the existence of de facto control of the trust does not expose the grantor to this rule, as the issue is the rights of the grantor under the trust. In the case of Estate of Goodwyn v. Commissioner, TC Memo 1976-238, the fact that the grantor effectively ran the trust with the trustees rubber stamping his decisions with minimal oversight did not cause the trust to be treated as a grantor trust. The court found that despite their actions (or lack thereof), the trustees were still legally responsible for the actions of the grantor and the grantor himself did not possess the required rights.

Note

The result in Estate of Goodwyn, when contrasted with disregard of any limitations imposed by a fiduciary duty, emphasizes that this test is very mechanical in its operation. Even though the grantor may be effectively exercising control (as was the case in Goodwyn), that is not enough to trigger inclusion. Conversely, even though a grantor does not actually exercise the power and would be risking liability for breach of fiduciary duty if he or she did so, the fact that the power exists in the grantor's hands will trigger inclusion.


Administrative Powers under Which Grantor Can or Does Benefit [IRC §675]


Entire Course (including glossary) (165)

Under IRC §675, a grantor is treated as the owner of the trust if the grantor has any of the following powers:

  • Power to deal with the trust for less than full and adequate consideration—this includes the power to deal with any person, not just the grantor, in this fashion. The power does not trigger grantor status if the power can be exercised only with the approval and consent of an adverse party. [IRC §675(1)]

Example 10.6

Olivia establishes a trust, but insists that the trust contain a provision allowing her to require the trust to acquire assets from her son based on terms that she gives to the trust. By retaining that right, Olivia will be treated as the owner of the trust's asset and, therefore, taxable on all income generated by the trust.

(continued on next page)


Administrative Powers under Which Grantor Can or Does Benefit [IRC §675] (cont'd)

  • Power to borrow without adequate interest or security—this power will not count if a trustee other than the grantor is authorized under a general lending power to make loans to any person without regard to interest or security. [IRC §675(2)]

    Loaning to related entities may not work quite as many advisors might expect. In the case of Buehner v. Commissioner, 65 TC 723 (1976), the Tax Court found that a loan to a corporation 100% owned by the grantor did not trigger this provision. The corporation was found to have existed for many years, the loan was reflected on the corporate books, and there was no indication the funds were diverted for the grantor's benefit.

    However, in the case of Bennett v. Commissioner, 9 TC 470 (1982), the Tax Court found that a loan to a partnership did count as indirect loan to the partners, triggering this provision, although loans to a successor corporation were, like the loans in Buehner, not treated as being an indirect loan to the grantor.

    Note

    Because of the complexities of dealing with the loan provisions, normally it is best if the grantor avoids being involved with loans to him/herself or any entity to which he or she is related. If the grantor wishes to go ahead in any event, careful review of the transaction by the tax advisor and legal counsel is highly recommended.


Administrative Powers under Which Grantor Can or Does Benefit [IRC §675] (cont'd)

  • Borrowing of trust funds—if the grantor borrows funds from the trust and does not completely repay the loan before the beginning of the taxable year, the trust is treated as owned by the grantor. This power does not trigger inclusion if the loan (a) has adequate interest and adequate security, and (b) is made by a trustee other than the grantor, who is not related to the grantor or a subservient trustee of the grantor. For purposes of this rule all references to the grantor include the grantor's spouse. [IRC §675(3)]

    Note

    This trigger is unique in that it is activated not by a power contained in the trust, but rather by the operation of the trust. Thus, rather than reviewing the trust document to determine whether the trust must be treated as owned by the grantor, the advisor must look to the actual operation of the trust.

Example 10.7

Henrietta establishes a trust for her infant grandson, Jeremy, and appoints her son, Nathan, as the trustee. The trust is established to provide funds to be used to pay for Jeremy's college education.

A year later Henrietta's home is destroyed in a fire in mid-December. Nathan, being concerned for his mother's welfare, writes a check from the trust to Henrietta for funds so that she can get temporary shelter and replace certain items immediately. Nathan does not charge his mother interest on the loan and requires no security. The loan is not repaid by the beginning of the trust year. The trust is treated as wholly owned by Henrietta.

(continued on next page)


Administrative Powers under Which Grantor Can or Does Benefit [IRC §675] (cont'd)

  • Possession of a specified general power of administration exercisable by any person without the approval or consent of any person in fiduciary capacity. The powers covered by this rule include:

    • Power to vote the stock or other securities of a corporation in which the holdings of the grantor are significant from the viewpoint of voting control;

    • Power to control the investment of trust funds either by directing investments or reinvestments, or by vetoing proposed investments or reinvestments, to the extent that the trust funds consist of stocks or securities of corporations in which the holdings of the grantor and the trust are significant from the viewpoint of voting control; or

    • Power to reacquire the trust corpus by substituting other property of an equivalent value. [IRC §675(4)]

Note

The power to reacquire assets of the trust by substituting property of an equivalent value under IRC §675(4) is one of the most often used powers to create an intentionally defective grantor trust, as including this power, if properly drafted, will not trigger inclusion in the grantor's estate. Quite often there is absolutely no intention to actually make use of the provision in the trust to exchange the assets, but the inclusion of the power creates the intentionally defective grantor trust.

Note

Like other power-based provisions, determining whether a trust has triggered grantor trust treatment under these rules requires the advisor to carefully review the trust document for the existence of one of those proscribed powers and the trust transactions for a proscribed loan.


Trustee, Beneficiary, or Other Person Has the Power to Take Principal or Income for Him/Herself [IRC §678]

If a trust is not otherwise treated as grantor trust to the original grantor under the rules noted above, any person who has power to take principal or income for him/herself may be treated as the owner of the trust pursuant to the grantor trust rules, unless the person renounces or gives up the tainted power—and does not retain any of the powers described above that would normally trigger grantor trust treatment.

Note

These provisions pose a risk to any person involved in a fashion with a trust whenever the trust is not otherwise treated as a grantor trust to the original grantor under one of the provisions discussed earlier. Advisors must realize that, due to the existence of the rules at IRC §678, a person need not be a grantor to be subject to the grantor trust rules.


Trustee, Beneficiary, or Other Person Has the Power to Take Principal or Income for Him/Herself [IRC §678] (cont'd)

This provision, found at IRC §678(a), provides that a person will be treated as the owner of any portion of a trust if either:

  • The person has a power exercisable solely by her/himself to take principal or income from the trust, or

  • The person has previously partially released or modified the power to take from the trust and retains control which would, under the standard grantor trust rules of IRC §§671-677 discussed above, cause a person to be treated as owner of such a portion of the trust if that person had been the grantor.

Note

The idea behind this provision is fairly clear—since the person has (or had) the power to take the property and elected to let it remain in the trust, the party effectively trades places with the grantor. The decision not to take the funds from the trust is similar to a decision to place funds into a trust that the individual established him/herself.

Example 10.8

Nathan is the trustee for a trust that is to provide for the college education of Nathan's son, Jeremy. The trust is funded by Nathan's mother. However, the trust provides that Nathan's brother, Keith, may demand a payment of funds from trust income, trust principal, or both. Regardless of whether Keith exercises that power, Keith will be treated as the owner of the trust.


Trustee, Beneficiary, or Other Person Has the Power to Take Principal or Income for Him/Herself [IRC §678] (cont'd)

IRC §678(b) contains the provision that if another party is treated as the owner of the trust under the other grantor trust rules, then this rule will not apply to this person who has a power over income. In other words, a tie goes to the grantor, since only one party will actually be subject to tax on the income of the trust.

However, the IRS has privately ruled (see Private Letter Ruling 200603040) that a similar result applies in the case of a Crummey trust for the grantor's wife, where the wife had, under the Crummey power, the right to withdraw both principal and income of the trust immediately following the grantor's contribution. The IRS found that because the trust is a grantor trust under IRC §677 (since all income and corpus were payable to the grantor's spouse for life), all income was taxable to the grantor and not to his spouse. The IRS came to a similar conclusion for a non-spouse beneficiary of a similar trust in Private Letter Ruling 200730011.

Note

Thus, despite the letter of the law suggesting that the protection only arises when the power is limited to income, the IRS has expanded the rule to allow it to apply in any case where the grantor would be required to treat the trust as owned by him/herself. Arguably, Congress did not intend for the income to be taxed to multiple individuals, so this provision applies only if no other provision causes the trust to be treated as owned by the original grantor.


Trustee, Beneficiary, or Other Person Has the Power to Take Principal or Income for Him/Herself [IRC §678] (cont'd)

A person other than the grantor who has a proscribed power can avoid the provisions of this rule if the person renounces or disclaims the power within a reasonable time after the person becomes aware of the power. [IRC §678(d)]

Note

Clearly, to take advantage of the provision under IRC §678(d) for a person to avoid this treatment, the individual needs to document when he/she became aware of the issue and ensure that a valid renunciation or disclaimer is executed as soon as possible.

Example 10.9

In Example 10.8, let's assume that Keith was not aware of this power since neither his mother nor Nathan informed him of this power in the trust. Ten years later, Keith's mother dies and Keith is appointed executor of the estate. While reviewing his mother's records, he comes across the trust document and discovers this provision in the trust. In order to avoid being treated as the owner of the trust, Keith must take immediate action to renounce or disclaim the power to avoid being treated as the owner of the trust assets.


Trustee, Beneficiary, or Other Person Has the Power to Take Principal or Income for Him/Herself [IRC §678] (cont'd)

Another special provision applies so that an unintended consequence doesn't occur when establishing trusts for grandchildren. In such cases the grantor may name his/her child (the grandchild's parent) as trustee for the trust whose beneficiary is that person's child. In such a case, the trustee will generally have a legal obligation to support the beneficiary and, since the trust will most likely allow distributions from the trust for the benefit of the beneficiary, this creates a situation where the trustee now has a power to take income from the trust that would effectively reduce the amount the trustee would need to pay to support his/her child.

Since, under these rules, the existence of the power and not the use of it is the key, the trustee under the general rule would be treated as the owner of the entire trust and subject to tax as if the person were the owner. This would be true even if the trustee never actually used any of the trust's income to offset his/her legal obligation to support the grandchild.

IRC §678(c) provides that in such a case where a trustee has the power to apply the income from the trust to the support or maintenance of an individual the trustee is under a personal obligation to support, the rule will not be applied except to the extent that income of the trust is used to provide such support.

Example 10.10

Frank establishes a trust for the benefit of his grandchild, Denise, a minor. Frank's son, Peter, is Denise's father and under applicable state law has a legal obligation to provide for Denise's support. The trust provides that income may be distributed to Denise as necessary to provide for her needs as determined by the trustee. So long as Peter does not actually make a distribution of income to provide for Denise's support, there will be no inclusion of trust income by Peter. But if Peter does make use of the income of the trust to provide for Denise's support, a portion of the trust's income will be taxable to Peter.


Trustee, Beneficiary, or Other Person Has the Power to Take Principal or Income for Him/Herself [IRC §678] (cont'd)

The second prong of the tests (release of power with retained interests that would be a grantor trust interest) creates issues for “five and five” power holders. Under a “five and five” power, a holder is allowed to withdraw the greater of 5% of the value of the trust corpus or $5,000 each year, with the power lapsing if not used in a particular year. If the power is allowed to lapse, an increasing portion of the trust can be treated as owned by the beneficiary per Revenue Ruling 67-241.

Private Letter Ruling 200022035 gives an example of the application of this rule to such a trust. This case involves a decedent's trust with a spouse beneficiary. The spouse had a “five and five” withdrawal right available each year, as well as a lifetime power to appoint all or any part of trust income. The right was noncumulative, so if not exercised, the power would lapse.

The ruling notes that during the time that an annual “five and five” power is outstanding, the beneficiary has the right to take trust principal and income for herself. As such, the beneficiary is taxable on the income of that portion of the trust during the period that option is open.

When the power lapses, the spouse's right to appoint trust income became a problem. While she no longer had the right to take the principal, she could distribute income to herself, a power that if retained by a grantor would create a grantor trust under the provisions of IRC §677. Thus, each year the spouse becomes the owner of increasing amount of the trust, equal to the amount she failed to take.

The ruling also held that if she takes a distribution under the “five and five” power in a later year, it will be deemed to come from her pro rata share of each asset of the trust's corpus she is treated as owning.


Trustee, Beneficiary, or Other Person Has the Power to Take Principal or Income for Him/Herself [IRC §678] (cont'd)

The effect of this is that she picks up her proportionate share of all trust income and deductions each year based on her share of trust corpus computed under these rules. Such items will include her proportionate share of capital gains.

Example 10.11

Laura holds a “five and five” power under a qualified terminable interest (QTIP) trust. For the first year, the trust has trust corpus of $1,000,000. Thus, Laura has a right to withdraw $50,000 (the greater of $5,000 or 5% of trust corpus). When Laura allows this right to lapse, she is treated as the owner of $50,000 of the trust corpus and, therefore, an allocable portion of the trust income. If she continues to allow the power to lapse, over time she will become the owner of an increasing portion of the trust.

IRC §2514(e) provides an exclusion from the gift tax treatment of release of a power of appointment being treated as a taxable gift for lapses of “five and five” powers (with IRC §2041(b)(2) providing a similar estate tax result), thus such options are seen quite often in estate planning trusts. Advisors must remember that although the estate and gift problems are “taken care of” with such a power, they do create some income tax complexity.

In private rulings (see, for example, PLR 9523029), the IRS has attempted to apply similar rules to beneficiaries with Crummey powers. Since often the holder of the Crummey power will be named as a party to which income in the future may be distributed or which the income will be accumulated, the IRS argues that the beneficiary is subject to the grantor trust rules on these amounts.

Note

Regardless of whether the IRS is or is not correct in this view, advisors generally should let clients know about the risk where a Crummey power is released by an income beneficiary of the trust where the trust is likely to show taxable income in the future.


Study Question 26

Which of the following does not accurately describe the grantor trust rules?

AA party other than the one originally transferring assets to the trust may end up being treated as of the owner of some or all of the property in the trust under these rules.
BHaving a trust treated as a grantor trust is an indication of a mistake in the design of the trust.
CA transfer by a grantor trust may nevertheless be treated as a completed gift for transfer tax purposes if certain conditions are met.
DThe grantor trust rules were originally meant to prevent individuals from accessing lower tax rates in a trust while retaining control over the assets.

Study Question 27

Which of the following powers would not cause treatment of the trust principal as being wholly owned by the grantor under IRC §676?

AThe grantor's stockbroker, who has no interest in the trust aside from receiving commissions on the purchase of investments by the trust, has the right to appoint beneficiaries of the trust.
BThe grantor serves as trustee of the trust.
CThe grantor retains the power to appoint a trust protector who has the ability to terminate the trust and receive corpus.
DThe grantor, a year after forming the trust, marries Joan the sole principal beneficiary of the trust who had the right to appoint new principal beneficiaries of the trust.

Study Question 28

Natalie establishes a trust that provides for income to be paid for life to her daughter, Erica, with the balance of the trust passing to her grandson, Nick, at her daughter's death. Natalie retains a unilateral power to change the beneficiaries of the trust in the future. Which of the following most accurately describes this trust?

AAs long as Natalie does not exercise her power to change the beneficiaries, she will not be treated as the owner of the trust.
BIf Natalie can exercise the power only in the interests of the beneficiaries, she will not be treated as the owner of the trust.
CAs long as Natalie does not exercise her power to change the beneficiaries to reclaim the property for herself, she will not be treated as the owner of the trust.
DRetention of the power to change beneficiaries causes Natalie to be treated as the owner of the trust.

Study Question 29

Lisa holds a “five and five” power under a qualified terminable interest trust. For the first year, the trust has trust corpus of $500,000. Which of the following most accurately describes the impact of her choice to allow the power to lapse for the first year?

ALisa has $5,000 of taxable income.
BLisa has $25,000 of taxable income.
CLisa is subject to gift tax on the value of the trust that she did not withdraw.
DLisa is treated as the owner of $25,000 of the trust assets.

General

In some ways grantor trusts were the first “disregarded entity,” recognized years before that concept reappeared in the check-the-box regulations applicable to single member LLCs.

The rules are “anti-abuse” rules that were aimed at removing the ability to use multiple trusts to obtain access to lower tax brackets. Thus, the rules are meant to return the income to the party treated as the grantor so that it will be taxed at the “proper” rate for someone who retains powers that provide for effective retention of some form of control or enjoyment of the assets in the trust.


Entire Course (including glossary) (166)

Income Treated as That of the Grantor

Since the grantor is treated as the owner of that portion of the trust for purposes of income tax (though not necessarily for transfer tax purposes), the grantor ends up reporting his/her share of all income, deductions, credits, etc. that the trust incurs during the grantor's tax year.

Example 10.12

Karen has established a revocable living trust that is treated as a grantor trust since Karen has the right to transfer the trust assets back to herself. The trust receives $10,000 of qualified dividends, pays $5,000 in investment interest expense, and pays $1,000 to an investment advisor. Karen will report the $10,000 of dividends as qualified dividends on Schedule B, include the investment interest paid on Form 4952, Investment Interest Expense Deduction, to compute the amount (if any) of allowable investment interest to be deducted, and include the investment advisor fees as an investment expense on Schedule A and Form 4952 (for years when miscellaneous itemized deductions subject to 2% AGI are allowed). All of these items of income and deduction will be included in computing the net investment tax liability for Karen on Form 8960, Net Investment Income Tax—Individuals, Estates, and Trusts.

Note

For tax years beginning after 2017 and before 2026, the investment advisor fees are not a deductible item since miscellaneous deductions subject to the 2% of AGI limitation are suspended by the Tax Cuts and Jobs Act of 2017.


Transactions Between Grantor and Trust

Similarly, since the grantor is treated as the owner of the trust assets, no gain or loss is recognized on a sale of assets between the grantor and the trust, and no income would be recognized for payments made to the trust (say, for rent of property held by the trust).

Ownership of Grantor Trust

If the trust is only partially a grantor trust (that is, the person is treated as the owner of only a portion of the trust), then only that proportion is reported by the grantor and the remainder is reported using standard trust income tax reporting provisions.

If, as is often the case, the trust is treated as a 100% grantor trust (that is, the grantor is treated as owning 100% of trust corpus), the trust has no income tax liability.

Example 10.13

Return to Karen's trust; the trust will not have any tax liability under the regular income tax rules, alternative minimum tax rules, or the net investment income tax rules. All of the income tax consequences have been transferred to Karen and away from the trust.

Note

The reporting rules related to this matter were discussed in Chapter 1. Regardless of the reporting option used by the trust, the resulting tax on the grantor's return will be the same, as well as the income tax treatment of the trust.


Intentionally Defective Grantor Trust (IDGT)

Note that it is very possible to end up with grantor trust treatment on a trust where the person treated as the owner has no right to withdraw assets from the trust, either currently or in the future—and, in the case of an intentionally defective grantor trust, this is often by design. The fact that the person does not have access to funds from which to pay the tax does not prevent the imposition of the tax.

The danger arises if this situation is not one that arose by design in forming an IDGT or similar vehicle, but rather one where the taxpayer and his/her advisors accidentally run afoul of the grantor trust rules.


Entire Course (including glossary) (167)

Grantor Trust and Exclusion of Gain on Sale of Residence

If a taxpayer is treated as the owner of a trust under the grantor trust rules described above and the trust owns a residence, the taxpayer will be treated as the owner of the residence during the period he or she is treated as owner of the trust holding the residence. [Reg. §1.121-1(c)(3)(i)]

The period will, therefore, count as time the residence was owned by the grantor for purposes of qualifying for the gain exclusion under IRC §121 for sale of a principal residence.

Example 10.14

Walter and Emily, a married couple, establish a joint revocable living trust and transfer their residence into the trust. Five years later, they sell the residence resulting in a $200,000 gain. They lived in the home the entire period it was in the trust; thus, they meet the two-year use requirement. Because they are also the deemed owners of all trust assets under the grantor trust rules, they will also be treated as meeting the two-year ownership rule under IRC §121. Thus, the entire $200,000 gain will be excluded from income.


Grantor Trust and Exclusion of Gain on Sale of Residence (cont'd)

Note that any portion of the gain that is not treated as allocable to the beneficiary under the grantor trust rules will not be eligible for any exclusion of gain under IRC §121, since a trust does not have a principal residence and doesn't “use” the property. IRC §121 creates issues whenever ownership is held by one party or entity, but the use as the residence involves a separate party. The grantor trust rules provide an exception to these rules due to looking through the trust and treating the party as the owner of the trust corpus.

Example 10.15

Instead of having sold the home while held in the revocable living trust, assume that the couple continued to hold the trust through the date that Walter dies. The residence is transferred following Walter's death to a bypass trust that provides income to Emily for life, with the remaining balance of the trust at Emily's death passing to their children. Emily rents the home from the trust and remains living in the home for five more years. At that point the home is sold for a $100,000 gain. In this case the entire $100,000 gain will be treated as a taxable long-term capital gain to the trust.



Entire Course (including glossary) (168)

General

Intentionally defective grantor trusts are an estate planning tool that advisors will encounter from time to time. These trusts take advantage of the fact that the transfer tax rules do not perfectly dovetail with the grantor trust rules for income tax purposes.

A transfer to a trust may be a completed gift and the asset removed from the trustor's estate even though the grantor trust rules treat the asset as continuing to be owned by the grantor. So even though the assets are long gone from the trustor's estate (and direct control), the trustor will pay tax on all income generated by those assets and, as well, recognize any gain or loss on sale.

To view this interactivity please view chapter 10, page 44

Interactivity information:

How IDGTs Work

Initially, advisors wonder what insanity would move someone to intentionally volunteer to pay taxes on assets that are outside the individual's reach. But there is a method to this “madness” due to the position the IRS has taken to date on such arrangements.

An IDGT effectively has trust corpus that is allowed to grow without having either to use trust corpus to pay taxes or to require beneficiaries of the trust to use their own assets to pay the tax burden if the items giving rise to trust income flow onto the beneficiaries' Schedules K-1. Rather, the grantor effectively “subsidizes” the payment of taxes, resulting in an indirect transfer from the trustor to the trust's beneficiaries.

While there is an argument that this represents an indirect gift to the beneficiaries, the IRS has not required recognizing this as a gift for transfer tax purposes.

Study Question 30

Which of the following is not a benefit of using an intentionally defective grantor trust (IDGT) to transfer assets?

AAn IDGT allows the trust corpus to grow without using trust assets to pay taxes.
BAn IDGT permits the grantor to transfer an additional indirect benefit to the trust beneficiaries without incurring gift tax.
CThe grantor reduces the value of her/his estate that would be subject to estate taxes by the amount of income taxes paid on the trust.
DThe grantor bears the burden of paying income taxes on assets that s/he no longer controls.

For federal estate tax purposes, the general rule states that property includible in the decedent's gross estate must be valued at fair market value as of the date of the decedent's death. The alternate valuation is an exception to the general rule, which allows the property in the gross estate to be valued as of the date six months after the decedent's death.

The executor may only elect the alternate valuation if the election decreases the value of the decedent's gross estate, and the sum of the estate tax and the generation-skipping transfer tax imposed on the property.

If property has been distributed, sold, exchanged, or otherwise disposed of within the six months, the property is valued as of the date of ownership change. Property “affected by mere lapse of time” may be valued on date of death or the date of distribution, sale, exchange or other disposition, whichever occurs first.

A contract in which an insurance entity unconditionally undertakes a legal obligation to provide specified pension benefits to specific individuals in return for a fixed consideration or premium. An annuity contract is irrevocable and involves the transfer of significant risk from the employer to the insurance entity. Annuity contracts are also called allocated contracts.

FASB ASC Master Glossary

An annuity is a contract sold by life insurance companies that guarantees a fixed or variable payment to the annuitant (beneficiary of contract) at some future time.

A fixed annuity will be paid out in regular installments (monthly, quarterly, or annually) varying only with the payout method elected.

A variable annuity will pay out a variable amount in regular installments. The amount of payout varies with the value of the account.

All capital and investment proceeds that remain inside the annuity accumulate tax-deferred.

Basis is the amount used by the taxpayer to compute the amount of gain or loss upon sale or disposition of an asset. In most cases, the basis is the carrying value (acquisition cost plus additions or improvements minus accumulated depreciation).

Initial basis is typically the acquisition cost (i.e., fair market value); however, in some cases a new owner uses (assumes) the basis of the asset in the hands of the previous owner (usually lower than fair market value) as with a gift or tax deferred exchange.

A beneficiary is the party for whose benefit a will, a trust, an insurance policy, or a contract is created. The party may be an individual or organization (e.g., charity, school, club, or business). The party may receive title to property by will or by equitable interest in a trust.

The beneficiary may be specified as an income beneficiary (one who receives the income from the assets) or as a principal or remainder beneficiary (one who receives the trust assets after the interest of the income beneficiary terminates—also called the remainderman).

The settlor of a trust can be the beneficiary as long as the settlor is not both the sole beneficiary and the sole trustee.

A bypass trust is a testamentary trust that allows a surviving spouse to receive income for life from the assets of a trust. After the death of the surviving spouse, the trust assets go to other heirs of the deceased. Often those heirs are the children. The trust assets bypass the surviving spouse.

In a family or closely held business, the bypass trust becomes an important estate tax planning tool to minimize the estate and assure that the applicable credit amount is fully utilized.

Consideration is a required element of a contract. It is some act of forbearance or detriment incurred, or benefit or promise thereof, given to and received by the other party in exchange for an act or promise by the second party. Consideration is an element of the contract that supports the promise given, causing the promise to be enforceable. It is the inducement to enter into a contract.

The benefit or detriment need not have a monetary value to be sufficient legal consideration (e.g., the waiving of a legal right would be considered a detriment incurred).

Consideration must be mutual for a valid contract—both parties must give consideration. The relative value of the two considerations, however, need not be equal to be legally sufficient.

Corpus is the assets of a trust (the principal).

An irrevocable trust in which a beneficiary possesses a right to withdraw some or all of the property contributed for a period of time (usually 30 days), after which time the power lapses and the property is governed by the terms of the trust document; the beneficiary's right to withdraw is considered a gift of a present interest for gift tax purposes, thereby qualifying contributions for the gift tax annual exclusion; irrevocable life insurance trusts (see below) are usually Crummey trusts.

A decedent is the deceased individual whose estate is being administered.

A disregarded entity is a business entity having a single owner, which can be either an individual or another business entity. If the entity is disregarded for income tax purposes, its activities are treated as a sole proprietorship (if the owner is an individual) or as a branch or division of the owning entity (if the owner is a business entity).

An amount paid from a retirement plan as a reportable transaction, a distribution can be rolled over, or paid directly to the plan participant (or a beneficiary).

An estate is a taxable, organizational entity used to wind up the affairs and distribute the property of a deceased person. It comes into existence only upon a person's death and holds title to the property of the deceased and exists for a limited time. An estate succeeds to the title of all property of the deceased and is liable for debts. It must pay federal estate tax, applicable state inheritance tax, federal income tax, and any other tax that becomes due on the real and personal property of the estate. (Contrast to trust.)

Administration of the estate is handled by an executor or executrix (if so named in the will and empowered by the court) or a court-appointed administrator (if an executor or executrix is not named in the will or the decedent dies without a will).

Estate taxes are excise taxes levied on the transfer of a person's property on that person's death. There are federal estate taxes and state estate taxes. The amount of the federal estate tax is determined by applying the relevant tax rates to the taxable estate.

An executor is a person named in the will and empowered by the court to administer the decedent's estate, act for the estate, and carry out the terms of the will. An executor is empowered to marshal the assets and pay the debts of the estate, and distribute the remaining assets as specified in the will. An executor is empowered to sell assets to pay debts. It is a fiduciary relationship. An executor has certain powers, duties, and liabilities that are identical to those of administrators.

Exemption is a tax term referring to either income exemption or personal exemption.

Income exemption is a source of income not included in gross income, such as Temporary Assistance for Needy Families, Veterans Administration (VA) disability, workers' compensation, insurance benefits, damages for injury or illness, child support, gifts, inheritance, life insurance proceeds received because of a person's death, and some scholarships.

Personal exemption is the amount of adjusted gross income (AGI) specified by tax law as exempt from taxation for the taxpayer, the spouse if married, and every dependent claimed on the tax return. It is the amount deducted from AGI for every dependent claimed. The amount is indexed for inflation. Note, however, the Tax Cuts and Jobs Act of 2017 reduces the personal exemption to zero for tax years beginning after December 31, 2017, and before January 1, 2026.

A fiduciary is one who holds a position of trust with respect to another party or its property. It is the fiduciary's duty to act selflessly for the benefit of another with undivided loyalty, obedience, and diligence—with due care and in good faith. This is the primary duty of an agent to the principal, of a trustee to the trust, and of an executor to the estate.

Fiduciary accounting income refers to income or loss determined in accordance with state law. Most states have adopted some version of the Uniform Principal and Income Act, which specifies how items are to be characterized by a trust or an estate. Fiduciary account accounting income is distinct from both generally accepted accounting principles and tax accounting rules. Fiduciary tax returns depend, in part, on the application of the fiduciary accounting rules contained in relevant state law.

A generation is a group of contemporaneous individuals, more specifically in a family generation, and would include the offspring having common parentage and constituting a stage of descent. It also may indicate the average time interval between the birth of parents and the birth of their offspring. For purposes of the federal generation-skipping transfer tax, the statute has established 25 years as a generation.

The gift tax is a tax on the transfer of property by one individual to another while receiving nothing, or less than full value, in return. The tax applies whether or not the donor intends the transfer to be a gift. The gift tax applies to the transfer by gift of any type of property. A gift is made if property (including money) is given, or the use of or income from property is given, without expecting to receive something of at least equal value in return. If something is sold at less than its full value or if an interest-free or reduced-interest loan is made, it may be a gift. (Source: https://www.irs.gov/businesses/small-businesses-self-employed/gift-tax)

For federal estate tax purposes, the estate tax is imposed on the taxable estate of a decedent. The gross estate must first be determined in order to calculate the taxable estate. As defined in IRC Section 2031, the gross estate includes the value at time of death of all property, real or personal, tangible or intangible, wherever located, as provided in IRC Sections 2031–2045.

Gross income is all revenues from whatever source derived minus certain exclusions, which are specifically provided by law. It includes both of the following:

  • Earned income (salary, wages, tips, professional fees, business or farm income, and other compensations received for personal services performed)

  • Unearned income (interest, dividends, rents, royalties, capital gains, unemployment, and Social Security benefits)

The following sources of income are not included in gross income:

  • Welfare

  • Veterans Administration (VA) disability

  • Workers' compensation

  • Insurance benefits

  • Damages for injury or illness

  • Child support

  • Gifts

  • Inheritance

  • Life insurance proceeds received because of a person's death

  • Some scholarships

An income beneficiary is a person or legal entity qualified to receive income from an established trust. A trust can designate one or more beneficiaries to share the income based on the percentage assigned to each in the trust document. Simple trusts require that the income be distributed to the beneficiaries. Beneficiaries must report the income allocated to them even if they have not had constructive receipt.

Form K-1 is issued to the beneficiaries to report their share of the trust income or loss.

As distinguished from a bequest or devise, an inheritance is property acquired through laws of descent and distribution from a person who dies without leaving a will. Property so acquired takes as its basis, for gain or loss on later disposition or for depreciation, the fair market value at the date of the decedent's death. An inheritance of property is not a taxable event, but the income from an inheritance is taxable.

An installment is one payment in a series of payments required over time to settle a debt.

Irrevocable means not subject to being invalidated due to withdrawal or cancellation. It is said of trusts, offers and acceptances to contracts (prior to agreement), and offers (and acceptances of offers) to sell (under U.C.C.) in certain circ*mstances and/or under certain contractual specifications.

Trusts are considered to be irrevocable (i.e., the settlor or grantor does not have the right to terminate the trust during the specified time of the trust) unless such right was reserved and specified by the settlor when the trust was established.

Irrevocable offers are the exception to the general rule that offers are revocable by notification to the offeree by the offeror any time prior to acceptance (effective when received by the offeree).

Example

Irrevocable offers include the following:

  • Paid for option (made irrevocable by contract between the parties)

  • Unilateral contract when there is substantial performance by the offeree

  • Stated time of a written offer signed by a merchant (even if there is no return consideration) (U.C.C. 2-205)

A life estate is the name of the property or income interest owned by the life tenant.

A minor is a person not of legal age. Any individual under the age of majority. In most states, the age of majority is 18; in some it is 21. Legal age includes the full right to vote and make contracts, own property directly, and so forth.

In the past, most states maintained a legal age of 21. In 1971, the 26th Amendment to the United States Constitution was ratified. Section 1 states in full: “The right of citizens of the United States, who are 18 years of age or older, to vote shall not be denied or abridged by the United States or by any State on account of age.” Legal voting age is now 18 in all states, but some states delay other rights of majority to age 21.

A partner is an individual, estate, trust, corporation, or other entity that owns a capital or profits interest in a partnership. A partner's taxable income for a tax year includes his distributive share of certain partnership items for the partnership's tax year ending with or within the partner's tax year. A partner must report his distributive share of partnership items on his tax return whether or not it is actually distributed. These items are furnished to the partner on Schedule K-1 (Form 1065). See the instructions for Form 1065 for more information.

Probate is a court procedure that proves a will to be valid or invalid. In current usage, the term has been expanded to include all matters related to the administration of estates.

QTIP is an acronym for qualified terminable interest property trust. The QTIP trust allows assets to be transferred between spouses. Generally created at death, the decedent's will creates a trust whereby the spouse receives trust income for life, payable at least annually if not more frequently. The trust qualifies for the unlimited marital deduction. In addition, the remainder beneficiary(s) of the trust are named by the decedent and cannot be changed by the surviving spouse. The QTIP trust is a vehicle used by the decedent to ensure the ultimate beneficiary(s) of the trust property cannot be changed. In practice, this type of trust can be used to ensure the trust property passes to the decedent's heirs in the event the surviving spouse remarries.

Real property is land and anything erected or growing on or permanently attached to the land (immovables), such as land, buildings, and growing trees, which includes the surface, air rights, and contents of the land.

Real property may be converted into personal property by severance (sand, growing trees, and minerals). Personal property may be converted into realty (fixture) by attachment, such as the construction of structures like buildings, roads, bridges, and fences using cement, bricks, lumber, a central air-conditioning or heating system, etc.

The Secretary of the Treasury publishes interpretations of the law in the form of regulations. They do not have the force and effect of law except in those cases in which the law on a particular subject calls for rules on that subject to be expounded through regulations.

Many regulations are first issued in proposed form, with a designated time period in which practitioners and other interested parties can provide written or oral comments. Because proposed regulations are simply invitations to comment, courts have consistently held that they lack any authoritative weight. Taxpayers must be careful when taking a tax-filing position that contradicts a proposed regulation, however, because the regulation could be issued in temporary or final form without material changes.

A final regulation is the final stage in the process of issuance of regulations after the comments have been received and considered. After release as a final regulation, the regulation has the force of law, subject to the right to challenge an interpretive regulation.

A remainder is an estate that takes effect and may be enjoyed after the expiration of a life estate, or any other intervening interest.

A remainderman is one who receives the trust assets after the interest of the income beneficiary terminates.

The income beneficiary may previously have had a life estate, or any other intervening interest.

A revocable trust is a trust that can be revoked by the person establishing the trust.

A statute of limitations is a legal provision that bars actions at law on contracts unless they are brought within a prescribed period of time. The statute may be tolled (clock is stopped; running is suspended) and begun anew. The U.C.C. statute of limitations is four years.

A surviving spouse is a taxpayer whose spouse died during either of the two taxable years immediately preceding the current taxable year and who remains unmarried and who maintains the home as the household (i.e., provides more than 50% of the support) of a dependent child for the entire year. A surviving spouse will qualify for the special filing status of qualifying widow(er), which uses the same tax rate schedule as married filing jointly.

Testamentary describes a trust, bequest, or gift created by will that takes effect when the creator dies.

A trust is a fiduciary relationship in which one person holds legal title to property subject to an equitable obligation to safeguard or use the property for the benefit of another. A trust represents a separation of legal and equitable title and may be of two types—inter vivos or testamentary. A trust may have different characteristics, such as accumulation, charitable, Clifford, complex, constructive, irrevocable, living, private, or revocable.

A trustee is a party designated (by the trust instrument) or appointed (by the court) to manage the trust or bankruptcy. A trust is a fiduciary relationship. The trustee receives legal title to the property to use for the benefit of the beneficiary of the trust or the creditors of the bankrupt. A trustee can be an individual or organizational entity (e.g., a bank or trust company).

In bankruptcy, a trustee is the representative of the debtor's estate and can operate the business and employ professionals (accountants, attorneys) with prior court approval. A trustee is paid by the estate as part of the administrative priority. (Compensation is limited and must be approved by the court.) A trustee can be elected by the creditors or appointed by the court and can be the debtor, management of the debtor, or an outside third party.

IRC Section 2056 specifies that an unlimited marital deduction is available for computing the taxable estate of an individual where any part of the deceased person's estate passes or has passed to the surviving spouse. For estates of decedents dying after 1981, the deduction is unlimited in amount. Effective for estates of decedents dying after November 10, 1988, the deduction is denied where property passes to a surviving spouse who is not a U.S. citizen [IRC §2056(d)(1)] unless the transfer is made to a qualified domestic trust.

A will is a legal declaration of an individual's intent as to the disposition of property after death. A person who dies with a will is said to die testate; a person who dies without a will is said to die intestate. A will becomes effective only upon the death of the individual (maker). It is ambulatory (i.e., it may be changed by completely rewriting or by codicil or revoked by the maker during his lifetime). A will does not affect the rights of joint tenants (right of survivorship). A will may specify a general (“I leave to [named charity] the sum of $10,000”) or specific (“I leave to my brother our father's gold watch”) legacy (disposition of property under a will). The will is used to devise the testator's property (i.e., stipulate the recipient of property by will).

Welcome to Form 1041 - A Practical Guide. Below is the full list of final exam questions associated with this course. When you launch the final exam for this course, it will contain a randomized subset of the questions below, totaling 50 questions. During the actual final exam, the questions will not appear in the same order as they do below. Note: Each attempt at the final exam will result in a new randomized subset of the questions below. You must earn a score of at least 70.00% in order to pass the exam and receive CPE credit for this course.After you have answered all the questions, select the "Submit Answers" button to receive your score.

Exam Question 1

Which of the following is true regarding a trustor?

AThe trustor transfers property to the trust.
BA trustor always has the right to modify the trust document during his/her life.
CIn a living trust the trustor always retains the right to withdraw all trust assets during his/her life.
DA trustor may not modify the treatment of a cash receipt as principal and income in the trust document in a manner different than that provided under the applicable principal and income act.

Exam Question 2

Who generally sets the rules for computing fiduciary accounting income?

AThe American Institute of Certified Public Accountants
BThe Financial Accounting Standards Board
CThe Internal Revenue Service
DThe state legislature for the state under whose law the trust is governed

Exam Question 3

Which of the following would cause a trust to be treated as a complex trust for income tax purposes for the current tax year?

AThe trust requires that an amount equal to accounting income be distributed to the trust's income beneficiary.
BThe trust distributes the balance of trust principal in its final year.
CThe trust received tax-exempt income during the tax year.
DThe trust was originally funded during the current tax year.

Exam Question 4

Which of the following most accurately describes complex and simple trusts?

AOnce a trust is treated as a complex trust, it is never again treated as a simple trust for tax purposes.
BSimple trusts are allowed a deduction for charitable contributions.
CA trust that makes a distribution from corpus during the year will be treated as a complex trust.
DA trust will generally be treated as a simple trust in the year it terminates.

Exam Question 5

A decedent's estate must file Form 1041 if it has how much gross income?

A$100 or more
B$200 or more
C$500 or more
D$600 or more

Exam Question 6

A grantor trust uses the alternative reporting method whereby the grantor's TIN is provided to reporting entities rather than the TIN of the trust. The grantor provides a completed Form W-9 to the trustee with the grantor's information. Under this filing method, when must the Form 1041 be filed?

ADecember 31 of the tax year
BApril 15 of the following year
CJune 30 of the following year
DForm 1041 is not filed under this method

Exam Question 7

If a calendar year trust files Form 7004, Application for Automatic Extension of Time to File Certain Business Income, Information, and Other Returns , when is the extended deadline?

AApril 15
BJune 30
CSeptember 30
DOctober 31

Exam Question 8

Which of the following is a true statement regarding the taxation of interest and dividends on a fiduciary income tax return?

AQualified dividends on a fiduciary return do not have the availability to be taxed at the lower capital gains rate.
BInterest and dividends received in the year of the decedent's death are reported in their entirety on the Form 1041 of the estate, not Form 1040.
CIf a trust receives municipal bond interest, it must allocate administrative expenses between taxable and tax-exempt income.
DBrokers are required to issue consolidated Form-1099 to trusts and estates by January 31 rather than the February 15 due date for individuals.

Exam Question 9

Which of the following is not automatically an eligible S corporation shareholder?

AA bypass trust with the decedent's spouse as a beneficiary
BA revocable living trust as long as the grantor of the trust meets the requirements to be an S shareholder
CA qualified voting trust
DA trust created primarily to exercise the voting power of the stock that is transferred to the S corporation

Exam Question 10

Which of the following is true regarding a depreciation deduction by a trust?

AIt is allocated in its entirety to an income beneficiary and reported on Schedule K-1.
BIt is allocated in its entirety to the trust on Form 1041.
CIt is allocated to the trust up to the amount of any reserve for depreciation for trust accounting purposes established by the trustee under the terms of the trust or applicable law.
DIt is not allowed.

Exam Question 11

Which of the following is true concerning S corporation shares?

AA trust eligible to be an electing small business trust can hold S Corporation shares only if the beneficiary, not the trustee, elects to do so.
BS corporation shares held by a trust making a §645 election to be combined with the decedent's estate will cause the S election to be terminated.
CA trust established by a decedent's will can hold S Corporation shares for a period of two years from the date shares are transferred to the trust by the decedent's will, without terminating the S election.
DS Corporation shares held after the death of the initial beneficiary of a Qualifying Subchapter S Trust, unless the successor income beneficiary consents to the election within 75 days, will cause the termination of the S election.

Exam Question 12

Which of the following Forms or Schedules is filed on a unique version for trusts/estate income tax returns, rather than the version filed with individual income tax returns?

ASchedule C
BSchedule D
CSchedule E
DForm 4797

Exam Question 13

Which of the following most accurately describes the difference between the treatment of a trade or business for an individual and for a trust or estate?

AA trust or estate is not subject to self-employment tax on income arising from the active conduct of a trade or business.
BAn estate is not subject to the passive activity rules with relation to the operation of a trade or business.
CExpenses of a trade or business are not deductible in computing the adjusted gross income of a trust or estate.
DA trust's or estate's income from a trade or business is never subject to inclusion as net investment income under §1411.

Exam Question 14

Which of the following is not a test that can be used to show that a trust met the material participation standards?

AThe decedent had materially participated in the activity during the portion of the year he/she was alive.
BThe trust participates in the activity more than 1,000 hours during the year.
CThe trust participates more than 100 hours during the year and no other person had more participation in the activity during the year.
DThe trustee's participation in the activity for the year constitutes substantially all of the participation in such activities of all individuals for such year.

Exam Question 15

For a trust or estate, IRC §179:

Adeduction must be entirely allocated to income beneficiaries on a Schedule K-1.
Bdeduction must be deducted on the trust return.
Cdeduction can only be claimed to the extent of net income from a trade or business operated by the trust or estate.
Ddeduction is not available for use.

Exam Question 16

Which of the following is permitted to claim bonus depreciation for inherited property received from a decedent?

AEstates
BTrusts
CThe beneficiaries
DNone of these parties are eligible to claim bonus depreciation in this scenario

Exam Question 17

Which of the following is true regarding a deduction for depreciation by an estate?

AIt is allocated between the trust and beneficiaries based on the income allocated to each.
BIt is allocated entirely to the estate for tax purposes.
CIt is allocated entirely to the beneficiaries for tax purposes.
DIt may be allocated to income beneficiaries only if the executor maintains a reserve for depreciation.

Exam Question 18

Which of the following items can be included in the definition of investment income for purposes of computing the allowed investment interest deduction only if the trust makes a special election?

AQualified dividends
BInterest on corporate bonds
CFarm income
DIncome from a working interest in oil and gas where the taxpayer does not materially participate in the activity

Exam Question 19

Which of the following is true of property taxes?

AForeign real estate taxes are deductible by the trust or estate.
BProperty taxes are not deducted in computing the trust or estate's taxable income.
CProperty taxes may only be deducted on personal property if the tax is an ad valorem tax (that is, a tax based on the value of the property).
DProperty taxes paid in the year of the decedent's death are deductible only on the decedent's final Form 1040.

Exam Question 20

How must “bundled” fees charged by a corporate trustee to the trust be treated?

ABundled fees are by default fully deductible in computing trust adjusted gross income.
BBundled fees may only be treated as itemized deductions subject to the 2% of adjusted gross income limitation.
CBundled fees are to be allocated, using a reasonable method, between expenses allowed in computing adjusted gross income and those not allowed.
DBundled fees are not to be allocated by considering the percentage of the fiduciary's time on investment advice versus other trust matters, such as distributions to beneficiaries.

Exam Question 21

Which of the following does not accurately describe the charitable contribution deduction of a trust or estate computed on Schedule A of Form 1041?

AA trust or estate is limited to a maximum charitable contribution deduction of no more than 60% of the trust or estate's adjusted gross income.
BA deductible charitable contribution must be authorized pursuant to the terms of the governing instrument for the trust or estate.
CA deductible contribution must be made from amounts included in gross income of the trust or estate.
DA trust or estate may elect to claim the deduction in the year prior to the year in which the deductible contribution was made.

Exam Question 22

For 2021, which of the following is true regarding the net operating loss of a trust or estate?

AThe net operating loss deduction can offset 100% of taxable income.
BIt must be carried back, no election to relinquish the carryback is allowed.
CIt cannot arise from a trade or business loss or casualty loss.
DIt must be carried forward, no carryback is allowed.

Exam Question 23

In 2021, which of the following expenses would not be allowed as a deduction on the income tax return of a trust or estate?

AFees paid for the preparation of the fiduciary tax return
BAppraisal fees paid to determine the value of an asset that will be part of trust distributions
CInvestment advisory fees typically charged to investors
DPayment of a fiduciary fees, including fiduciary bond premium

Exam Question 24

Which of the following is not true regarding “accounting income”?

AThe term is defined both by the trust document and by the applicable law governing the trust.
BThis represents the amount that must be mandatorily distributed by terms of any trust that qualifies as a simple trust.
CThe standards for computing accounting income are issued by the Financial Accounting Standards Board.
DThe number most often differs significantly from taxable income of the trust or estate.

Exam Question 25

Which of the following states has adopted the 2000 Uniform Principal and Income Act along with the 2008 amendments to the Act?

ACalifornia
BIllinois
CNew York
DWyoming

Exam Question 26

How are net receipts from an interest in timber treated?

ANet receipts are allocated 10% to income and 90% to principal.
BNet receipts are allocated entirely to income if received under a contract to cut timber from land owned by the trust.
CNet receipts are allocated to income to the extent the amount of timber cut does not exceed the rate of growth during a period there is a beneficiary with a mandatory income interest.
DNet receipts must be accounted for by the trustee under the separate business accounting rule.

Exam Question 27

Which of the following expenditures of a trust or estate is divided equally between principal and income under the provisions of the Uniform Act?

ARegular recurring taxes, such as property taxes imposed on real property held by the trust
BEstate taxes paid by the estate
COrdinary repairs of trust property
DInvestment advisory services

Exam Question 28

Which of the following is true of distributable net income (DNI)?

ADNI is not relevant in determining the income distribution deduction, which is based solely on distributions made to the beneficiaries.
BDNI is only necessary to be computed for a complex trust.
CWhen figuring the income distribution deduction for a simple trust, one needs to know the amount of the trust's distributable net income, reduced by net tax-exempt income for the tax year.
DIf the amount of trust income required to be distributed by the trustee is in excess of DNI, the income distribution deduction is not limited.

Exam Question 29

Which of the following is true regarding DNI and a complex trust?

AThe income distribution deduction does not consider DNI for a complex trust.
BDNI does not consider tax-exempt income for a complex trust.
CIf DNI limits the distribution deduction, it will be absorbed by Tier 1 distributions first.
DAny amounts paid over and above the required distributions are considered Tier II distributions and reduce DNI.

Exam Question 30

Distributions to the beneficiaries consist of all except which of the following?

AA gift or specific bequest under the terms of an inter vivos trust as of the date of the inception of the trust
BIncome required to be distributed currently to beneficiaries under the terms of the trust
CAmounts properly credited to the beneficiary under the terms of the trust
DA distribution from the residuary estate

Exam Question 31

What does the election under IRC §663(b) to treat a qualifying distribution as made in the preceding year require?

AThe consent of affected beneficiaries
BThe consent of the majority of beneficiaries
CThe consent of all beneficiaries
DAn election by the trustee or executor

Exam Question 32

The amount elected to be treated as a prior year distribution under IRC §663(b):

Ashould normally be zero since trusts and estates have more favorable tax brackets than do individuals.
Bis limited to amounts actually paid or credited during the 65 days after the end of the trust's or estate's tax year.
Ccan be more than distributable net income for the tax year in question.
Dmust be consented to by any beneficiary whose K-1 is affected by the election.

Exam Question 33

Which of the following is not a final year deduction which may be distributed to beneficiaries on the final K-1 of a trust or estate?

AExcess deductions on termination
BCapital loss carryovers
CNet operating loss carryover
DCharitable contribution carryover

Exam Question 34

The Smith Trust distributes all of its assets to Bonnie, the sole beneficiary, and terminates on December 31, 2022, when it has a capital loss carryover of $15,000 for transactions during 2022. Bonnie has ordinary income of $50,000 and capital gains of $5,000 for the taxable year 2022. What is Bonnie's capital loss carryover to 2023 after she claims the allowable capital loss for 2022?

A$0
B$5,000
C$7,000
D$10,000

Exam Question 35

The Claire Smithson Trust had taxable income in 2022 of $15,000, all of which consisted of ordinary income (no long-term capital gains or qualified dividends). What is the trust's ordinary income tax for the year?

A$0
B$1,500.00
C$3,812.50
D$5,550.00

Exam Question 36

The alternative minimum tax for a trust or estate is:

Athe excess (if any) of the tentative minimum tax for the trust or estate over the tax computed using the regular tax rates.
Bcomputed on Form 6251.
Ccomputed without regard to an income distribution deduction.
Dimposed at a flat 28% tax rate.

Exam Question 37

The personal exemption for a trust required to distribute all of its income currently is which of the following?

A$0.
B$100.
C$300.
D$600.

Exam Question 38

Net investment income for purposes of the net investment tax imposed by IRC §1411 includes all except which of the following?

AInterest on U.S. government bonds
BCapital gains on the sale of corporate stock
CDistributions received from an individual retirement annuity
DRental income, not from a trade or business

Exam Question 39

For the net investment income tax, the adjusted gross income threshold for a bankruptcy estate is which of the following?

A$0
B$125,000
C$200,000
D$250,000

Exam Question 40

Which of the following is not a trust or estate exempted from the application of the net investment income tax under IRC §1411?

APurely charitable trusts
BDecedent's estate
CQualified retirement plan trusts
DQualified tuition program trusts (529 Plans)

Exam Question 41

Dave Mason died on October 23, 2020. His estate selects a September year-end; therefore, the first fiscal year ends September 30, 2021. His estate will be subject to the requirement to make estimated tax payments beginning for the tax year-ended of which of the following?

ASeptember 30, 2021
BSeptember 30, 2022
CSeptember 30, 2023
DSeptember 30, 2024

Exam Question 42

If more than 60 days late, the minimum penalty imposed on the trust or estate for late filing of a 2022 Form 1041 tax return is the lesser of the amount of unpaid tax on the return or which of the following dollar amounts?

A$1
B$100
C$210
D$435

Exam Question 43

What is the penalty for a fraudulent failure to file a return?

A5% per month with a maximum penalty of 25%
B10% per month with a maximum penalty of 50%
C15% per month with a maximum penalty of 75%
D20% per month with a maximum penalty of 100%

Exam Question 44

Which of the following accurately describes an automatic extension for trusts?

AThe extension is automatically granted if requested timely.
BAs in the case for a corporation, the extension request for a trust must be accompanied by a full payment of the tax expected to be due.
CThe extension is for five months.
DThe filing of an extension for the trust automatically extends the due date of the tax returns of the beneficiaries.

Exam Question 45

The Georgianna Randall Trust files its 2021 return on the due date, April 15, 2022, and pays $3,500 in taxes due with the return. If the trustee wishes to file a claim for refund, she must file the claim no later than which of the following?

AApril 15, 2023
BApril 15, 2024
CApril 15, 2025
DApril 15, 2026

Exam Question 46

The executor for the estate of Thurston Howell, V, files a request for prompt assessment of taxes on November 1, 2022. The IRS must assess any additional taxes within how many months of the date the request was made?

A6 months
B12 months
C18 months
D24 months

Exam Question 47

Which of the following is not considered a power to distribute income to or for the benefit of the grantor under IRC §677?

AA power is exercisable so that a distribution may be made to a grantor or the grantor's spouse
BA possibility exists that the grantor might receive an interest back as a surviving spouse under a statutory right of election
CA power is exercisable by the grantor's spouse to accumulate income for the benefit of the grantor
DA power is exercisable by the grantor to apply the income of the trust to pay premiums on policies of insurance on the life of the grantor or the grantor's spouse

Exam Question 48

Under IRC §673(a), the grantor of a trust is treated as the owner of a portion of that trust if s/he has a reversionary interest in either the principal or income of that portion of the trust if the value of the reversionary interest exceeded how much of the value of the portion in question at the inception of the trust?

A5%
B7.5%
C8.5%
D10%

Exam Question 49

Which of the following does not accurately describe grantor trusts?

AGrantor trusts are treated similarly to single member LLC disregarded entities for income tax purposes.
BThe items of income and deduction allocable to the grantor under these rules are reported as if earned/paid directly by the grantor on their Form 1040.
CTo be treated as a grantor trust, the grantor must withdraw assets from the trust.
DA grantor who has the right to appoint beneficiaries is holding a power that will trigger treatment of the trust as a grantor trust.

Exam Question 50

Which of the following is correct regarding the exclusion of gain from the sale of a principal residence under IRC §121?

AIf a taxpayer is treated as the owner of a trust under the grantor trust rules and the trust owns a residence, the taxpayer will be treated as the owner of the residence during the period he or she is treated as owner of the trust holding the residence and the exclusion of gain under the IRC §121 applies.
BThe IRC §121 exclusion is available to any trust where the property is used as a principal residence by a beneficiary of the trust.
CThe IRC §121 exclusion can be used by a credit shelter trust so long as the surviving spouse of the original owner of the home resides in it.
DThe IRC §121 exclusion is available to trusts, though both two-year periods (the ownership and use periods) start over on the day the property is transferred to the trust.

Entire Course (including glossary) (2024)

FAQs

What is a course glossary? ›

A glossary is a list of terms and definitions, concepts, or themes related to a specific subject. Course developers may use glossary interactions to provide learners with additional context around new or specialized terms.

What is a glossary in a study? ›

A glossary is a collection of words pertaining to a specific topic. In your thesis or dissertation, it's a list of all terms you used that may not immediately be obvious to your reader. In contrast, an index is a list of the contents of your work organized by page number.

What should be included in a glossary of terms? ›

A glossary is a list of terms that traditionally appears at the end of an academic paper, a thesis, a book, or an article. The glossary should contain definitions for terms in the main text that may be unfamiliar or unclear to the average reader.

What is a list of courses called? ›

A course catalog is a document that lists courses taught at a school by discipline, complete with a brief description of each course. In addition to a brief overview of the course, the description will sometimes include the grade level, prerequisites, course length, amount of credit, etc.

What is a glossary example? ›

A glossary example in a science textbook may contain the words: allele, biology, cell, DNA, mitochondria, zygote, etc. Glossaries are a great resource for readers to rely on when learning new content; glossaries help distinguish between words and content-specific words.

What does a glossary entry look like? ›

Glossaries may be formatted in a number of ways, but generally terms are listed in alphabetical order with their definitions, and a line space separates each entry. They are generally placed before notes and works-cited lists and may appear as part of an appendix before those items.

What is the main purpose of glossary? ›

A glossary is a list of terms and their definitions, often found at the end of a book or other document. The purpose of a glossary is to provide definitions for words or phrases that may be unfamiliar to the reader, or that have a specialized or technical meaning within the context of the document.

What is another name for a glossary? ›

A glossary (from Ancient Greek: γλῶσσα, glossa; language, speech, wording), also known as a vocabulary or clavis, is an alphabetical list of terms in a particular domain of knowledge with the definitions for those terms.

Should I include a glossary? ›

Your glossary only needs to include terms that your reader may not be familiar with, and it's intended to enhance their understanding of your work. Glossaries are not mandatory, but if you use a lot of technical or field-specific terms, it may improve readability to add one.

How to make an academic glossary? ›

The basic format for a glossary is a list of words in alphabetical order, each with a definition that explains what it means. Each definition you write should: Set out the meaning of the term using the simplest language possible. Keep your audience in mind here again so you can tailor the terminology used.

What is the correct format for a glossary? ›

Each glossary is an alphabetically arranged list of a subject's terms, with definitions. Each term is followed by one or more explanatory (encyclopedia-style) definitions. (For example, see Glossary of architecture).

What are the requirements of a glossary? ›

Consider the following when developing a glossary:
  • definitions should be clear, concise, and brief,
  • acronyms should be spelled out if used in a definition,
  • stakeholders should have easy and reliable access to glossaries, and.
  • the editing of a glossary should be limited to specific stakeholders.

What should a course include? ›

At a minimum, the syllabus should contain the following: course title, time, and location; prerequisites; required texts and other materials; course topics; major assignments and exams; course policies on grading, academic integrity, attendance, and late work; and contact information for instructor and assistants to ...

What do you call a section of a course? ›

Each course consists of one or more sections often called topics. The course sections (topics) are learning content holders of the course lessons and study material.

What are terms in courses? ›

Semester or Term:

Semester and term are used to identify the formally designated periods during which classes are scheduled.

What is the purpose of a glossary? ›

A glossary is a list of terms and their definitions, often found at the end of a book or other document. The purpose of a glossary is to provide definitions for words or phrases that may be unfamiliar to the reader, or that have a specialized or technical meaning within the context of the document.

How do you write an academic glossary? ›

The basic format for a glossary is a list of words in alphabetical order, each with a definition that explains what it means. Each definition you write should: Set out the meaning of the term using the simplest language possible. Keep your audience in mind here again so you can tailor the terminology used.

What should a course description include? ›

A course description serves to state the rationale for the course and give an overview of key content covered, skills and knowledge to be learned, and how it will benefit the student.

Where is the glossary in a textbook? ›

The glossary is often found at the end of a book or article and is usually in alphabetical order. A glossary can also come at the end of a chapter or even in footnotes.

References

Top Articles
Orange County things to do, starting Aug. 16
Latitude and longitude | Definition, Examples, Diagrams, & Facts
Global Foods Trading GmbH, Biebesheim a. Rhein
Lamb Funeral Home Obituaries Columbus Ga
Katmoie
Don Wallence Auto Sales Vehicles
Comforting Nectar Bee Swarm
Kobold Beast Tribe Guide and Rewards
Phenix Food Locker Weekly Ad
Paula Deen Italian Cream Cake
Slapstick Sound Effect Crossword
Https Www E Access Att Com Myworklife
Paketshops | PAKET.net
Grand Park Baseball Tournaments
Becky Hudson Free
Call Follower Osrs
Blog:Vyond-styled rants -- List of nicknames (blog edition) (TouhouWonder version)
Troy Athens Cheer Weebly
Puretalkusa.com/Amac
Jellyfin Ps5
No Hard Feelings - Stream: Jetzt Film online anschauen
Nhl Tankathon Mock Draft
Curver wasmanden kopen? | Lage prijs
Maxpreps Field Hockey
Www.dunkinbaskinrunsonyou.con
Hannaford To-Go: Grocery Curbside Pickup
Craigslistodessa
Prot Pally Wrath Pre Patch
Craigs List Jonesboro Ar
1145 Barnett Drive
3 Ways to Drive Employee Engagement with Recognition Programs | UKG
Trinket Of Advanced Weaponry
FREE Houses! All You Have to Do Is Move Them. - CIRCA Old Houses
Dentist That Accept Horizon Nj Health
Basil Martusevich
Restaurants Near Calvary Cemetery
Eleceed Mangaowl
Gold Dipping Vat Terraria
2700 Yen To Usd
Skyward Marshfield
Craigslist Odessa Midland Texas
Sallisaw Bin Store
Silicone Spray Advance Auto
Quaally.shop
Paperlessemployee/Dollartree
Craigslist Pet Phoenix
Tanger Outlets Sevierville Directory Map
Powah: Automating the Energizing Orb - EnigmaticaModpacks/Enigmatica6 GitHub Wiki
Marion City Wide Garage Sale 2023
Glowforge Forum
Convert Celsius to Kelvin
Turning Obsidian into My Perfect Writing App – The Sweet Setup
Latest Posts
Article information

Author: Lilliana Bartoletti

Last Updated:

Views: 5902

Rating: 4.2 / 5 (73 voted)

Reviews: 88% of readers found this page helpful

Author information

Name: Lilliana Bartoletti

Birthday: 1999-11-18

Address: 58866 Tricia Spurs, North Melvinberg, HI 91346-3774

Phone: +50616620367928

Job: Real-Estate Liaison

Hobby: Graffiti, Astronomy, Handball, Magic, Origami, Fashion, Foreign language learning

Introduction: My name is Lilliana Bartoletti, I am a adventurous, pleasant, shiny, beautiful, handsome, zealous, tasty person who loves writing and wants to share my knowledge and understanding with you.